You are on page 1of 96

Podre po Imperium Liczb

Cz 1

Liczby wymierne
Andrzej Nowicki

Olsztyn, Toru, 2008

WYM - 15(584) -27.08.2008

Spis treci
Wstp

1 Wstpne informacje o liczbach wymiernych


1.1 Rwnoci z liczbami wymiernymi . . . . . . . .
1.2 Cakowito pewnych liczb wymiernych . . . .
1.3 Przedstawianie liczb wymiernych w szczeglnej
1.4 Podzbiory zbioru liczb wymiernych . . . . . .
1.5 Rne zadania o liczbach wymiernych . . . . .
2 Rozkady jedynki na uamki proste
2.1 Oglne fakty o rozkadach jedynki . . . . . .
2.2 Jedynka i dwa uamki proste . . . . . . . . .
2.3 Jedynka i trzy uamki proste . . . . . . . . .
2.4 Jedynka i cztery uamki proste . . . . . . . .
2.5 Jedynka i pi uamkw prostych . . . . . . .
2.6 Jedynka i sze uamkw prostych . . . . . .
2.7 Jedynka i wicej ni sze uamkw prostych
2.8 Rne fakty o rozkadach jedynki . . . . . .

3.6
3.7
3.8

Rwnanie

1
x
1
x

1
y
1
y

2
z
1
z

.
.
.
.
.

.
.
.
.
.

.
.
.
.
.

.
.
.
.
.

.
.
.
.
.

.
.
.
.
.

.
.
.
.
.

.
.
.
.
.

.
.
.
.
.

.
.
.
.
.

.
.
.
.
.

.
.
.
.
.

.
.
.
.
.

.
.
.
.
.

.
.
.
.
.

.
.
.
.
.

.
.
.
.
.

.
.
.
.
.
.
.
.

.
.
.
.
.
.
.
.

.
.
.
.
.
.
.
.

.
.
.
.
.
.
.
.

.
.
.
.
.
.
.
.

.
.
.
.
.
.
.
.

.
.
.
.
.
.
.
.

.
.
.
.
.
.
.
.

.
.
.
.
.
.
.
.

.
.
.
.
.
.
.
.

.
.
.
.
.
.
.
.

.
.
.
.
.
.
.
.

.
.
.
.
.
.
.
.

.
.
.
.
.
.
.
.

.
.
.
.
.
.
.
.

.
.
.
.
.
.
.
.

.
.
.
.
.
.
.
.

.
.
.
.
.
.
.
.

.
.
.
.
.
.
.
.

.
.
.
.
.
.
.
.

.
.
.
.
.
.
.
.

.
.
.
.
.
.
.
.

.
.
.
.
.
.
.
.

12
12
13
13
14
14
14
15
16

proste
. . . . .
. . . . .
. . . . .
. . . . .

.
.
.
.

.
.
.
.

.
.
.
.

.
.
.
.

.
.
.
.

.
.
.
.

.
.
.
.

.
.
.
.

.
.
.
.

.
.
.
.

.
.
.
.

.
.
.
.

.
.
.
.

.
.
.
.

.
.
.
.

.
.
.
.

.
.
.
.

.
.
.
.

.
.
.
.

.
.
.
.

.
.
.
.

.
.
.
.

17
17
17
18
21

. . . . . . . . . . . . . . . . . . . . . . . . . . . . . . . . . . . .

23

3 Rozkady liczb wymiernych na uamki


3.1 Rozkady liczb naturalnych . . . . . .
3.2 Rozkady liczb wymiernych . . . . . .
3.3 Sumy dwch uamkw prostych . . .
3.4 Rwnanie x1 + y1 = z1 . . . . . . . . .
3.5

. . . . .
. . . . .
postaci
. . . . .
. . . . .

5
5
6
9
9
10

1
t

Rwnanie + = +
. . . . . . . . . . . . . . . . . . . . . . . . . . . . . . . . . .
Sumy trzech uamkw prostych . . . . . . . . . . . . . . . . . . . . . . . . . . . . . . .
Zadania rne . . . . . . . . . . . . . . . . . . . . . . . . . . . . . . . . . . . . . . . .

4 Odwrotnoci wyrazw pewnych cigw


4.1 Niecakowito sumy odwrotnoci wyrazw cigu
4.2 Odwrotnoci wyrazw cigu arytmetycznego . .
4.3 Odwrotnoci kolejnych liczb naturalnych . . . .
4.4 Naprzemienne sumy uamkw prostych . . . . .
4.5 Odwrotnoci liczb pierwszych . . . . . . . . . . .
4.6 Odwrotnoci liczb potgowych . . . . . . . . . .
4.7 Odwrotnoci liczb kwadratowych . . . . . . . . .
4.8 Odwrotnoci liczb trjktnych . . . . . . . . . .
4.9 Odwrotnoci szecianw . . . . . . . . . . . . . .
4.10 Granice . . . . . . . . . . . . . . . . . . . . . . .

24
24
28

.
.
.
.
.
.
.
.
.
.

29
29
29
30
32
33
33
34
35
36
37

5 Rozwinicia dziesitne liczb wymiernych


5.1 Tablice rozwini dziesitnych pewnych liczb wymiernych . . . . . . . . . . . . . . . .
5.2 Okresy rozwini dziesitnych liczb wymiernych . . . . . . . . . . . . . . . . . . . . .
5.3 Rne zadania o rozwiniciach dziesitnych liczb wymiernych . . . . . . . . . . . . . .

39
39
41
42

.
.
.
.
.
.
.
.
.
.

.
.
.
.
.
.
.
.
.
.

.
.
.
.
.
.
.
.
.
.

.
.
.
.
.
.
.
.
.
.

.
.
.
.
.
.
.
.
.
.

.
.
.
.
.
.
.
.
.
.

.
.
.
.
.
.
.
.
.
.

.
.
.
.
.
.
.
.
.
.

.
.
.
.
.
.
.
.
.
.

.
.
.
.
.
.
.
.
.
.

.
.
.
.
.
.
.
.
.
.

.
.
.
.
.
.
.
.
.
.

.
.
.
.
.
.
.
.
.
.

.
.
.
.
.
.
.
.
.
.

.
.
.
.
.
.
.
.
.
.

.
.
.
.
.
.
.
.
.
.

.
.
.
.
.
.
.
.
.
.

.
.
.
.
.
.
.
.
.
.

.
.
.
.
.
.
.
.
.
.

.
.
.
.
.
.
.
.
.
.

6 Przystawanie modulo m dla liczb wymiernych


6.1 Definicje . . . . . . . . . . . . . . . . . . . . .
6.2 Przystawanie i mianowniki . . . . . . . . . . .
6.3 Przystawanie i dodawanie . . . . . . . . . . . .
6.4 Przystawanie jako relacja rwnowanoci . . .
6.5 Przystawanie i mnoenie . . . . . . . . . . . .
6.6 Przystawanie modulo 2 . . . . . . . . . . . . .
6.7 Przystawanie modulo pk . . . . . . . . . . . .

.
.
.
.
.
.
.

.
.
.
.
.
.
.

.
.
.
.
.
.
.

.
.
.
.
.
.
.

.
.
.
.
.
.
.

.
.
.
.
.
.
.

.
.
.
.
.
.
.

.
.
.
.
.
.
.

.
.
.
.
.
.
.

.
.
.
.
.
.
.

.
.
.
.
.
.
.

.
.
.
.
.
.
.

.
.
.
.
.
.
.

.
.
.
.
.
.
.

.
.
.
.
.
.
.

.
.
.
.
.
.
.

.
.
.
.
.
.
.

.
.
.
.
.
.
.

43
43
44
45
45
46
46
47

7 Podzielno dla liczb wymiernych


7.1 Rozkad kanoniczny liczb wymiernych . . . . . . . . . .
7.2 Relacja podzielnoci w Q . . . . . . . . . . . . . . . .
7.3 Nwd i nww dla liczb wymiernych. Definicje i przykady
7.4 Nwd i nww dla liczb wymiernych. Wasnoci . . . . . .
7.5 Wzgldnie pierwsze liczby wymierne . . . . . . . . . . .

.
.
.
.
.

.
.
.
.
.

.
.
.
.
.

.
.
.
.
.

.
.
.
.
.

.
.
.
.
.

.
.
.
.
.

.
.
.
.
.

.
.
.
.
.

.
.
.
.
.

.
.
.
.
.

.
.
.
.
.

.
.
.
.
.

.
.
.
.
.

.
.
.
.
.

.
.
.
.
.

.
.
.
.
.

49
49
49
50
51
52

8 Twierdzenie Wolstenholme i jego uoglnienia


8.1 Wspczynniki Ai . . . . . . . . . . . . . . . . . . . . . . . . .
8.2 Wspczynniki Ai dla liczb pierwszych . . . . . . . . . . . . .
8.3 Zastosowania dla liczb pierwszych i iloczynw . . . . . . . . .
8.4 Sumy odwrotnoci iloczynw . . . . . . . . . . . . . . . . . . .
8.5 Odwrotnoci liczb wzgldnie pierwszych: podstawowe fakty . .
8.6 Odwrotnoci liczb wzgldnie pierwszych: Twierdzenia Gessela
8.7 Twierdzenie Wolstenholme i inne twierdzenia . . . . . . . . . .
8.8 Rne zadania . . . . . . . . . . . . . . . . . . . . . . . . . . .

.
.
.
.
.
.
.
.

.
.
.
.
.
.
.
.

.
.
.
.
.
.
.
.

.
.
.
.
.
.
.
.

.
.
.
.
.
.
.
.

.
.
.
.
.
.
.
.

.
.
.
.
.
.
.
.

.
.
.
.
.
.
.
.

.
.
.
.
.
.
.
.

.
.
.
.
.
.
.
.

.
.
.
.
.
.
.
.

.
.
.
.
.
.
.
.

.
.
.
.
.
.
.
.

53
53
54
55
57
58
60
61
64

9 Liczby postaci x1 /x2 + x2 /x3 + + xs /x1


9.1 Podstawowe wasnoci zbiorw Bs i As . . . . . . . . . . . . . . . . .
9.2 Zbir B2 . . . . . . . . . . . . . . . . . . . . . . . . . . . . . . . . . .
9.3 Zbir B3 i liczby (a3 + b3 + c3 )/abc . . . . . . . . . . . . . . . . . . .
9.4 Nieskoczono zbioru A3 . . . . . . . . . . . . . . . . . . . . . . . . .
9.5 Przykady liczb naturalnych nalecych do A3 . . . . . . . . . . . . .
9.6 Wystpowanie danej liczby w rozkadach liczb ze zbioru A3 . . . . . .
9.7 Zbir B3 . . . . . . . . . . . . . . . . . . . . . . . . . . . . . . . . . .
9.8 Liczby postaci x/y + y/z + z/x, gdzie x, y, z s liczbami cakowitymi
9.9 Zbir A4 . . . . . . . . . . . . . . . . . . . . . . . . . . . . . . . . . .

.
.
.
.
.
.
.
.
.

.
.
.
.
.
.
.
.
.

.
.
.
.
.
.
.
.
.

.
.
.
.
.
.
.
.
.

.
.
.
.
.
.
.
.
.

.
.
.
.
.
.
.
.
.

.
.
.
.
.
.
.
.
.

.
.
.
.
.
.
.
.
.

.
.
.
.
.
.
.
.
.

66
66
69
70
72
73
77
79
83
85

.
.
.
.
.
.
.

.
.
.
.
.
.
.

.
.
.
.
.
.
.

.
.
.
.
.
.
.

Spis cytowanej literatury

88

Skorowidz

91

ii

Wstp
Gwnym tematem prezentowanej serii ksiek s liczby i ich przerne wasnoci. Autor
od najmodszych lat zbiera wszelkie fakty i ciekawostki dotyczce najpierw liczb cakowitych
i wielomianw o wspczynnikach cakowitych, a nastpnie dotyczce rwnie liczb wymiernych, rzeczywistych, zespolonych oraz wielomianw nad tymi zbiorami liczbowymi. Nazbierao si sporo interesujcego materiau, ktrego wybrane fragmenty bd tu przedstawione.
Materia pochodzi z wielu rnych rde. S tu zadania i problemy, ktre znajdziemy w
popularnych czasopismach matematycznych. Wrd tych czasopism jest wychodzce od 1894
roku (przewanie 10 numerw w roku) The American Mathematical Monthly. S wrd tych
czasopism rwnie: angielskie czasopismo Mathematical Gazette, , kanadyjskie Crux Mathematicorum, rosyjskie Kwant, chiskie Matematical Excalibur, itp. Godnymi uwagi s rwnie
polskie czasopisma popularno-naukowe: Delta, czasopismo dla nauczycieli Matematyka oraz
inne.
Istotn rol w prezentowanym materiale odegray zadania z olimpiad i konkursw matematycznych caego wiata. Kadego roku pojawiaj si opracowania, ksiki oraz artykuy
dotyczce zada z rnych zawodw matematycznych. Wspomnijmy tylko o prestiowych
seriach ksiek z zawodw International Mathematical Olympiad oraz Putnam Mathematical
Competition. Sporo oryginalnych zada znajduje si w opracowaniach dotyczcych olimpiad
matematycznych w Rosji lub w pastwach byego Zwizku Radzieckiego. Polska rwnie ma
wartociowe serie tego rodzaju ksiek.
Zebrany materia pochodzi rwnie z rnych starych oraz wspczesnych podrcznikw
i ksiek z teorii liczb. Wykorzystano liczne ksiki popularno-naukowe oraz prace naukowe
publikowane w rnych czasopismach specjalistycznych. S tu te pewne teksty pochodzce
z internetu.
Wikszo prezentowanych faktw ma swoje odnoniki do odpowiedniej literatury. Odnoniki te wskazuj tylko wybrane miejsca, w ktrych mona znale albo informacje o danym
zagadnieniu, albo rozwizanie zadania, albo opowiedni dowd. Bardzo czsto omawiany temat
jest powtarzany w rnych pozycjach literatury i czsto trudno jest wskaza oryginalne rda.
Jeli przy danym zagadnieniu nie ma adnego odnonika do literatury, to oznacza to, e albo
omawiany fakt jest oczywisty i powszechnie znany, albo jest to wasny wymys autora.
Elementarna teoria liczb jest wspaniaym rdem tematw zachcajcych do pisania
wasnych programw komputerowych, dziki ktrym mona dokadniej pozna badane problemy. Mona wykorzysta znane komputerowe pakiety matematyczne: MuPad, Mathematica,
CoCoA, Derive, Maple i inne. W prezentowanej serii ksiek znajdziemy sporo wynikw i
tabel uzyskanych gwnie dziki pakietowi Maple.
We wszystkich ksikach z serii Podre po Imperium Liczb stosowa bdziemy jednolite oznaczenia. Zakadamy, e zero nie jest liczb naturaln i zbir {1, 2, 3, . . . }, wszystkich
liczb naturalnych, oznaczamy przez N. Przez N0 oznaczamy zbir wszystkich nieujemnych
liczb cakowitych, czyli zbir N wzbogacony o zero. Zbiory liczb cakowitych, wymiernych,
rzeczywistych i zespolonych oznaczamy odpowiednio przez Z, Q, R oraz C. Zbir wszystkich
liczb pierwszych oznaczamy przez P.
1

Najwikszy wsplny dzielnik liczb cakowitych a1 , . . . , an oznaczamy przez nwd(a1 , . . . , an )


lub, w przypadkach gdy to nie prowadzi do nieporozumienia, przez (a1 , . . . , an ). Natomiast
najmniejsz wspln wielokrotno tych liczb oznaczamy przez nww(a1 , . . . , an ) lub [a1 , . . . , an ].
Zapis a | b oznacza, e liczba a dzieli liczb b. Piszemy a - b w przypadku, gdy a nie dzieli
b. Cz cakowit liczby rzeczywistej x oznaczamy przez [x]. Jeli m jest liczb naturaln, to (m) jest liczb wszystkich liczb naturalnych mniejszych lub rwnych m i wzgldnie
pierwszych z liczb m.
Pewne zamieszczone tutaj fakty przedstawione s wraz z ich dowodami. Pocztek dowodu
oznaczono przez D.. Pojawiaj si rwnie symbole R., U., W. oraz O. informujce odpowiednio o pocztku rozwizania, uwagi, wskazwki i odpowiedzi. Wszystkie tego rodzaju
teksty zakoczone s symbolem . Skrt Odp. rwnie oznacza odpowied.
Spis cytowanej literatury znajduje si na kocu tej ksiki. W pewnych podrozdziaach
podano rwnie literatur dodatkow lub uzupeniajc. Informuje o tym symbol F.

ooooo
Pierwsza ksika z serii Podre po Imperium Liczb powicona jest liczbom wymiernym, czyli liczbom postaci ab , gdzie a i b s liczbami cakowitymi, przy czym b jest rne od
zera.
Ksika ta skada si z dziewiciu rozdziaw. Trzy z tych rozdziaw (rozdziay 2, 3 i
4) s o liczbach wymiernych postaci n1 , gdzie n jest liczb naturaln. Takie liczby nazywaj si uamkami prostymi lub uamkami egipskimi. W rozdziaach tych badane s gwnie
zagadnienia zwizane z rozkadami liczb wymiernych na skoczon sum uamkw prostych.
Uamkami prostymi zajmujemy si rwnie w rozdziale 8. W 1862 roku J. Wolstenholme
udowodni, e jeli p > 5 jest liczb pierwsz, to licznik uamka
a
b

1
2

=1+

1
3

1
p1

+ +

jest podzielny przez p2 . Przedstawiamy dwa rne dowody tego faktu. W rozdziale 8 podajemy
przerne warianty twierdzenia Wolstenholme oraz zajmujemy si problemami stowarzyszonymi z tym twierdzeniem. Gwn motywacj do powstania tego rozdziau bya pikna praca
I. M. Gessela, opublikowana w 1998 roku w The American Mathematical Monthly. Rozdziay
6 i 7 peni funkcj pomocnicz. Przedstawiamy w nich zagadnienia potrzebne do zrozumienia
pewnych dowodw z rozdziau 8.
Kada liczba rzeczywista ma swoje dokadnie jedno nieskoczone rozwinicie dziesitne.
Dana liczba rzeczywista jest wymierna wtedy i tylko wtedy, gdy jej nieskoczone rozwinicie
dziesitne jest od pewnego miejsca okresowe. Pewnym problemom zwizanym z tym zagadnieniem powicony jest rozdzia 5.
W ostatnim rozdziale zajmujemy si dodatnimi liczbami wymiernymi postaci
x1
x2

x2
x3

+ +

xs1
xs

xs
x1 ,

gdzie x1 , . . . , xs s liczbami naturalnymi. Godnym uwagi jest fakt, e jeli s = 12, to kada liczba naturalna wiksza od 11 ma powysz posta. Udowodni to w 2000 roku A. V.
Bondarenko.
Zamy teraz, e s = 3 i rozpatrzmy zbir wszystkich liczb naturalnych n takich, e
n=

x
y

y
z

+ xz ,

dla pewnych liczb naturalnych x, y, z. Oznaczmy ten zbir przez A3 . Zauwamy, e liczby 3,
5 i 6 nale do zbioru A3 . Mamy bowiem:
3=

1
1

1
1

+ 11 ,

5=

1
2

2
4

+ 41 ,

6=

2
12

12
9

+ 92 .

Czy liczba 4 naley do zbioru A3 ? Odpowied na to pytanie przez dugi czas nie bya znana.
Rozstrzygn ten problem dopiero w 2000 roku wspomniany wyej Bondarenko. Udowodni
on, e 4 do tego zbioru nie naley.
Mona udowodni, e do zbioru A3 naley nieskoczenie wiele liczb naturalnych. Mona
rwnie udowodni (i to nie jest a tak bardzo skomplikowane), e zbir A3 pokrywa si ze
zbiorem wszytkich liczb naturalnych postaci
a3 +b3 +c3
,
abc

gdzie a, b, c N.

Czytelnika zainteresowanego tego rodzaju zagadnieniami zapraszamy do ostatniego rozdziau.


3

Wstpne informacje o liczbach wymiernych

Kad liczb postaci ab , gdzie a i b s liczbami cakowitymi oraz b 6= 0, nazywamy liczb


wymiern. Zbir wszystkich liczb wymiernych oznaczamy przez Q.
oooooooooooooooooooooooooooooooooooooooooooooooooooooooooooooooooooooo

1.1

Rwnoci z liczbami wymiernymi

oooooooooooooooooooooooooooooooooooooooooooooooooooooooooooooooooooooo
1.1.1.

925
6+10

9
6

1.1.2.

1
4

16
64

166
664

1666
6664

= .

([Kw] 2/88 42).

1.1.3.

1
5

19
95

199
995

1999
9995

= .

([Kw] 2/88 42).

1.1.4.

4
8

49
98

499
998

4999
9998

= .

([Kw] 2/88 42).

1.1.5.

26
53

286
583

25
10 ,

2886
5883

12164
55+40

28886
58883

121
55

64
40 ,

= .

850
2+5

8
2

50
5 . ([Kw] 9/72 21).

([KoM] Gy1959).

1.1.6. Zapis a1 a2 . . . as oznacza liczb naturaln, ktrej kolejnymi cyframi s odpowiednio


a1 , a2 , . . . , as . Jeli a, b, c s takimi cyframi, e ab/bc = a/c, to
a/c = ab/bc = abb/bbc = abbb/bbbc = .
([Fom] 14/64).
1010 1 0101
1.1.7 (N. Anning). Dany jest uamek 1100
1 0011 zapisany w dowolnym systemie numeracji.
Jeli w liczniku i mianowniku rodkow cyfr 1 zastpimy dowoln nieparzyst liczb nastpujcych po sobie jedynek, to uamek ten nie zmieni wartoci. ([S64] 159).

1.1.8.

(db)(dc)
(ab)(ac)

1.1.9.

14
13

1.1.10.

23 1
23 +1

24
35

34
57

33 1
33 +1

1.1.11. Jeli x =
1.1.12. Jeli x =
([BaL] 154).

(dc)(da)
(bc)(ba)

n4
(2n1)(2n+1)

+ +

bc
1+ab ,
ab
a+b ,

1003 1
1003 +1

y=
y=

(da)(db)
(ca)(cb)

([BaL] 145).

n(n+1)(n2 +n+1)
6(2n+1)

dla n N.

([Mat] 3/52 49).

3367
5050 . ([Fom] 18/91).

ca
1+ca ,

bc
b+c ,

= 1.

z=
z=

ab
1+ab ,

ca
c+a ,

to x + y + z = xyz.

([BaL] 146).

to (1 + x)(1 + y)(1 + z) = (1 x)(1 y)(1 z).

Liczby wymierne.
2

1. Wstpne informacje o liczbach wymiernych


2

c
b
1.1.13. Niech x = a +b
, y = a +c
, z = b +c2bca , gdzie a, b, c N. Jeli x+y+z =
2ab
2ac
1, to dwie z liczb x, y, z s rwne 1, a pozostaa 1. ([OM] Leningrad 1982).

1.1.14. Jeli

1
a

1.1.15. Jeli

a1
b1

1
b

+
a2
b2

1
c

a3
b3

1
a+b+c ,

to

1
b2n+1

1
c2n+1

1
(a+b+c)2n+1 . ([Oss] G75.2-5).

oraz (p1 , p2 , p3 ) 6= (0, 0, 0), to




dla wszystkich n N.

1
a2n+1

n

a1
b1

n
n
p1 an
1 +p2 a2 +p3 a3
n +p bn
p1 bn
+p
b
2
3
1
2
3

([OM] Kanada 1969).

oooooooooooooooooooooooooooooooooooooooooooooooooooooooooooooooooooooo

1.2

Cakowito pewnych liczb wymiernych

oooooooooooooooooooooooooooooooooooooooooooooooooooooooooooooooooooooo
1.2.1. Niech a, b Q. Jeli a + b Z i ab Z, to a, b Z.
1.2.2. Niech a, b, c Q. Jeli a + b + c Z, ab + bc + ca Z i abc Z, to a, b, c Z.
1.2.3. Wyznaczy wszystkie trjki (x, y, z) liczb wymiernych dodatnich, dla ktrych wszystkie
liczby x + y + z, x1 + y1 + z1 , xyz s naturalne. ([OM] Polska 1993/1994).
R. Jeli (x, y, z) jest tak trjk, to x, y, z s liczbami naturalnymi. Wszystkie trjki (x, y, z)
takie, e x > y > z: (1, 1, 1), (3, 3, 3), (2, 2, 1), (6, 3, 2), (4, 4, 2). 
2

1
1
1.2.4. Niech x = ab+1
, y = ba+1
, gdzie a, b N. Jeli x + y jest liczb cakowit, to liczby
x i y te s cakowite. ([OM] St.Petersburg 1993, [Fom] 17/93).
7
n, gdzie n N, jest cakowita.
1.2.5. Kada liczba postaci 15 n5 + 31 n3 + 15

([OM] Australia 1994).

1.2.6. Dla kadej liczby naturalnej n liczba




jest cakowita.

2
1



2
2



2
3

2
n

([OM] Czechy-Sowacja 1998/1999).

1.2.7. Niech n N. Liczby

21n3
4

15n+2
6

nie mog by jednoczenie cakowite.

1.2.8. Kad liczb naturaln mona przedstawi w postaci


turalnymi. ([OM] Moskwa 1996/1997, [OM] Modawia 2001).

ab+1
a+b ,

D. Niech (a, b) = (2n 1, 2n + 1) lub (n + 1, n2 + n 1). Wtedy

([M-sj] 463).

gdzie a, b s liczbami na-

ab+1
a+b

= n. 

1.2.9. Kad liczb naturaln wiksz od 1 i nie bdc postaci 2n + 2 mona przedstawi w
postaci ab + a+1
b+1 , gdzie a, b N. ([OM] Moskwa 2000/2001).

Liczby wymierne.

1. Wstpne informacje o liczbach wymiernych

1.2.10. Niech n N. Znale liczb wszystkich par (x, y) liczb naturalnych takich, e n =
xy
22
36
44
63
x+y . Przykady: 1 = 2+2 , 2 = 3+6 = 4+4 = 6+3 . ([Putn] 1960).
R. Problem sprowadza si do opisu liczby rozwiza naturalnych rwnania (x n)(y n) = n2 .
Zachodzi jeden z przypadkw: (x n < 0, y n < 0) lub (x n > 0, y n > 0).
Jeli x n < 0 i y n < 0, to 1 6 x < n i 1 6 y < n, std n < x n < n i n < y n < n,
czyli |x n| < n i |y n| < n. W tym przypadku mamy sprzeczno: n2 = |x n||y n| < n2 .
Niech x n > 0 i y n > 0. Niech (a, b) bdzie dowoln par liczb naturalnych takich, e ab = n2 .
Przyjmijmy: x := a + n, y := b + n. Wtedy (x n)(y n) = ab = n2 . Kada wic taka para (a, b)
wyznacza naturalne rozwizanie rozpatrywanego rwnania. Takich par jest oczywicie tyle ile jest
naturalnych podzielnikw liczby n2 .
Odpowied. Liczba wszystkich takich naturalnych par jest rwna (n2 ), gdzie (n2 ) jest liczb
wszystkich podzielnikw naturalnych liczby n2 . Jeli a jest podzielnikiem naturalnym liczby n2 , to
(x, y) = (n + a, n + n2 /a) jest naturalnym rozwizaniem. Kade rozwizanie jest tej postaci. 

1.2.11. Kad liczb naturaln wiksz od 1 mona przedstawi w postaci


liczbami naturalnymi. ([OM] Moskwa 2000/2001).
1.2.12. Niech a, n N, (a, n) 6= (1, 1). Rwnanie
ralnych rozwiza. ([Crux] 1990 s.172 z.1556).
1.2.13. Jeli a 6= b Z, n N, to liczba

x2 +y 2
axy+1

gdzie a, b s

= n2 ma nieskoczenie wiele natu-

22n1 (a2n +b2n )(a+b)2n


(ab)2

jest cakowita. ([OMm] 1997/1998).

1.2.14. Liczba postaci

2a2 1
b2 +2 ,

gdzie a, b Z, nie jest cakowita.

1.2.15. Liczba postaci

a2 +b2
a2 b2 ,

gdzie a, b N, a 6= b, nie jest cakowita.


n

a2 +b
ab+1 ,

([IMO] Longlist 1992).


([KoM] Gy1959).

y
. Jeli jakie cztery kolejne wyrazy cigu (an ) s
1.2.16. Niech x, y C, x 6= y, an = x xy
liczbami cakowitymi, to wszystkie wyrazy tego cigu s liczbami cakowitymi.
([OM] Rumunia 2002).

1.2.17. Niech a, b bd liczbami naturalnymi i niech




xn = a +

1
2

n

+ b+

1
2

n

W cigu (xn ) jest tylko skoczenie wiele liczb cakowitych.


1.2.18. Jeli liczba

(m+3)n +1
3m

1.2.19. Jeli liczba

m2 +n2 +1
mn

(Newman problem 30).

jest cakowita, to jest nieparzysta.

jest cakowita, to jest rwna 3.

m2 +n2 +6
jest cakowita,
mn
([OM] Estonia 1995/1996, [Crux] 2002 s.74).

1.2.20. Jeli liczba

([IMO] 1967).

([LeH] A5).

to jest szecianem liczby cakowitej.

Liczby wymierne.

1. Wstpne informacje o liczbach wymiernych

1.2.21. Istnieje nieskoczenie wiele par (n, m) liczb naturalnych takich, e 1 < n < m i
2
2 1
liczba m +n
jest cakowita. ([Crux] z.1746).
mn
1.2.22 ([Crux] 2001 z.2534 s.276-279).
2

+a
Oznaczmy za (x, y) = x +y
. Niech A bdzie zbiorem tych wszystkich liczba cakowitych
xy
a, dla ktrych liczba za (x, y) jest cakowita dla nieskoczenie wielu par (x, y) liczb naturalnych. Jeli a A, to przez E(a) oznacza bdziemy zbir wszystkich liczb cakowitych postaci
za (x, y), x, y N.

(1) Niech a Z. Jeli istniej liczby naturalne x, y takie, e liczba za (x, y) jest cakowita,
to takich par (x, y) N2 jest nieskoczenie wiele.
(2) Zbr A jest nieskoczony. Kada liczba postaci d2 , gdzie d N, naley do A. Mamy
bowiem zd2 (d, d) = dla wszystkich N.
(3) Liczba 0 naley do A i E(0) = {2}.
(4) Jeli a = d2 , gdzie d N, to a A i zbir E(a) jest nieskoczony; jest nawet
rwny N. Wynika to z (2).
(5) Jeli a A i a nie jest postaci d2 , gdzie d N, to zbir E(a) jest skoczony.
(6) Niech a N0 . Niech za (x, y) = , gdzie x, y, N. Wtedy 6 a + 2.
1.2.23. Jeli x, y, z s parami rnymi liczbami cakowitymi i n jest liczb naturaln, to liczba
xn
(xy)(xz)

jest cakowita.

yn
(yx)(yz)

zn
(zx)(zy)

([Kurs] 175(1959), [Bryn] 1.1).

n +1
1.2.24. Znale wszystkie pary (m, n) liczb naturalnych, dla ktrych mn1
jest liczb cakowit. Odp. (2, 2), (2, 1), (1, 2), (3, 1), (1, 3), (5, 2), (2, 5), (5, 3), (3, 5). ([IMO] 1994).

a
1.2.25. Niech a, b Z. Jeli 2ab2 b
3 +1 jest liczb cakowit, to (a, b) = (2n, 1) lub (n, 2n) lub
4
(8n n, 2n), gdzie n N. ([IMO] Shortlist 2003).

1.2.26. Znale wszystkie pary (x, y) liczb naturalnych, dla ktrych liczby
turalne. Odp. (3, 2), (2, 3), (1, 1), (2, 1), (1, 2). ([OM] Polska 1994/1995).
2

x+1
y ,

y+1
x

s na-

1.2.27. Niech a = (x+y+z)


, gdzie x, y, z N. Jeli a jest liczb cakowit, to a = 1, 2,
xyz
3, 4, 5, 6, 8 lub 9 (a nie moe by sidemk). ([OM] Mongolia 2000).
1.2.28. Jeli p jest liczb pierwsz i n N, to liczba
1
1+1(p1)

nie jest cakowita.

2
1+2(p1)

([Mon] 96(8)(1989) E3249).

+ +

n
1+n(p1)

Liczby wymierne.

1. Wstpne informacje o liczbach wymiernych

oooooooooooooooooooooooooooooooooooooooooooooooooooooooooooooooooooooo

1.3

Przedstawianie liczb wymiernych w szczeglnej postaci

oooooooooooooooooooooooooooooooooooooooooooooooooooooooooooooooooooooo
1.3.1. Dowolny uamek nieskracalny p/q, gdzie p, q s liczbami naturalnymi, przy czym q
jest nieparzyste, mona przedstawi w postaci 2kn1 , dla pewnych liczb naturalnych n i k.
([Balt] 1995).

1.3.2. Kada liczba wymierna z odcinka (0, 1) o nieparzystym mianowniku jest postaci
n

xyz
x2 +y 2 +z 2

dla pewnych liczb naturalnych x, y, z, gdzie {a} oznacza cz uamkow liczby a.


([KoM] 1997(7) N 146).
1.3.3. Niech a N. Kada liczba wymierna z odcinka (0, 1) jest skoczonym iloczynem liczb
n(n+3)
postaci (n+1)(n+2)
, gdzie n > a. ([Mon] 98(2)(1991) E3347).
1.3.4. Kada liczba wymierna w 6= 1 ma nieskoczenie wiele przedstawie w postaci w =
gdzie x i y s wzgldnie pierwszymi liczbami cakowitymi. ([Wm] 7 37).

x+1
y+1 ,

1.3.5. Niech m N. Kad liczb wymiern w > 1 mona przedstawi w postaci




w = 1+

1
k



1+

1
k+1

1 +

1
k+s

gdzie k jest liczb naturaln > m oraz s jest nieujemn liczb cakowit.
([Mat] 1/58 60, [S64] 199).

1.3.6. Kada liczba wymierna dodatnia w ma jednoznaczne przedstawienie w postaci


w = a1 +

a2
2!

a3
3!

+ +

ak
k! ,

gdzie a1 , a2 , . . . ak s nieujemnymi liczbami cakowitymi takimi, e a2 < 2, a3 < 3, . . . , ak < k


oraz ak 6= 0. ([Mat] 1/80 62).
oooooooooooooooooooooooooooooooooooooooooooooooooooooooooooooooooooooo

1.4

Podzbiory zbioru liczb wymiernych

oooooooooooooooooooooooooooooooooooooooooooooooooooooooooooooooooooooo
1.4.1. Znale wszystkie podzbiory S Q speniajce nastpujce warunki:
(1) jeli a, b S, to a + b S;
(2) jeli a jest niezerow liczb wymiern, to dokadnie jedna z liczb a i a naley do S.
([Bryn] 2.1).

10

Liczby wymierne.

1. Wstpne informacje o liczbach wymiernych

O. S cztery takie podzbiory; zbiory wszystkich liczb wymiernych: dodatnich, nieujemnych,


ujemnych, niedodatnich. 

1.4.2. Niech S bdzie podzbiorem zbioru liczb wymiernych zawierajcym


warunek
1
x
x S = x+1
S i x+1
S.
Wtedy kada liczba wymierna z przedziau (0, 1) naley do S.

1
2

i speniajcym

([OM] W.Brytania 2005).

D. (Indukcja ze wzgldu na mianowniki uamkw). Niech n > 3 i zamy, e kady uamek


p
n,

a
b

taki, e b < n oraz 1 6 a < b, naley do S. Rozwamy uamek


gdzie 1 6 p < n. Niech q = n p.
Oczywicie q < n. Jeli p = q, to n = 2p i wtedy np = 21 S. Jeli p < q, to (na mocy zaoenia)
x
1
x = pq S i std np = x+1
S. Jeli p > q, to x = pq S i std np = x+1
S. 

oooooooooooooooooooooooooooooooooooooooooooooooooooooooooooooooooooooo

1.5

Rne zadania o liczbach wymiernych

oooooooooooooooooooooooooooooooooooooooooooooooooooooooooooooooooooooo
1.5.1. Jeeli liczba naturalna nie jest k-t potg liczby naturalnej, to nie jest te k-t potg
liczby wymiernej. ([S50] s.6).
1.5.2. Niech a, b R, a + b = 1. Jeli liczby a3 i b3 s wymierne, to a i b te s liczbami
wymiernymi. ([OM] Polska 1994/1995).
1.5.3.
Istnieje nieskoczenie wiele par (x, y) liczb wymiernych takich, e x 6= y oraz
p
3
i x + y 2 s liczbami wymiernymi. ([OM] Niemcy 2003/2004).

1.5.4. Niech S = Q r {1, 0, 1}, f : S S, f (x) = x x1 . Wtedy

f n (S) = .

x2 + y 3

([Putn] 2001).

n=1

1.5.5. Jedyn funkcj f : Q Q tak, e f (1) = 2 oraz


f (xy) = f (x)f (y) f (x + y) + 1
jest funkcja f (x) = x + 1.

dla x, y Q,

([Bryn] 6.1).

1.5.6. Funkcja f : Q Q Q spenia nastpujce wasnoci:


(1) f (a, b) = f (b, a),
(2) f (f (a, b), c) = f (a, f (b, c)),
(3) f (0, 0) = 0,
(4) f (a + c, b + c) = f (a, b) + c,
dla wszystkich a, b, c Q. Wykaza, e f (a, b) = max(a, b) dla wszystkich a, b Q lub
f (a, b) = min(a, b) dla wszystkich a, b Q. (H. Derksen 1997).

Liczby wymierne

1. Wstpne informacje o liczbach wymiernych

11

1.5.7. Poda przykad podpiercienia ciaa Q rnego od Z i Q.


1.5.8. Niech S N bdzie zbiorem multyplikatywnym. (tzn. 1 S oraz a, b S ab S).
Niech
n
ao
ZS = x Q; aZ sS x =
.
s
Wykaza, e:
(1) ZS jest podpiercieniem ciaa Q;
(2) Kady podpiercie ciaa Q jest postaci ZS dla pewnego multyplikatywnego podzbioru
S zbioru N.
1.5.9. Znale najmniejszy podpiercie ciaa Q zawierajcy uamki 1/2 i 1/3.
1.5.10. Jeli a, b Z, b 6= 0, to przez Z[a/b] oznaczamy najmniejszy podpiercie ciaa Q
zawierajcy liczb a/b.
(1) Wykaza, e Z[2/7] = Z[3/7].
(2) Jeli a, b Z, b 6= 0 oraz nwd(a, b) = 1, to Z[a/b] = Z[1/b].
F W. N. Wagutien, O uamkach Fareya, [Kw] 8/75 33-39.
N. J. Wilenkin, Z historii uamkw, [Kw] 5/87 34-36.
P. W. niady, Teoria liczb i geometria, (o uamkach Fareya), [Dlt] 4/95 1-3.
J. Wrblewski, Wasnoci przystawania liczb wymiernych i zespolonych, [Mat] 2/80 113-116.

Rozkady jedynki na uamki proste

Uamkiem prostym nazywamy kad dodatni liczb wymiern postaci


liczb naturaln.

1
n,

gdzie n jest

oooooooooooooooooooooooooooooooooooooooooooooooooooooooooooooooooooooo

2.1

Oglne fakty o rozkadach jedynki

oooooooooooooooooooooooooooooooooooooooooooooooooooooooooooooooooooooo
2.1.1. Dla dowolnej liczby naturalnej s rwnanie
1
x1

1
x2

+ +

1
xs

=1

ma co najmniej jedno naturalne rozwizanie i naturalnych rozwiza jest skoczenie wiele.


([S64] 144).
2.1.2. Jeli s > 3 jest liczb naturaln, to rwnanie x11 + x12 + + x1s = 1 posiada co najmniej
jedno naturalne rozwizanie takie, e x1 < x2 < < xs . Jeli liczb takich rozwiza
oznaczymy przez ls , to ls+1 > ls . Przykady: l3 = 1, l4 = 6, l5 = 72, l6 = 2320. ([S64] 145, Maple).
Pierwsza cz powyszej tezy wynika natychmiast z nastpujcego faktu, ktrego atwo udowodni
metod indukcji matematycznej.

2.1.3. Rozwamy cig (an ), liczb naturalnych zdefiniowanych rekurencyjnie nastpujco:


a1 = 2,

an+1 = a1 a2 an + 1, dla n N.

Dla kadej liczby naturalnej n zachodzi rwno


1
1
1
1
+
+ +
+
= 1.
a1 a2
an a1 a2 an
([MM] 34(1)(1960) z.397).

2.1.4. Jeli (an ) jest cigiem takim, e a1 = 2 i an+1 = a1 a2 an + 1, dla n N, to


+ a12 + + a1n < 1. ([Fom] 38/86, wynika z 2.1.3).

1
a1

2.1.5. Dla kadej liczby naturalnej n liczb 1 mona przedstawi jako sum parami rnych
uamkw prostych o mianownikach podzielnych przez n. ([Cmj] 9(1)(1978) s.43).
2.1.6. Rozkady jedynki na sum n parami rnych uamkw prostych dla n > 3:
(1) 1 =

1
21

1
22

+ +

1
2n2

1
32n3

1
62n3 .

(2) Niech r1 = 1, rn+1 = rn (rn + 1). Wtedy 1 =


12

1
r1 +1

+ +

1
rn1 +1

1
rn . ([Dlt] 4/84 16).

Liczby wymierne.

2. Rozkady jedynki na uamki proste

13

2.1.7. Jeli x1 < x2 < < xs s liczbami naturalnymi takimi, e


xs < s2

s1

1
x1

+ x12 + + x1s = 1, to

1
x1

+ x12 + + x1s = 1, to

([Cmj] 28(1)(1997) s.71).

2.1.8. Jeli x1 < x2 < < xs s liczbami naturalnymi takimi, e


xs < 2s! . ([Ko04] 166).

2.1.9. Niech x1 , x2 , . . . , xs , xs+1 bd dodatnimi liczbami rzeczywistymi takimi, e


1
1+x1

Wtedy x1 x2 xs+1 > ss+1 .

1
1+x2

+ +

1
1+xs+1

= 1.

([Crux] 2000 s.167).

oooooooooooooooooooooooooooooooooooooooooooooooooooooooooooooooooooooo

2.2

Jedynka i dwa uamki proste

oooooooooooooooooooooooooooooooooooooooooooooooooooooooooooooooooooooo
1
x

2.2.1. Jedynym rozwizaniem naturalnym rwnania

1
x

2.2.2. Jeli x, y s liczbami naturalnymi takimi, e

1
y

1
y

x > 2 i y > 3. Mamy wic:

1
y

1
2

1
3

5
6.

< 1, to
1
x

D. Zamy, e x 6 y s liczbami naturalnymi takimi, e


1
x

= 1 jest para (x, y) = (2, 2).

1
y

1
x

1
y

6 56 .

< 1. Wtedy jest oczywiste, e

oooooooooooooooooooooooooooooooooooooooooooooooooooooooooooooooooooooo

2.3

Jedynka i trzy uamki proste

oooooooooooooooooooooooooooooooooooooooooooooooooooooooooooooooooooooo
2.3.1. Znale wszystkie trjki (x, y, z) liczb naturalnych takich, e x 6 y 6 z oraz
1
x

Odp. (2, 4, 4), (2, 3, 6), (3, 3, 3).

1
y

1
z

= 1.

([GaT] 1/48, [Kw] 1/88 44).

2.3.2. Nie istniej liczby naturalne a i b takie, e

1
a2

1
ab

2.3.3. Jedynymi cakowitymi rozwizaniami rwnania


(1, 1). ([Br80] 46).

1
x2

2.3.4. Jeli x, y, z s liczbami naturalnymi takimi, e


([Fom] 15/86).

1
x

1
b2

= 1.

1
xy

1
x2

1
z

< 1, to

1
y

([WyKM] 150-52).

= 1 s pary (1, 1) i

1
x

1
y

1
z

41
42 .

14

Liczby wymierne.

2. Rozkady jedynki na uamki proste

oooooooooooooooooooooooooooooooooooooooooooooooooooooooooooooooooooooo

2.4

Jedynka i cztery uamki proste

oooooooooooooooooooooooooooooooooooooooooooooooooooooooooooooooooooooo
2.4.1. Rwnanie x1 + y1 + z1 + 1t = 1 ma dokadnie 14 naturalnych rozwiza takich, e
x 6 y 6 z 6 t. Rozwizaniami tymi s:
(2, 3, 7, 42), (2, 3, 8, 24), (2, 3, 9, 18), (2, 3, 10, 15), (2, 3, 12, 12), (2, 4, 5, 20), (2, 4, 6, 12), (2, 4, 8, 8),
(2, 5, 5, 10), (2, 6, 6, 6), (3, 3, 4, 12), (3, 3, 6, 6), (3, 4, 4, 6), (4, 4, 4, 4).

Wrd nich jest dokadnie 6 takich rozwiza, w ktrych liczby x, y, z, t s parami rne.
([S64] 143, Maple).
2.4.2. Znale wszystkie liczby naturalne x, y, z speniajce rwno
Odp. {x, y, z} = {2, 3, 7}. ([Mat] 2/1985 z.1134).

1
x

1
y

1
z

1
xyz

= 1.

2.4.3. Trjka (1, 2, 4) (wraz z jej permutacjami) jest jedynym rozwizaniem naturalnym rw1
1
1
1
nania xy
+ yz
+ zx
+ xyz
= 1. ([Bedn] 37).
oooooooooooooooooooooooooooooooooooooooooooooooooooooooooooooooooooooo

2.5

Jedynka i pi uamkw prostych

oooooooooooooooooooooooooooooooooooooooooooooooooooooooooooooooooooooo
2.5.1. 1 =

1
3

1
4

1
5

1
6

2.5.2. 1 =

1
2

1
3

1
7

1
78

+
+

1
20 . ([S59] 237).
1
91 ,

1=

1
2

1
5

1
6

1
10

1
30 ,

1=

1
2

1
5

1
6

1
12

1
20 . (Maple).

2.5.3. Rwnanie x11 + x12 + x13 + x14 + x15 = 1 ma dokadnie 147 naturalnych rozwiza takich,
e x1 6 x2 6 x5 . Wrd nich s dokadnie 72 takie rozwizania, w ktrych liczby x1 , . . . , x5
s parami rne. (Maple).
oooooooooooooooooooooooooooooooooooooooooooooooooooooooooooooooooooooo

2.6

Jedynka i sze uamkw prostych

oooooooooooooooooooooooooooooooooooooooooooooooooooooooooooooooooooooo
2.6.1. 1 =

1
3

1
4

1
6

2.6.2. 1 =

1
2

1
3

1
10

1
7

+
1
24

1
12

+
1
72

1
42 . ([S59] 237).

1
90 ,

1=

1
3

1
4

1
6

1
10

1
12

1
15 . (Maple).

2.6.3. Rwnanie x11 + x12 + + x16 = 1 ma dokadnie 3462 naturalnych rozwiza takich, e
x1 6 x2 6 x6 . Wrd nich jest dokadnie 2320 takich rozwiza, w ktrych liczby x1 , . . . , x6
s parami rne. (Maple).

2.6.4. Jeli x1 , . . . , x6 s liczbami naturalnymi takimi, e


jedna z liczb x1 , . . . , x6 jest parzysta. ([G-if] 210).

1
x1

+ +

1
x6

= 1, to co najmniej

Liczby wymierne.

2. Rozkady jedynki na uamki proste

15

oooooooooooooooooooooooooooooooooooooooooooooooooooooooooooooooooooooo

2.7

Jedynka i wicej ni sze uamkw prostych

oooooooooooooooooooooooooooooooooooooooooooooooooooooooooooooooooooooo
2.7.1 (Maple). Jedynka i 7 uamkw prostych:
1=

1
3

1
4

1
8

1
10

1
12

1
15

1
24 ;

1
3

1
4

1
9

1
10

1
12

1
15

1
18 .

1
3

1
4

1
6

1
12

1
20

1
21

1
28

1=

2.7.2 (Maple). Jedynka i 8 uamkw prostych:


1=

1
3

1
4

1
6

1
8

1
24

1
30

1
36

1
45 ;

1=

1
30 .

5
= 1, gdzie x 6 y 6 z, ma dokadnie dwa rozwizania
2.7.3. Rwnanie x1 + y1 + z1 + xyz
naturalne (2, 3, 11) i (2, 5, 5). ([Mat] 4/2001 z.1513).

2.7.4. Niech s N i niech x1 , . . . , xs bd takimi liczbami naturalnymi, e x11 + + x1s = 1.


Jeli liczba s jest parzysta, to co najmniej jedna z liczb x1 , . . . , xs jest rwnie parzysta.
D. Niech s bdzie liczb parzyst i przypumy, e wszystkie liczby x1 , . . . , xs s nieparzyste.
Oznaczmy: m = x1 x2 xs oraz mi = xmi dla i = 1, . . . , s. Wtedy wszytkie liczby m1 , . . . , ms oraz m
s nieparzyste i mamy rwno m = m1 + +ms . Po lewej stronie tej rwnoci jest liczba nieparzysta,
a po prawej stronie mamy liczb parzyst. 
2.7.5. Niech s N i niech x1 , . . . , xs bd takimi liczbami naturalnymi, e x11 + + x1s = 1.
Jeli s 6 8, to co najmniej jedna z liczb x1 , . . . , xs jest parzysta. Dla nieparzystych s wikszych
lub rwnych 9 ju tak nie musi by. ([MM] 24(1)(1950) z.51).
2.7.6. Jedynka i 9 parami rnych uamkw prostych o nieparzystych mianownikach.
(1) 1 =
(2) 1 =
(3) 1 =

1
1
1
1
1
1
1
1
1
3 + 5 + 7 + 9 + 11 + 15 + 35 + 45 + 231 . (P.N.Nagara [MM] 24(1)(1950), [Dlt] 3/83 17).
1
1
1
1
1
1
1
1
1
3 + 5 + 7 + 9 + 11 + 15 + 33 + 45 + 385 . (O.H.Hoke [MM] 24(1)(1950)).
1
1
1
1
1
1
1
1
1
3 + 5 + 7 + 9 + 11 + 15 + 21 + 165 + 693 . (P.N.Nagara [MM] 24(1)(1950)).

2.7.7. Liczba wszystkich cigw (x1 , . . . , x10 ) liczb naturalnych takich, e


jest nieparzysta. ([Putn] 1997, [Zw] 2000).

1
x1

+ + x110 = 1,

2.7.8. Jedynka i 11 parami rnych uamkw prostych o nieparzystych mianownikach.


(1) 1 =
(2) 1 =
2.7.9. 1 =

1
1
1
1
1
1
1
1
1
1
1
3 + 7 + 21 + 27 + 35 + 45 + 63 + 105 + 135 + 189 + 315 . (F.L.Miksa [MM] 24(1)(1950)).
1
1
1
1
1
1
1
1
1
1
1
3 + 5 + 7 + 9 + 15 + 21 + 27 + 35 + 45 + 105 + 945 . ([S59] 237).

1
3

1
5

1
7

1
9

1
15

1
21

1
35

1
45

1
63

1
75

1
105

1
135

1
675 . ([S59] 237).

16

Liczby wymierne

2. Rozkady jedynki na uamki proste

oooooooooooooooooooooooooooooooooooooooooooooooooooooooooooooooooooooo

2.8

Rne fakty o rozkadach jedynki

oooooooooooooooooooooooooooooooooooooooooooooooooooooooooooooooooooooo
2.8.1. Niech A bdzie zbiorem wszystkich liczb naturalnych n, dla ktrych istniej liczby
naturalne a1 , . . . , ak (niekoniecznie rne) takie, e a1 + + ak = n i a11 + + a1k = 1.
Wykaza, e:
(1) kada liczba kwadratowa naley do A;
(2) jeli n, m A, to nm A;
(3) jeli n A, to 2n + 2 A;
(4) jeli n A, to 2n + 9 A;
(5) 1, 4, 9, 10, 11, 16, 17, 18, 20, 22 A;
(6) jeli n > 24, to n A.

([OM] USA 1978, [Kw] 2/80 37, [IMO] Longlist 1992).

2.8.2. Niech A bdzie zbiorem wszystkich liczb naturalnych n, dla ktrych istniej liczby
naturalne a1 , . . . , ak (niekoniecznie rne) takie, e a1 + + ak = n i a11 + + a1k = 1.
Wiedzc, e wszystkie liczby naturalne z przedziau [33, 73] nale do zbioru A wykaza, e
kada liczba naturalna wiksza od 32 naley do zbioru A. ([OM] USA 1978).
D. ([San4j]). Najpierw wykaemy, e jeli n A, to 2n + 8 A oraz 2n + 9 A. Niech n =
a1 + a2 + + ak i a11 + + a1k = 1. Wtedy 2n + 8 = 2(a1 + a2 + + ak ) + 4 + 4 oraz
1
2a1

1
2a2

+ 2a1k +

1
4

Ponadto, 2n + 9 = 2(a1 + a2 + + ak ) + 3 + 6 oraz


Zatem
()

1
4

=
1
2a1

1
2

1
4

1
2a2

1
4

= 1.

+ 2a1k +

1
3

1
6

1
2

1
3

1
6

= 1.

n A = 2n + 8 A i 2n + 9 A.

Niech P (n) oznacza zdanie wszystkie liczby naturalne od n do 2n + 7 nale do zbioru A. Z zaoenia
wiemy, e zdanie P (33) jest prawdziwe. Z () natomiast wynika, e jeli zdanie P (n) jest prawdziwe,
to zdanie P (n + 1) jest rwnie prawdziwe. Teza wynika zatem na mocy indukcji matematycznej. 
F O. T. Iboldin, L. D. Kurlandczyk, Rozbicie jedynki, [Kw] 7/87 48-52.

Rozkady liczb wymiernych na uamki proste

oooooooooooooooooooooooooooooooooooooooooooooooooooooooooooooooooooooo

3.1

Rozkady liczb naturalnych

oooooooooooooooooooooooooooooooooooooooooooooooooooooooooooooooooooooo
3.1.1. 2 =

1
2

1
3

1
4

1
5

1
6

1
7

1
8

1
9

1
10

1
15

1
230

1
57960 . ([Mat] 3/56 5, [S59] 236).

1
1
1
1
1
1
1
1
1
3.1.2. 2 = 12 + 13 + 14 + 15 + 16 + 81 + 10
+ 12
+ 15
+ 20
+ 24
+ 30
+ 40
+ 60
+ 120
. ([Mat] 3/56 5, [S59] 236).

3.1.3. Istniej parami rne liczby naturalne a1 , . . . , a1974 takie, e


([WyKM] 637-74).

1
a1

+ +

1
a1974

= 2.

D. Startujemy od rwnoci 2 = 11 + 12 + 13 + 16 i dla ostatniego uamka kilkakrotnie wykorzystujemy


rwno

1
6k

1
12k

1
20k

1
30k .

3.1.4. Niech x1 , x2 , . . . , xn bd takimi liczbami naturalnymi, e adna z nich nie jest pocztkowym fragmentem adnej innej (na przykad 12 jest pocztkowym fragmentem liczb 12, 125
lub 12405). Zachodzi wtedy nierwno: x11 + x12 + + x1n < 3. ([Balt] 2000).
3.1.5. Kada liczba naturalna jest skoczon sum parami rnych uamkw prostych o nieparzystych mianownikach. ([S57a] 75).
3.1.6. Kada liczba naturalna jest skoczon sum parami rnych uamkw prostych o bezkwadratowych mianownikach. ([Cmj] 9(1)(1978) s.43).
3.1.7. Znale wszystkie trjki x 6 y 6 z liczb naturalnych, dla ktrych
naturaln. Odp. (1, 1, 1), (1, 2, 2), (2, 3, 6), (2, 4, 4), (3, 3, 3). ([Jedr] B.2).

1
x

+ y1 + z1 jest liczb

3.1.8. Istnieje taka permutacja (xn ) zbioru wszystkich liczb naturalnych, e dla kadej liczby
naturalnej m istnieje n takie, e m = x11 + x12 + + x1n . ([OM] Norwegia 1995).
oooooooooooooooooooooooooooooooooooooooooooooooooooooooooooooooooooooo

3.2

Rozkady liczb wymiernych

oooooooooooooooooooooooooooooooooooooooooooooooooooooooooooooooooooooo
3.2.1. Kada liczba wymierna z przedziau (0, 1) jest skoczon sum parami rnych uamkw prostych. ([Mat] 6/56 59, [Br80] 27, [B-rs] 259).
3.2.2. Kada liczba wymierna jest skoczon sum parami rnych uamkw prostych.
([S57a] 46, [Mon] NT-227).

17

18

Liczby wymierne.

3. Rozkady liczb wymiernych na uamki proste

3.2.3. Kad liczb wymiern w z przedziau (0, 1) mona przedstawi w postaci


w=

1
q1

1
q2

+ +

1
qr ,

gdzie q1 , . . . , qr s liczbami naturalnymi takimi, e q1 < q2 < < qr oraz qk dzieli qk+1 dla
k = 1, . . . , r 1. ([Kw] 2/71 29 M24).
U. Przedstawienie takie nie musi by jednoznaczne. Mamy np.

3.2.4. Niech k N i niech w Q. Rwnanie w =


wiele rozwiza naturalnych. ([Isaa] s.48).

1
x1

3
8

1
4

1
8

2
2k+1

1
2k+1

1
2k+3

1
(k+2)(2k+1)

1
(k+2)(2k+1)(2k+3) . ([S59] 238).

3.2.6.

2
2k+1

1
2k+1

1
2k+3

1
(k+1)(2k+3)

1
(k+1)(2k+1)(2k+3) . ([S59] 238).

3.2.7. Niech n > 2 bdzie liczb naturaln. Rozwamy uamki postaci


wzgldnie pierwszymi liczbami naturalnymi takimi, e

Suma wszystkich takich uamkw jest rwna 12 .

1
24 .

+ x12 + + x1k posiada tylko skoczenie

3.2.5.

a < b 6 n,

1
3

1
ab ,

gdzie a i b s

a + b > n.
([Ibe] 1996, [Crux] 2002 s.215).

3.2.8. Dla kadej liczby naturalnej n > 6 istniej liczby naturalne x1 , . . . , xn takie, e 1997
1998 =
1
1
1
x1 + x2 + + xn . Ponadto, w kadym takim rozkadzie istniej dwie liczby xi , xj , ktre nie
1997
1
1
1
s wzgldnie pierwsze. Dla n = 6 mamy: 1998
= 12 + 13 + 19 + 37
+ 37
+ 999
, ([OM] Szwecja 1998).
oooooooooooooooooooooooooooooooooooooooooooooooooooooooooooooooooooooo

3.3

Sumy dwch uamkw prostych

oooooooooooooooooooooooooooooooooooooooooooooooooooooooooooooooooooooo
3.3.1 (A. Schinzel). Niech m, n N. Liczba wymierna m
n jest sum dwch uamkw prostych
wtedy i tylko wtedy, gdy istniej liczby naturalne a, b takie, e a dzieli n, b dzieli n i a + b
dzieli m. ([S57a] 6).
3.3.2. Niech m i n bd wzgldnie pierwszymi liczbami naturalnymi. Liczba wymierna m
n
jest sum dwch uamkw prostych wtedy i tylko wtedy, gdy istniej wzgldnie pierwsze liczby
naturalne a, b takie, e a dzieli n, b dzieli n i a + b dzieli m. ([S57a] 7).
3.3.3. Niech m i n bd wzgldnie pierwszymi liczbami naturalnymi. Liczba wymierna m
n jest
sum dwch uamkw prostych o wzgldnie pierwszych mianownikach wtedy i tylko wtedy, gdy
m2 4n jest liczb kwadratow. ([S57a] 14).

Liczby wymierne.

3. Rozkady liczb wymiernych na uamki proste

19

3.3.4. Kada liczba wymierna z przedziau ( 56 , 1) nie jest sum dwch uamkw prostych.
(Wynika to z 2.2.2).
3.3.5. adna liczba wymierna postaci

3
7n

nie jest sum dwch uamkw prostych. ([Mat] 2/56 8).

3.3.6. Pewne pary (a, b), dla ktrych rwnanie

1
1
x+y

a
b

nie posiada naturalnych rozwiza.

(3, 7), (3, 13), (3, 19), (3, 31), (3, 37), (3, 43), (3, 49),
(4, 5), (4, 13), (4, 17), (4, 25), (4, 29), (4, 37), (4, 41),
(5, 7), (5, 11), (5, 13), (5, 17), (5, 22), (5, 23), (5, 31), (5, 33), (5, 37), (5, 41), (5, 43), (5, 47),
(6, 7), (6, 13), (6, 19), (6, 31), (6, 37), (6, 43), (6, 49),
(7, 8), (7, 9), (7, 11), (7, 15), (7, 16), (7, 17), (7, 19), (7, 22), (7, 23), (7, 25), (7, 29), (7, 31), (7, 32),
(7, 37), (7, 43), (7, 44), (7, 46), (7, 47),
(8, 9), (8, 11), (8, 13), (8, 17), (8, 19), (8, 25), (8, 27), (8, 29), (8, 33), (8, 37), (8, 41), (8, 43),
(9, 10), (9, 11), (9, 13), (9, 19), (9, 22), (9, 23), (9, 25), (9, 29), (9, 31), (9, 37), (9, 38), (9, 41),
(9, 43), (9, 46), (9, 47), (9, 49). (Maple).

3.3.7. Pewne pary (a, b), dla ktrych rwnanie x1 + y1 =


naturalne (x, y) takie, e x 6 y.

a
b

posiada dokadnie jedno rozwizanie

(1, 1),
(3, 4), (3, 5), (3, 11), (3, 17), (3, 23), (3, 25), (3, 29), (3, 41), (3, 47),
(4, 7), (4, 9), (4, 11), (4, 19), (4, 23), (4, 31), (4, 43), (4, 47), (4, 49),
(5, 6), (5, 8), (5, 9), (5, 14), (5, 16), (5, 19), (5, 21), (5, 26), (5, 27), (5, 29), (5, 32), (5, 34), (5, 38),
(5, 39), (5, 46), (5, 49),
(6, 11), (6, 17), (6, 23), (6, 25), (6, 29), (6, 41), (6, 47),
(7, 10), (7, 13), (7, 18), (7, 26), (7, 27), (7, 33), (7, 34), (7, 38), (7, 39), (7, 41), (7, 45), (7, 50),
(8, 21), (8, 23), (8, 31), (8, 35), (8, 47), (8, 49),
(9, 16), (9, 17), (9, 20), (9, 26), (9, 28), (9, 32), (9, 34), (9, 35), (9, 44), (9, 50). (Maple).

3.3.8. Pewne pary (a, b), dla ktrych rwnanie


naturalne (x, y) takie, e x 6 y.

1
1
x+y

a
b

posiada dokadnie dwa rozwizania

(1, 2), (1, 3), (1, 5), (1, 7), (1, 11), (1, 13), (1, 17), (1, 19), (1, 23), (1, 29), (1, 31), (1, 37), (1, 41), (1, 43),
(1, 47),
(2, 3), (2, 5), (2, 7), (2, 11), (2, 13), (2, 17), (2, 19), (2, 23), (2, 29), (2, 31), (2, 37), (2, 41), (2, 43), (2, 47),
(3, 8), (3, 10), (3, 16), (3, 22), (3, 34), (3, 46),
(4, 21), (4, 27), (4, 33),
(5, 12), (5, 18), (5, 28),
(7, 12), (7, 20), (7, 24), (7, 30), (7, 36), (7, 40),
(8, 15), (8, 39), (8, 45). (Maple).

3.3.9. Pewne pary (a, b), dla ktrych rwnanie


naturalne (x, y) takie, e x 6 y.
(1, 4), (1, 9), (1, 25), (1, 49),

1
1
x+y

a
b

posiada dokadnie trzy rozwizania

20

Liczby wymierne.

3. Rozkady liczb wymiernych na uamki proste

(2, 9), (2, 25), (2, 49),


(3, 14), (3, 26), (3, 28), (3, 32), (3, 35), (3, 38),
(4, 15), (4, 35), (4, 39), (4, 45),
(5, 24), (5, 36), (5, 42), (5, 44), (5, 48),
(6, 35),
(7, 48). (Maple).

3.3.10. Pewne pary (a, b), dla ktrych rwnanie x1 + y1 = ab posiada dokadnie cztery rozwizania naturalne (x, y) takie, e x 6 y: (1, 8), (1, 27), (2, 27), (3, 20), (3, 44), (3, 50). (Maple).
3.3.11. Pewne pary (a, b), dla ktrych rwnanie
naturalnych (x, y) takich, e x 6 y.

1
x

+ y1 =

a
b

posiada dokadnie pi rozwiza

(1, 6), (1, 10), (1, 14), (1, 15), (1, 16), (1, 21), (1, 22), (1, 26), (1, 33), (1, 34), (1, 35), (1, 38), (1, 39), (1, 46),
(2, 15), (2, 21), (2, 33), (2, 35), (2, 39),
(3, 40). (Maple).

3.3.12. Pewne pary (a, b), dla ktrych rwnanie x1 + y1 = ab posiada dokadnie sze rozwiza
naturalnych (x, y) takich, e x 6 y: (1, 32), (3, 56), (3, 70), (3, 100), (5, 84). (Maple).
3.3.13. Pary (a, b), dla ktrych rwnanie x1 + y1 = ab posiada dokadnie 7 rozwiza naturalnych (x, y) takich, e x 6 y: (1, 64), (3, 80). S to wszystkie takie wzgldnie pierwsze pary
dla a 6 b 6 100. (Maple).
3.3.14. Pary (a, b), dla ktrych rwnanie
nych (x, y) takich, e x 6 y:

1
x

+ y1 =

a
b

posiada dokadnie 8 rozwiza natural-

(1, 12), (1, 18), (1, 20), (1, 28), (1, 44), (1, 45),
(1, 50), (1, 52), (1, 63), (1, 68), (1, 75), (1, 76), (1, 92), (1, 98), (1, 99),
(2, 45), (2, 63), (2, 75), (2, 99).

S to wszystkie takie wzgldnie pierwsze pary dla a 6 b 6 100.

(Maple).

3.3.15. Pary (a, b), dla ktrych rwnanie x1 + y1 = ab posiada dokadnie 11 rozwiza naturalnych (x, y) takich, e x 6 y: (1, 24), (1, 40), (1, 54), (1, 56), (1, 88). S to wszystkie takie
wzgldnie pierwsze pary dla a 6 b 6 100. (Maple).
3.3.16. Pary (a, b), dla ktrych rwnanie x1 + y1 = ab posiada dokadnie 13 rozwiza naturalnych (x, y) takich, e x 6 y: (1, 36), (1, 100). S to wszystkie takie wzgldnie pierwsze pary
dla a 6 b 6 100. (Maple).
3.3.17. Pary (a, b), dla ktrych rwnanie x1 + y1 = ab posiada dokadnie 14 rozwiza naturalnych (x, y) takich, e x 6 y: (1, 30), (1, 42), (1, 48), (1, 66), (1, 70), (1, 78), (1, 80). S to wszystkie
takie wzgldnie pierwsze pary dla a 6 b 6 100. (Maple).

Liczby wymierne.

3. Rozkady liczb wymiernych na uamki proste

21

1
3.3.18. Rwnanie x1 + y1 = 96
posiada dokadnie 17 rozwiza naturalnych (x, y) takich, e
1
1
1
x 6 y. Rwnanie x + y = 72 posiada dokadnie 18 takich rozwiza. (Maple).

3.3.19. Pary (a, b), dla ktrych rwnanie x1 + y1 = ab posiada dokadnie 23 rozwizania naturalne (x, y) takie, e x 6 y: (1, 60), (1, 84), (1, 90). S to wszystkie takie wzgldnie pierwsze
pary dla a 6 b 6 100. Dla takich (a, b) liczba 23 jest maksymalna. (Maple).
3.3.20. Jeli r jest jedn z liczb 9, 10, 12, 15, 16, 19, 20, 21, 22, to nie ma takich wzgldnie
pierwszych liczb naturalnych 1 6 a 6 b 6 100, e rwnanie x1 + y1 = ab posiada dokadnie r
rozwiza naturalnych x 6 y. (Maple).
oooooooooooooooooooooooooooooooooooooooooooooooooooooooooooooooooooooo

3.4

Rwnanie

1
x

1
y

1
z

oooooooooooooooooooooooooooooooooooooooooooooooooooooooooooooooooooooo
3.4.1.

1
n

1
n+1

3.4.2.

1
2n

1
2n +2i

1
,
2n +22ni

dla i = 0, 1, . . . , n.

3.4.3.

1
pn

1
pn +pi

1
,
pn +p2ni

dla i = 0, 1, . . . , n.

1
n(n+1) ,

1
n2

1
n(n+1)

1
n2 (n+1) ,

1
n3

1
n(n2 +1)

1
n3 (n2 +1) .

3.4.4. Rwnania postaci n1 = x1 + y1 i ich wszystkie naturalne rozwizania (x, y), x 6 y, dla
pewnych n. Liczb rozwiza oznaczono przez r.
(1) n = 2; (3, 6), (4, 4); r = 2.
(2) n = 3; (4, 12), (6, 6); r = 2.
(3) n = 4; (5, 20), (6, 12), (8, 8); r = 3.
(4) n = 5; (6, 30), (10, 10); r = 2.
(5) n = 6; (7, 42), (8, 24), (9, 18), (10, 15), (12, 12); r = 5.
(6) n = 7; (8, 56), (14, 14); r = 2.
(7) n = 8; (9, 72), (10, 40), (12, 24), (16, 16); r = 4.
(8) n = 9; (10, 90), (12, 36), (18, 18); r = 3.
(9) n = 10; (11, 110), (12, 60), (14, 35), (15, 30), (20, 20); r = 5.
(10) n = 25; (26, 650), (30, 150), (50, 50); r = 3.
(11) n = 50; (51, 2550), (52, 1300), (54, 675), (55, 550), (60, 300), (70, 175), (75, 150),
(100, 100); r = 8.

(12)

n = 75; (76, 5700), (78, 1950), (80, 1200), (84, 700), (90, 450), (100, 300), (120, 200),

(150, 150); r = 8.

(13) n = 100; (101, 10100), (102, 5100), (104, 2600), (105, 2100), (108, 1350), (110, 1100),
(116, 725), (120, 600), (125, 500), (140, 350), (150, 300), (180, 225), (200, 200); r = 13.
(14)

n = 200; (201, 40200), (202, 20200), (204, 10200), (205, 8200), (208, 5200), (210, 4200),

(216, 2700), (220, 2200), (225, 1800), (232, 1450), (240, 1200), (250, 1000), (264, 825), (280, 700),
(300, 600), (325, 520), (360, 450), (400, 400); r = 18. (Maple).

22

Liczby wymierne.

3. Rozkady liczb wymiernych na uamki proste

3.4.5. Niech n N i niech x, y N. Nastpujce warunki s rwnowane.


(1) Para (x, y) jest rozwizaniem rwnania

1
x

1
y

= n1 .

(2) Para (x, y) jest postaci (n + a, n + b), gdzie a jest podzielnikiem naturalnym liczby n2
2
oraz b = na .
D. Wynika to z tego, e rwno

1
x

1
y

1
n

jest rwnowana rwnoci (x n)(y n) = n2 . 

3.4.6. Niech n N i niech (n2 ) oznacza liczb naturalnych podzielnikw liczby n2 .


(1) Liczba naturalnych rozwiza (x, y) rwnania
(2) Liczba naturalnych rozwiza (x, y) rwnania

1
1
1
2
x + y = n jest rwna (n ).
1
1
1
x + y = n takich, e x 6 y, jest

rwna

(n2 )+1
. (Wynika to z 3.4.5).
2

3.4.7. Dla kadej liczby naturalnej n liczba naturalnych rozwiza (x, y) rwnania
jest nieparzysta. (Wynika to z 3.4.6).

1
x

+ y1 =

1
n

3.4.8. Jeli p jest liczb pierwsz, to rwnanie x1 + y1 = p1 ma dokadnie trzy naturalne


rozwizania, mianowicie: (x, y) = (p + 1, p + p2 ), (2p, 2p), (p + p2 , p + 1). (Wynika to z 3.4.5).
3.4.9. Niech n N. Nastpujce warunki s rwnowane.
(1) Rwnanie x1 + y1 = n1 ma dokadnie trzy naturalne rozwizania (x, y) (przy czym
rozwizania (x, y) i (y, x), gdzie x 6= y, uwaamy za rne).
(2) Liczba n jest pierwsza.

([Dlt] 6/83 8, [GaT] 4/73, wynika to z 3.4.6).

3.4.10. Jeli p > 2 jest liczb naturaln, to nastpujce dwa warunki s rwnowane:
(1) p jest liczb pierwsz;
(2) rwnanie
([OMm] 1997/98).

1
x

1
p

1
y

ma dokadnie jedno rozwizanie w zbiorze liczb naturalnych.

3.4.11. Niech n N. Jeli liczba naturalnych rozwiza (x, y) rwnania x1 + y1 = n1 jest liczb
pierwsz, to n jest potg liczby pierwszej o wykadniku naturalnym. Implikacja odwrotna nie
musi zachodzi. Dla potgi n = 27 liczba rozwiza jest rwna 15.
D. Niech n = p1 1 ps s bdzie rozkadem kanonicznym liczby n. Wiemy z 3.4.6(1), e liczba
naturalnych rozwiza rwnania x1 + y1 = n1 jest rwna (n2 ), czyli jest rwna liczbie r = (21 +
1
1) (2s + 1). Jeli r jest liczb pierwsz, to s = 1 i std n = p
1 . 

3.4.12. Niech n N. Jeli rwnanie x1 + y1 = n1 ma dokadnie 2005 rozwiza naturalnych


(x, y), to n jest liczb kwadratow. ([OM] W.Brytania 2005).

Liczby wymierne.

3. Rozkady liczb wymiernych na uamki proste

23

D. Niech n = p1 1 ps s bdzie rozkadem kanonicznym liczby n. Wiemy z 3.4.6(1), e liczba


naturalnych rozwiza rwnania x1 + y1 = n1 jest rwna (n2 ), czyli jest rwna liczbie r = (21 +
1) (2s + 1). Zamy, e r = 2005 = 5 401 (401 jest liczb pierwsz). Wtedy s = 2, (1 , 2 ) =
2
2
(2, 200) lub (200, 2). Zatem n = p1 p100
lub n = p100
2
1 p2 , czyli n jest liczb kwadratow. 

3.4.13. Wszystkie rozwizania w liczbach naturalnych rwnania x1 + y1 =


wzorw:
x = m(m + n)a, y = n(m + n)a, z = mna,
gdzie m, n, a N, nwd(m, n) = 1.

1
z

otrzymujemy ze

([Mat] 4/49 57, 4/55 71, [S59] 85).

3.4.14. Niech n N. Nie istnieje adne rozwizanie naturalne (x, y) rwnania


w ktrym liczby x i y s wzgldnie pierwsze.

1
x

1
y

1
n,

3.4.15. Jeli liczby naturalne x, y, z, ktrych najwikszy wsplny dzielnik jest rwny 1, speniaj rwnanie x1 + y1 = z1 , to x + y jest liczb kwadratow. ([OM] 37/2).
3.4.16. Jeli parami wzgldnie pierwsze liczby naturalne x, y, z speniaj rwnanie
to liczby x + y, x z, y z s kwadratowe. ([Tri] 146, [UsaT]).

1
1
x+y

= z1 ,

3.4.17. Niech x, y, z bd liczbami naturalnymi speniajcymi rwnanie x1 + y1 = z1 i niech


d = nwd(x, y, z). Wtedy liczby dxyz i (x z)d s kwadratowe. ([OM] W.Brytania 1998).
F J. Sandor, On the equation

1
x

1
y

1
z

in integers, [Sand] 57-58.

oooooooooooooooooooooooooooooooooooooooooooooooooooooooooooooooooooooo

3.5

Rwnanie

1
x

1
y

2
z

oooooooooooooooooooooooooooooooooooooooooooooooooooooooooooooooooooooo
3.5.1. Rwnania postaci n2 = x1 + y1 i ich wszystkie naturalne rozwizania (x, y), x 6 y, dla
pewnych n. Liczb rozwiza oznaczono przez r.
(1) n = 3; (2, 6), (3, 3); r = 2.
(2) n = 5; (3, 15), (5, 5); r = 2.
(3) n = 7; (4, 28), (7, 7); r = 2.
(4) n = 9; (5, 45), (6, 18), (9, 9); r = 3.
(5) n = 11; (6, 66), (11, 11); r = 2.
(6) n = 13; (7, 91), (13, 13); r = 2.
(7) n = 15; (8, 120), (9, 45), (10, 30), (12, 20), (15, 15); r = 5.
(8) n = 17; (9, 153), (17, 17); r = 2.
(9) n = 19; (10, 190), (19, 19); r = 2.
(10) n = 21; (11, 231), (12, 84), (14, 42), (15, 35), (21, 21); r = 5.
(11) n = 23; (12, 276), (23, 23); r = 2.
(12) n = 25; (13, 325), (15, 75), (25, 25); r = 3. (Maple).

24

Liczby wymierne.

3.5.2.

2
4k+1

1
2k+1

3. Rozkady liczb wymiernych na uamki proste

1
(2k+1)(4k+1) . ([Mat] 3/56 7, [S59] 238).

3.5.3. Jeli p > 2 jest liczb pierwsz, to uamek p2 ma dokadnie jedno przedstawienie w
postaci a1 + 1b , gdzie a, b N, a < b. ([Kurs] 103(1931), [Szn] 7.103).
W. 2ab = p(a + b). 

oooooooooooooooooooooooooooooooooooooooooooooooooooooooooooooooooooooo

3.6

1
x

Rwnanie

1
y

1
z

1
t

oooooooooooooooooooooooooooooooooooooooooooooooooooooooooooooooooooooo
3.6.1.

1
2n

1
2n(2n+1)

1
2n+1

1
n(2n+1) . ([Mat] 4/72 251).

3.6.2. Nie ma takich czterech liczb pierwszych a < b < c < d, e a1 + d1 = 1b + 1c . ([Mat] 4/72 251).
oooooooooooooooooooooooooooooooooooooooooooooooooooooooooooooooooooooo

3.7

Sumy trzech uamkw prostych

oooooooooooooooooooooooooooooooooooooooooooooooooooooooooooooooooooooo
1
n+2

1
n(n+1)

1
(n+1)(n+2) . ([S59] 86).

3.7.1.

1
n

3.7.2.

1
2k+1

1
3k+2

1
6k+3

3.7.3.

1
4k1

1
4k+1

1
(2k+1)(4k1)

1
(2k+1)(4k1)(4k+2) . ([S59] 238).

3.7.4.

1
4k+1

1
4k+3

1
(2k+1)(4k+3)

1
(2k+1)(4k+1)(4k+3) . ([S59] 238).

3.7.5.

4
3k1

1
k

1
3k1

3.7.6.

4
(4t+3)k1

1
(t+1)k

3.7.7.

4
4k+3

1
k+2

3.7.8.

4
7k+3

1
2k+1

3.7.9.

4
7k1

1
2k

1
2(7k1)

1
4k1

3.7.10.

5
4k1

1
k

3.7.11.

5
5k1

1
2k

3.7.12.

5
5k+3

1
k+1

3.7.13.

5
10k+7

3.7.14.

6
5k1

1
(3k+2)(6k+3) . ([MM] 49(1)(1976) s.34).

1
k(3k1) . ([Mat] 2/56 10).

1
(t+1)((4t+3)k1)

1
(k+1)(k+2)
1
2(7k+3)

1
2k

1
2(k+1)

1
k

1
5k1

1
(t+1)k((4t+3)k1) . ([Mat] 2/56 10).

1
(k+1)(4k+3) . ([Mat] 2/56 10).

1
2(2k+1)(7k+3) . ([Mat] 2/56 11).

1
2k(7k1) . ([Mat] 2/56 11).

1
k(4k1) . ([Mat] 2/56 12).

1
2k(10k1) . ([Mat] 2/56 12).

1
(k+1)(5k+3)

1
(k+1)(5k+3) . ([Mat] 2/56 11).

1
(k+1)(10k+7)

1
2(k+1)(10k+7) . ([Mat] 2/56 12).

1
k(5k1) . ([Mat] 2/56 13).

Liczby wymierne.
3.7.15.

6
6k1

1
2k

3. Rozkady liczb wymiernych na uamki proste


+

1
2k

25

1
k(6k1) . ([Mat] 2/56 13).

3.7.16. Znale wszystkie liczby naturalne x 6 y 6 z speniajce rwno

1
x

1
y

1
z

= 12 .

O ([S57a] 34, [MG] 84(500)(2000) s.213, Maple). Jest 10 rozwiza: (3, 7, 42), (3, 8, 24), (3, 9, 18),
(3, 10, 15), (3, 12, 12), (4, 5, 20), (4, 6, 12), (4, 8, 8), (5, 10, 10), (6, 6, 6). 
F W. Sierpiski, O rwnaniu

1
x

1
y

1
z

= 12 . Posadzkowanie, [S57a] 31-42.

3.7.17. Znale wszystkie liczby naturalne x 6 y 6 z speniajce rwno

1
x

1
y

1
z

= 45 .

1
x

1
y

1
z

= 67 .

Odp. (2, 5, 10), (2, 4, 20).

3.7.18. Znale wszystkie liczby naturalne x 6 y 6 z speniajce rwno


Odp. (2, 3, 14). ([Mat] 2/56 17).

3.7.19. Pewne rwnania postaci ab = x1 + y1 + z1 i ich wszystkie naturalne rozwizania (x, y, z),
x 6 y 6 z. Liczb rozwiza oznaczono przez r.
(1)

(a, b) = (1, 3);

(4, 13, 156), (4, 14, 84), (4, 15, 60), (4, 16, 48), (4, 18, 36), (4, 20, 30), (4, 21, 28), (4, 24, 24),

(5, 8, 120), (5, 9, 45), (5, 10, 30), (5, 12, 20), (5, 15, 15), (6, 7, 42), (6, 8, 24), (6, 9, 18), (6, 10, 15), (6, 12, 12), (7, 7, 21),
(8, 8, 12), (9, 9, 9);

r = 21.

(2)

(a, b) = (2, 3);

(2, 7, 42), (2, 8, 24), (2, 9, 18), (2, 10, 15), (2, 12, 12), (3, 4, 12), (3, 6, 6), (4, 4, 6);

(3)

(a, b) = (1, 4);

(5, 21, 420), (5, 22, 220), (5, 24, 120), (5, 25, 100), (5, 28, 70), (5, 30, 60),

(5, 36, 45), (5, 40, 40), (6, 13, 156), (6, 14, 84), (6, 15, 60), (6, 16, 48), (6, 18, 36), (6, 20, 30), (6, 21, 28),
(6, 24, 24), (7, 10, 140), (7, 12, 42), (7, 14, 28), (8, 9, 72), (8, 10, 40), (8, 12, 24), (8, 16, 16), (9, 9, 36),
(9, 12, 18), (10, 10, 20), (10, 12, 15), (12, 12, 12);

r = 28.

(4)

(a, b) = (3, 4);

(2, 5, 20), (2, 6, 12), (2, 8, 8), (3, 3, 12), (3, 4, 6), (4, 4, 4);

r = 6.

(5)

(a, b) = (1, 5);

(6, 31, 930), (6, 32, 480), (6, 33, 330), (6, 34, 255), (6, 35, 210), (6, 36, 180),

(6, 39, 130), (6, 40, 120), (6, 42, 105), (6, 45, 90), (6, 48, 80), (6, 50, 75), (6, 55, 66), (6, 60, 60), (7, 18, 630),
(7, 20, 140), (7, 21, 105), (7, 30, 42), (7, 35, 35), (8, 14, 280), (8, 15, 120), (8, 16, 80), (8, 20, 40), (8, 24, 30),
(9, 12, 180), (9, 15, 45), (9, 18, 30), (10, 11, 110), (10, 12, 60), (10, 14, 35), (10, 15, 30), (10, 20, 20),
(11, 11, 55), (12, 12, 30), (12, 15, 20), (15, 15, 15);

(6)

(a, b) = (2, 5);

r = 36.

(3, 16, 240), (3, 18, 90), (3, 20, 60), (3, 24, 40), (3, 30, 30), (4, 7, 140),

(4, 8, 40), (4, 10, 20), (4, 12, 15), (5, 6, 30), (5, 10, 10), (6, 6, 15);

(7)

(a, b) = (3, 5);

r = 12.

(2, 11, 110), (2, 12, 60), (2, 14, 35), (2, 15, 30), (2, 20, 20), (3, 4, 60),

(3, 5, 15), (3, 6, 10), (4, 4, 10), (5, 5, 5);

r = 10.

(8)

(a, b) = (4, 5);

(2, 4, 20), (2, 5, 10);

r = 2.

(9)

(a, b) = (1, 6);

(7, 43, 1806), (7, 44, 924), (7, 45, 630), (7, 46, 483), (7, 48, 336), (7, 49, 294),

(7, 51, 238), (7, 54, 189), (7, 56, 168), (7, 60, 140), (7, 63, 126), (7, 70, 105), (7, 78, 91), (7, 84, 84),
(8, 25, 600), (8, 26, 312), (8, 27, 216), (8, 28, 168), (8, 30, 120), (8, 32, 96), (8, 33, 88), (8, 36, 72),
(8, 40, 60), (8, 42, 56), (8, 48, 48), (9, 19, 342), (9, 20, 180), (9, 21, 126), (9, 22, 99), (9, 24, 72),
(9, 27, 54), (9, 30, 45), (9, 36, 36), (10, 16, 240), (10, 18, 90), (10, 20, 60), (10, 24, 40), (10, 30, 30),
(11, 14, 231), (11, 15, 110), (11, 22, 33), (12, 13, 156), (12, 14, 84), (12, 15, 60), (12, 16, 48), (12, 18, 36),
(12, 20, 30), (12, 21, 28), (12, 24, 24), (13, 13, 78), (14, 14, 42), (14, 15, 35), (14, 21, 21), (15, 15, 30),
(15, 20, 20), (16, 16, 24), (18, 18, 18);

r = 57.

r = 8.

26

Liczby wymierne.

3. Rozkady liczb wymiernych na uamki proste

(10)

(a, b) = (5, 6);

(2, 4, 12), (2, 6, 6), (3, 3, 6), (3, 4, 4);

r = 4.

(11)

(a, b) = (1, 7);

(8, 57, 3192), (8, 58, 1624), (8, 60, 840), (8, 63, 504), (8, 64, 448), (8, 70, 280),

(8, 72, 252), (8, 84, 168), (8, 88, 154), (8, 105, 120), (8, 112, 112), (9, 32, 2016), (9, 33, 693), (9, 35, 315),
(9, 36, 252), (9, 42, 126), (9, 45, 105), (9, 56, 72), (9, 63, 63), (10, 24, 840), (10, 25, 350), (10, 28, 140),
(10, 30, 105), (10, 35, 70), (10, 40, 56), (11, 21, 231), (11, 22, 154), (12, 17, 1428), (12, 18, 252), (12, 20, 105),
(12, 21, 84), (12, 24, 56), (12, 28, 42), (14, 15, 210), (14, 16, 112), (14, 18, 63), (14, 21, 42), (14, 28, 28),
(15, 15, 105), (15, 21, 35), (16, 16, 56), (21, 21, 21);

(12)

(a, b) = (2, 7);

r = 42.

(4, 29, 812), (4, 30, 420), (4, 32, 224), (4, 35, 140), (4, 36, 126), (4, 42, 84),

(4, 44, 77), (4, 56, 56), (5, 12, 420), (5, 14, 70), (5, 20, 28), (6, 9, 126), (6, 12, 28), (6, 14, 21),
(7, 8, 56), (7, 14, 14), (8, 8, 28);

(13)

r = 17.

(a, b) = (3, 7); (3, 11, 231), (3, 12, 84), (3, 14, 42), (3, 15, 35), (3, 21, 21), (4, 6, 84),
r = 9.

(4, 7, 28), (5, 5, 35), (7, 7, 7);

(14) (a, b) = (4, 7);


(4, 4, 14); r = 7.

(2, 15, 210), (2, 16, 112), (2, 18, 63), (2, 21, 42), (2, 28, 28), (3, 6, 14),

(15)

(a, b) = (5, 7);

(2, 5, 70), (2, 6, 21), (2, 7, 14), (3, 3, 21);

(16)

(a, b) = (6, 7);

(2, 3, 42);

3.7.20. Liczba

8
11

r = 1.

r = 4.

(Maple).

nie jest sum trzech uamkw prostych.

([Mat] 2/56 18).

3.7.21. adna liczba wymierna z przedziau ( 41


42 , 1) nie jest sum trzech uamkw prostych.
(Wynika to z 2.3.4).
3.7.22. Wszystkie pary (a, b) wzgldnie pierwszych liczb naturalnych takich, e 1 6 a 6 b 6
30 i rwnanie x1 + y1 + z1 = ab nie posiada naturalnych rozwiza.
(8, 11), (8, 17), (9, 11), (9, 19), (10, 11), (12, 13), (12, 25), (12, 29), (13, 14), (14, 17), (14, 19), (14, 29),
(15, 16), (15, 17), (15, 19), (15, 23), (16, 17), (16, 23), (17, 19), (17, 23), (18, 19), (18, 23), (18, 29), (19, 23),
(19, 26), (19, 29), (20, 21), (20, 23), (20, 27), (20, 29), (21, 22), (21, 23), (21, 26), (21, 29), (22, 23), (22, 25),
(22, 27), (23, 25), (23, 26), (23, 29), (24, 25), (24, 29), (25, 26), (25, 27), (25, 29), (26, 27), (26, 29), (27, 28),
(27, 29), (28, 29), (29, 30). Jest 51 takich par. (Maple).

3.7.23. Wszystkie pary (a, b) wzgldnie pierwszych liczb naturalnych takich, e 1 6 a 6 b 6


30 i rwnanie x1 + y1 + z1 = ab posiada dokadnie jedno naturalne rozwizanie x 6 y 6 z.
(6, 7), (6, 13), (8, 9), (9, 29), (10, 13), (10, 23), (11, 13), (11, 17), (11, 23), (12, 17), (12, 19), (13, 16),
(13, 17), (13, 29), (14, 15), (14, 23), (16, 19), (16, 21), (16, 25), (17, 18), (17, 22), (18, 25), (19, 20), (19, 21),
(19, 22), (19, 25), (21, 25), (22, 29), (23, 24), (23, 27), (23, 28), (25, 28). S 32 takie pary. (Maple).

3.7.24. Wszystkie pary (a, b) wzgldnie pierwszych liczb naturalnych takich, e 1 6 a 6 b 6


30 i rwnanie x1 + y1 + z1 = ab posiada dokadnie 2 naturalne rozwizania x 6 y 6 z.
(4, 5), (6, 19), (7, 8), (7, 11), (7, 29), (8, 19), (9, 10), (9, 13), (9, 23), (11, 14), (13, 19), (13, 23), (13, 27),
(16, 29), (17, 20), (17, 21), (17, 25), (17, 26), (17, 27), (17, 29). Jest 20 takich par. (Maple).

Liczby wymierne.

3. Rozkady liczb wymiernych na uamki proste

27

3.7.25. Wszystkie pary (a, b) wzgldnie pierwszych liczb naturalnych takich, e 1 6 a 6 b 6


30 i rwnanie x1 + y1 + z1 = ab posiada dokadnie 3 naturalne rozwizania x 6 y 6 z.
(1, 1), (5, 11), (7, 9), (7, 23), (8, 13), (8, 25), (10, 17), (11, 12), (11, 16), (11, 19), (11, 25), (11, 26), (13, 15),
(15, 22), (19, 24), (19, 27), (19, 28). Jest 17 takich par. (Maple).

3.7.26. Wszystkie pary (a, b) wzgldnie pierwszych liczb naturalnych takich, e 1 6 a 6 b 6


30 i rwnanie x1 + y1 + z1 = ab posiada dokadnie 4 naturalne rozwizania x 6 y 6 z.
(4, 13), (4, 17), (5, 6), (5, 7), (7, 17), (8, 29), (9, 14), (9, 22), (10, 21), (11, 15), (11, 29), (13, 18), (14, 25),
(15, 26), (16, 27), (17, 28), (23, 30). Jest 17 takich par. (Maple).

3.7.27. Wszystkie pary (a, b) wzgldnie pierwszych liczb naturalnych takich, e 1 6 a 6 b 6


30 i rwnanie x1 + y1 + z1 = ab posiada dokadnie 5 naturalnych rozwiza x 6 y 6 z.
(7, 10), (7, 16), (7, 22), (11, 27), (13, 21), (13, 22), (13, 28), (17, 24). Jest 8 takich par.

(Maple).

3.7.28. Wszystkie pary (a, b) wzgldnie pierwszych liczb naturalnych takich, e 1 6 a 6 b 6


30 i rwnanie x1 + y1 + z1 = ab posiada dokadnie 6 naturalnych rozwiza x 6 y 6 z.
(3, 4), (4, 9), (5, 13), (5, 17), (7, 19), (8, 27), (11, 18), (13, 20). Jest 8 takich par.

(Maple).

3.7.29. Wszystkie pary (a, b) wzgldnie pierwszych liczb naturalnych takich, e 1 6 a 6 b 6


30 i rwnanie x1 + y1 + z1 = ab posiada dokadnie 7 naturalnych rozwiza x 6 y 6 z.
(4, 7), (4, 29), (5, 8), (6, 11), (6, 25), (8, 23), (9, 16), (9, 17), (10, 19), (10, 29), (12, 23), (15, 29), (19, 30).

Jest 13 takich par.

(Maple).

3.7.30. Wszystkie pary (a, b) wzgldnie pierwszych liczb naturalnych takich, e 1 6 a 6 b 6


30 i rwnanie x1 + y1 + z1 = ab posiada dokadnie 8 naturalnych rozwiza x 6 y 6 z.
(2, 3), (6, 17), (7, 13), (10, 27), (11, 24). Jest 5 takich par.

(Maple).

3.7.31. Wszystkie pary (a, b) wzgldnie pierwszych liczb naturalnych takich, e 1 6 a 6 b 6


30 i rwnanie x1 + y1 + z1 = ab posiada dokadnie 9 naturalnych rozwiza x 6 y 6 z.
(3, 7), (4, 11), (5, 23), (6, 29), (7, 15), (9, 20), (9, 25), (17, 30). Jest 8 takich par.

(Maple).

3.7.32. Wszystkie pary (a, b) wzgldnie pierwszych liczb naturalnych takich, e 1 6 a 6 b 6


30 i rwnanie x1 + y1 + z1 = ab posiada dokadnie 10 naturalnych rozwiza x 6 y 6 z.
(1, 2), (3, 5), (4, 25), (8, 21), (9, 28), (13, 25), (13, 30). Jest 7 takich par.

(Maple).

28

Liczby wymierne

3. Rozkady liczb wymiernych na uamki proste

3.7.33. Liczby r naturalnych rozwiza x 6 y 6 z rwnania


wzgldnie pierwszych liczb naturalnych 1 6 a 6 b 6 30.
(1) r = 128, (a, b) = (1, 16).
(2) r = 136, (a, b) = (1, 14).
(3) r = 156, (a, b) = (1, 25).
(4) r = 157, (a, b) = (1, 22).
(5) r = 160, (a, b) = (1, 12).
(6) r = 174, (a, b) = (1, 26).
(7) r = 196, (a, b) = (1, 15).
(8) r = 211, (a, b) = (1, 18).
(9) r = 230, (a, b) = (1, 27).
(10) r = 245, (a, b) = (1, 21).
(11) r = 292, (a, b) = (1, 20).
(12) r = 340, (a, b) = (1, 28).
(13) r = 366, (a, b) = (1, 24).
(14) r = 497, (a, b) = (1, 30). (Maple).
3.7.34. Znale wszystkie naturalne rozwizania rwnania

1
x

1
x

1
y

1
y

1
z

= 1t .

1
z

a
b

dla pewnych

([S59] 87).

3.7.35. Jeli n > 3 jest nieparzyst liczb naturaln niepodzieln przez 3, to istniej rne
liczby nieparzyste a, b, c takie, e n3 = a1 + 1b + 1c . ([Mon] 2000(1) s.62).
F T. R. Hagedorn, A proof of a conjecture on Egyptian fractions, [Mon] 1007(1)(2000) 62-63.

oooooooooooooooooooooooooooooooooooooooooooooooooooooooooooooooooooooo

3.8

Zadania rne

oooooooooooooooooooooooooooooooooooooooooooooooooooooooooooooooooooooo


3.8.1. 1

1
22



1
32





1


1
42

1
n2

n+1
2n . ([GeG] 15).

3.8.2. Jeli 1 + a1 1 + 1b 1 + 1c = 2, gdzie a 6 b 6 c s liczbami naturalnymi, to


(a, b, c) = (3, 4, 5), (3, 3, 8), (2, 6, 7), (2, 5, 9) lub (2, 4, 15). ([OM] W.Brytania 1995).
3.8.3. Istniej cztery liczby cakowite a, b, c, d takie, e |a|, |b|, |c|, |d| > 106 oraz a1 + 1b + 1c + d1 =
1
abcd . ([OM] Rosja 2006).
n

3.8.4. Niech d > 0 i niech Sn (d) = s = x11 + x12 + + x1n ; x1 , . . . , xn N, s < d , dla
n N. Zbir Sn (d) posiada maksymalny element. ([OM] Wgry-Izrael 2003).
F A. Bonning i inni, Writing a rational number in Egyptian forms, [MG] 86(2002), 432-436.
R. Cohen, Egyptian fraction expansions, [MM] 46(2)(1973) 76-80.
R. K. Guy, Egyptian fractions, [Gy04] 252-262.
R. K. Guy, A reciprocal diophantine equations, [Gy04] 309.
W. Narkiewicz, Egyptian fractions, [Nar86] 42-58.
1
, [Sand] 65-67.
J. Sandor, On the diophantine equation x11 + x12 + + x1n = xn+1
W. Sierpiski, O rozkadach liczb wymiernych. . . , [Mat] 2/56 8-19, [Mat] 3/56 1-8.
H. A. Simmons, On a cyclo-symmetric Diophantine equation, [Mon] 36(3)(1929) 148-155.

Odwrotnoci wyrazw pewnych cigw

oooooooooooooooooooooooooooooooooooooooooooooooooooooooooooooooooooooo

4.1

Niecakowito sumy odwrotnoci wyrazw cigu

oooooooooooooooooooooooooooooooooooooooooooooooooooooooooooooooooooooo
4.1.1. Liczba

1
1

1
2

1
3

+ + n1 , dla n > 2, nigdy nie jest cakowita.

1
1
1
n + n+1 + + n+k ,
([S50] 12, [Wino] 24, [Br80] 28, [DoC] 139).

4.1.2. Liczba

4.1.3. Liczba n1
4.1.4. Liczba

1
3

1
5

1
n+1

+ +

4.1.5. Jeli a N, to liczba

gdzie n, k N, nigdy nie jest cakowita.

1
n+k ,

1
2n+1

([Mon] 41(1)(1934) E46).

gdzie n, k N, nigdy nie jest cakowita.

nigdy nie jest cakowita.

1
1
1
1+a + 1+2a + + 1+na

([Maza] s.12).

([Wino] 24,[DoC] 140).

nigdy nie jest cakowita. ([IMO] Longlist 1979).

4.1.6. Jeli rosncy cig a1 , a2 , . . . , an jest arytmetyczny o wyrazach naturalnych i n > 2, to


liczba a11 + a12 + + a1n nie jest cakowita. ([Mon] 96(8)(1989) 741-742).
4.1.7. Niech m > n > 2 bd liczbami naturalnymi. Niech {a1 , . . . , as } bdzie zbiorem wszystkich liczb naturalnych mniejszych lub rwnych m i wzgldnie pierwszych z n. Wtedy liczba
1
1
1
a1 + a2 + + as nie jest cakowita. ([Mon] 79(10)(1972) E2327).
4.1.8. Niech a1 , . . . , an , gdzie n > 2, bd parami rnymi liczbami naturalnymi. Zamy,
e dokadnie jedna z nich jest liczb parzyst, a pozostae s liczbami nieparzystymi. Wtedy
liczba a11 + a12 + + a1s nie jest cakowita. ([MM] 56(5)(1983)).
+ a12 + + a1s = k jest liczb cakowit. Oznaczmy: m = a1 a2 an oraz
mi =
dla i = 1, . . . , n. Wtedy m1 , . . . , mn s liczbami cakowitymi i mamy rwno m1 + m2 +
+ mn = km. Prawa strona tej rwnoci jest parzysta. Natomiast lewa strona jest nieparzysta. 

D. Przypumy, e
m
ai

1
a1

oooooooooooooooooooooooooooooooooooooooooooooooooooooooooooooooooooooo

4.2

Odwrotnoci wyrazw cigu arytmetycznego

oooooooooooooooooooooooooooooooooooooooooooooooooooooooooooooooooooooo
4.2.1. Jeli (an ) jest cigiem arytmetycznym o niezerowych wyrazach, to
1
a1 a2

1
a2 a3

+ +

1
an an+1

29

n
a1 an+1 .

([Kw] 2/71 38).

30

Liczby wymierne.

4. Odwrotnoci wyrazw pewnych cigw

4.2.2. Niech (an ) bdzie cigiem arytmetycznym o wyrazach dodatnich i rnicy r > 0. Wtedy
1
r

ln an+1
a1 <

1
a1

1
1

+ +

4.2.3. Jeli cn =
1 2
2 cn

1
3

1
5

1
a2

+ +

1
2n1

1
an

1
a1

+ 1r ln aan1 .

([Bedn] 186-7).

dla n N, to

+ (cn c1 )2 + (cn c2 )2 + + (cn cn1 )2 = n2 .

([Kw] 7/81 25).

4.2.4. Jeli 2r jest najwiksz potg dwjki dzielc liczb naturaln n, to 22r jest najwiksz
1
potg dwjki dzielc licznik liczby wymiernej 11 + 13 + 15 + + 2n1
. ([Mon] 67(9)(1960) 924-925).
oooooooooooooooooooooooooooooooooooooooooooooooooooooooooooooooooooooo

4.3

Odwrotnoci kolejnych liczb naturalnych

oooooooooooooooooooooooooooooooooooooooooooooooooooooooooooooooooooooo
4.3.1. Rozwinicie dziesitne liczby

4.3.2.

1
kn

4.3.3.

4.3.4.
D.

1
n+1

1
n+1

1
n+2

+ +

1
2n

>

1
n+1

1
n+2

+ +

1
2n

> 35 , dla n > 3.

1
3

1
4

13
24 ,

1
k+2

1
n

14
24

>

1
k+3

+ +

1
4

+ +

1
n+1

+ +

4.3.6. 1 <

1
n+1

4.3.5.

1
kn+1

1
kn+2

1
n

1
n+2

1
n2

1
5

> 1.

+ +

13
24 ,

1
2k+2

1
kn+n1

1
6

>n

dla n > 2.

37
60

1
n+2

q
n

nie ma czystego okresu.

k+1
k

1 , dla n, k N.

([OM] Izrael 1995).

([BaL] 463, [Szn] 1.70, [G-if] 102).

([G-if] 204).

> 35 . Krok indukcyjny:

1
k+1

1
k+2

+ +

1
2k

1
k+1

1
k+2

+ +

1
2k

>

1
k+1

>

3
5.

1
k+2

+ +

1
2k

([Kurs] 125(1938)).

1
3n+1

([Mat] 4/50 64).

< 2.

([BoL] 51 s.55, [Szn] 1.70).

1
2k+1

1
2k+2

1
(2k+1)(2k+2)

1
k+1

Liczby wymierne.

4. Odwrotnoci wyrazw pewnych cigw

4.3.7. Jeli n jest liczb naturaln, to przez h(n) oznaczamy sum 1 +


Przyjmujemy dodatkowo, e h(0) = 0.
(1)
(2)
(3)

n(n + 1) n < n + h(n), dla n > 2.

31
1
2

1
3

+ + n1 .

([Putn] 1975).

1
n1

< n h(n), dla n > 3. ([Putn] 1975).


(n 1)n




n
1
+ 1. ([Kw] 8/77 45).
n
n + 1 1 < h(n) < n 1
nn

(4) n2 < h (2n 1) < n, dla n > 2. ([BoL] 40 s.55).


(5) Jeli h(n) > m, to h(3n) > m + 1. ([OM] Litwa 1993).
(6) [h ([en ])] = n, gdzie [x] oznacza cz cakowit liczby x. ([Bedn] 188).
(7) n + h(1) + h(2) + + h(n 1) = nh(n). ([OM] Kanada, [Kw] 7/89 76).
(8)

n1
P

(h(n) h(k))2 = 2n h(n).

([Kw] 9/90 25).

k=0

(9) 3h(1) + 5h(2) + 7h(3) + + (2n + 1)h(n) = (n + 1)2 h(n) 12 n(n + 1).
(10) h(n)2 > 2
4.3.8.

1
k

>

(m)
m

([TT] 2003).

1
2 h(2)

+ 13 h(3) + + n1 h(n) , dla n > 2.

n
P

gdzie sumowanie po lewej stronie przebiega wszystkie liczby natu-

k=1

1
k,

([OM] Modawia 1998).

ralne k takie, e 1 6 k 6 n i nwd(k, m) = 1. Przez (m) oznaczamy liczb wszystkich liczb naturalnych mniejszych lub rwnych m i wzgldnie pierwszych z m. ([IMO] 1978, [KoM] 2000(5) A240).
4.3.9. Czy z cigu 1, 12 , 31 , 41 , . . . mona wybra cig skadajcy si z
(1) 5,
(2) n,
(3) nieskoczenie wielu wyrazw, w ktrym kady wyraz, poczwszy od trzeciego, jest
rnic dwch wyrazw poprzednich ? ([Kw] 5/95, M1486).
R. (2). Niech n N. Rozpatrzmy n-wyrazowy cig Fibonacciego: 1, 2, 3, 5, 8, . . . , un1 , un . Zapiszmy wszystkie wyrazy w odwrotnej kolejnoci i podzielmy je przez ich najmniejsz wspln wielokrotno. Otrzymany cig spenia warunki zadania.
8
1
5
1
3
1
(1). Dla przykadu dla n = 5 otrzymujemy: nww(1, 2, 3, 5, 8) = 120, 120
= 15
, 120
= 24
, 120
= 40
,
2
1
1
1
=
,
=
.
Mamy
zatem
piciowyrazowy
cig
speniajcy
warunki
zadania:
120
60 120
120
1
1
1
1
1
,
,
,
,
.
15 24 40 60 120
1
1
1
1
1
1
1
1
1
Sprawdzamy: 15
24
= 40
, 24
40
= 60
, 40
60
= 120
.
(3). Wykaemy teraz, e nie istnieje aden nieskoczony cig speniajcy warunki zadania. Przypumy, e taki cig istnieje. Oznaczmy go przez (an = b1n ). Wwczas:

a3 = a1 a2 =

1
1
b 2 b1
m3

=
=
,
b1
b2
b1 b2
b1 b2

gdzie m3 < b1 b2 . Std dalej otrzymujemy:


a4 = a2 a3 =

m3
b 1 m3
m4
1

=
=
,
b2
b1 b2
b1 b2
b1 b2

n
gdzie m4 < b1 b2 . Na mocy indukcji stwierdzamy, e kady wyraz cigu (an ) jest postaci bm
, gdzie mn
1 b2
jest liczb naturaln mniejsz od b1 b2 . Takich wyrazw jest, wbrew przypuszczeniu, tylko skoczenie
wiele. 

32

Liczby wymierne.

4. Odwrotnoci wyrazw pewnych cigw

4.3.10. Czy w cigu 11 , 21 , 13 , . . . istnieje 5 wyrazw tworzcych cig arytmetyczny? Jaki jest
najduszy podcig arytmetyczny w tym cigu? ([TT] 1982).
O. Istnieje taki cig arytmetyczny dowolnej skoczonej dugoci. Niech m = nww(1, 2, 3, . . . , n).
1 2 3
n
, m, m, . . . , m
jest takim cigiem arytmetycznym dugoci n. 
Wtedy m

4.3.11. Niech n N i niech 11 + 12 + 13 + + n1 = abnn , gdzie liczby an i bn s wzgldnie pierwsze.


Istnieje tylko 19 liczb naturalnych n takich, e 5 - bn . S to: 1, 2, 3, 4, 20, 21, 22, 23, 24, 100,
101, 102, 103, 104, 120, 121, 122, 123, 124. ([Putn] 1997).
1
1
4.3.12. Na tablicy wypisano liczby 11 , 21 , 13 , . . . , 1998
, 1999
. Wybierzmy dwie z nich, powiedzmy
a i b, i zamiast nich napiszmy liczb a + b + ab. Powtarzamy to tak dugo, a otrzymamy tylko
jedn liczb na tablicy. Czy jest moliwe by t jedyn liczb bya liczba 2000 ?
([OM] Sowenia 1999).

R. Oznaczmy: a b = a + b + ab. Wtedy a b = b a oraz (a b) c = a (b c). Mona zatem


wybiera dwie liczby w dowolnej kolejnoci i zawsze pozostanie na kocu ta sama liczba. Poniewa
1
1 21 = 2, 2 13 = 3, . . . , 1998 1999
= 1999, wic kocow liczb jest 1999. 

4.3.13. Zbir wszystkich liczb wymiernych postaci


jest gsty w zbiorze [0, ). ([Cmj] 28(5)(1997) s.409).

1
n+1

1
n+2

+ +

1
n+m ,

gdzie n, m N,

F R. P. Boas Jr., J. W. Wrench Jr., Partial sums of the harmonic series, [Mon] 78(8)(1971) 864-870.

oooooooooooooooooooooooooooooooooooooooooooooooooooooooooooooooooooooo

4.4

Naprzemienne sumy uamkw prostych

oooooooooooooooooooooooooooooooooooooooooooooooooooooooooooooooooooooo
4.4.1. 1

1
2

1
3

1
4

1
2n

1
n+1

1
n+2

+ +

1
2n . ([BoL] 10 s.52).

4.4.2. Jeli n jest parzyste, to 1 12 + 13 41 + n1 = 2


4.4.3. Jeli ab = 1 12 + 13
1979. ([Kw] 8/80 34, [Br83] 10).

1
4

1
1318

1
1319 ,

1
n+2

1
n+4

+ +

1
2n

([Br80] 29).

gdzie a, b N, to a jest podzielne przez

4.4.4. Niech p bdzie liczb pierwsz postaci 3k + 2. Niech a, b bd takimi liczbami natural1
1
nymi, e ab = 1 12 + 13 14 + 2k
+ 2k+1
. Wtedy a jest podzielne przez p. ([Br83] 33).
1
4.4.5. Jeli p > 2 jest liczb pierwsz, to 1 21 + 13 p1
=
p
dzielenia liczby (2 2 )/p przez p. ([Wino] 72).

a
(p1)! ,

gdzie a jest reszt z

Liczby wymierne.

4. Odwrotnoci wyrazw pewnych cigw

4.4.6. Kada liczba wymierna


p
q

1
a1

p
q

33

(0, 1] ma jednoznaczne przedstawienie postaci


1
a1 a2

1
a1 a2 a3

+ (1)k1 a1 a21ak ,
3
8

gdzie a1 , . . . , ak N, 1 6 a1 < a2 < < ak1 < ak 1. Na przykad:


1
1
1
11
42 = 3 34 + 347 . ([Mon] 1999(3) 243).

1
2

1
24 ,

oooooooooooooooooooooooooooooooooooooooooooooooooooooooooooooooooooooo

4.5

Odwrotnoci liczb pierwszych

oooooooooooooooooooooooooooooooooooooooooooooooooooooooooooooooooooooo
4.5.1. Liczba 1 nie jest sum odwrotnoci parami rnych liczb pierwszych.
4.5.2. Nie istniej parami rne liczby pierwsze p1 , . . . , pn takie, e
cakowit. ([MM] 56(5)(1983) Q687, [Mat] 1/1996 s.45).

1
p1

([Szu87] 38).

+ +

1
pn

jest liczb

1
p1

+ + p1n = k
jest liczb cakowit. Oznaczmy: m = p1 p2 pn oraz mi =
dla i = 1, . . . , n. Wtedy m1 , . . . , mn
s liczbami cakowitymi i mamy rwno m1 + m2 + + mn = km. Prawa strona tej rwnoci jest
podzielna przez p1 . Natomiast lewa strona nie jest podzielna przez p1 . 

D. Niech p1 < p2 < < pn , gdzie n > 2, bd takimi liczbami pierwszymi, e


m
pi

4.5.3. Znale wszystkie liczby pierwsze p 6 q 6 r speniajce rwno p1 +


pewnej liczby naturalnej n. Odp. Jest tylko jedno rozwizanie: p = q = r = 3.
([OM] Norwegia 1994).

1
q

1
r

1
n

dla

oooooooooooooooooooooooooooooooooooooooooooooooooooooooooooooooooooooo

4.6

Odwrotnoci liczb potgowych

oooooooooooooooooooooooooooooooooooooooooooooooooooooooooooooooooooooo
4.6.1. Dla kadej liczby naturalnej m przy dostatecznie wielkim naturalnym s rwnanie
+

1
xm
s

= 1 ma rozwizanie w liczbach naturalnych.

1
xm
1

([S64] 149).

4.6.2. Niech p, q, r bd liczbami naturalnymi takimi, e nwd(p, r) = nwd(q, r) = 1. Wtedy


rwnanie x1p + y1q = z1r posiada nieskoczenie wiele rozwiza naturalnych.
D. Z zaoe wynika, e liczby pq i r s wzgldnie pierwsze. Istnieje zatem nieskoczenie wiele
par (m, n) liczb naturalnych takich, e npq mr = 1. Dla kadej takiej pary (m, n) otrzymujemy
naturalne rozwizanie (x, y, z) = (2nq , 2np , 2m ). 

34

Liczby wymierne.

4. Odwrotnoci wyrazw pewnych cigw

oooooooooooooooooooooooooooooooooooooooooooooooooooooooooooooooooooooo

4.7

Odwrotnoci liczb kwadratowych

oooooooooooooooooooooooooooooooooooooooooooooooooooooooooooooooooooooo
4.7.1. Jedynym naturalnym rozwizaniem rwnania
(2, 2, 2, 2). ([Mat] 1/58 64, [S64] 147, [B-zm] 19).

1
x2

1
y2

1
z2

1
t2

= 1 jest czwrka

4.7.2. Jeli s > 1, to nie istniej parami rne liczby naturalne x1 , . . . , xn takie, e
1
x2s

= 1.

4.7.4. Liczba
D. 1 <
2

+ +

([S64] 148b, [B-zm] 20).

4.7.3. Jeli s = 1, 4 lub s > 6, to rwnanie


naturalnych. ([S64] 148).

1
n

1
x21

1
12

1
12

1
22

1
n2 ,

+ +

1
x21

+ +

1
x2s

= 1 ma rozwizanie w liczbach

gdzie n > 1, nigdy nie jest cakowita.

1
1
1
+ 212 + + n12 < 1+ 12
+ 23
+ + (n1)n
= 1+ 1

1
2

1
2

([OM] Litwa 1993).


1
3

+ +

1
n1

1
n

< 2. 

4.7.5.

1
22

1
32

+ +

4.7.6.

1
12

1
22

1
32

+ +

1
n2

< 2 n1 .

4.7.7.

1
12

1
22

1
32

+ +

1
n2

>

4.7.8.

m
(n+1)(n+m+1)

4.7.9.

1
32

1
52

<

+ +

1
n2

< 32 .

1
(n+1)2

([Kw] 8/78 47).

1
(2n+1)2

([Crux] 1998 s.170).

3n
2n+1 . ([Crux] z.108).

1
(n+2)2

< 14 .

+ +

1
(n+m)2

<

m
n(n+m) . ([Siw] 75).

([BoL] 41 s.55, [Dlt] 1/77 6).

4.7.10. Jeli (an ) jest cigiem arytmetycznym o wyrazach dodatnich i rnicy r > 0, to
+ a12 + + a12 < a12 + ra11 dla n N. ([Bedn] 103).

1
a21

4.7.11.

1
122

1
152

1
202 . ([S54] 81).

4.7.12.

1
362

1
452

1
602 . ([Wm] 7 223, [S64] s.108).

4.7.13.

1
602

1
652

1
1562 . ([S54] 81).

4.7.14. Rwnanie

1
x2

+ y12 =

1
z2

ma nieskoczenie wiele rozwiza naturalnych.

([Mat] 4/55 73).

Liczby wymierne.

4. Odwrotnoci wyrazw pewnych cigw

35

D. x = a(b4 c4 ), y = 2abc(b2 + c2 ), z = 2abc(b2 c2 ), a, b, c N, b > c. 


4.7.15.

1
62

1
72

1
142

1
1
1
2 = 2 2 + 32
([Wm] 7 223, [S64] 148c).

1
212 . ([Wm] 7 223, [S64] s.108).

1
72

1
92

4.7.17. Niech s N. Rwnanie


ralnych. ([S64] 150).

1
x20

1
x21

+ +

4.7.18. Niech s > 1. Rwnanie


x0 < x1 < < xs . ([S64] 148a).

1
x20

4.7.16.

1
42

1
62

1
x21

1
122

+ +

1
142

1
x2s

1
x2s

1
212

1
362

1
452

1
602 .

ma nieskoczenie wiele rozwiza natu-

ma rozwizanie w liczbach naturalnych

F J. Sandor, On the equation x12 + y12 = z12 in integers, [Sand] 59-61.


W. Sierpiski, Uwagi do pewnego zagadnienia P. Erd
osa, [Wm] 7 221-228.

oooooooooooooooooooooooooooooooooooooooooooooooooooooooooooooooooooooo

4.8

Odwrotnoci liczb trjktnych

oooooooooooooooooooooooooooooooooooooooooooooooooooooooooooooooooooooo
Liczb trjktn nazywamy kad liczb naturaln postaci tn = n(n+1)
, gdzie n N.
2
4.8.1 (L. Moser).

1
tn1

4.8.2. Rozwamy rwnanie

1
tn

1
tx+1

1
tn+1

+ +

1
tx+2

1
tn2 n1

1
2n . ([Mat] 5/49 47).

1
tx+n

1
ty ,

+ +

gdzie n N.

(1) Jeli n nie jest liczb kwadratow (lub n = 1), to rwnanie to ma nieskoczenie wiele
rozwiza naturalnych. ([Mat] 1/74 59).
(2) Rwnanie nie ma naturalnych rozwiza dla n = 4 i n = 9.

([Mat] 1/76 56).

(3) Jeli n jest liczb kwadratow i n > 16, to rwnanie to ma skoczon (rn od zera)
liczb naturalnych rozwiza. ([Mat] 1/76 56).
4.8.3. Dla kadej liczby naturalnej n liczba
trjktnych. ([S64] 146a).

1
n

jest sum odwrotnoci n parami rnych liczb

4.8.4. Dla kadej liczby naturalnej n > 1 istniej liczby naturalne i < j takie, e
1
n

1
i(i+1)

1
(i+1)(i+2)

1
(i+2)(i+3)

+ +

1
j(j+1) .

([OM] Kanada 1973).

4.8.5. Dla kadej liczby naturalnej s 6= 2 rwnanie


liczbach trjktnych. ([S64] 144).

1
x1

4.8.6. Znale wszystkie trjki (m, n, r) N3 takie, e


([Mon] z.4666: 67(10)(1960) 1034-1035, 68(6)(1961) 578).

1
x2

1
tm

++

1
tn

1
xs

1
tr .

= 1 ma rozwizanie w

36

Liczby wymierne.

4. Odwrotnoci wyrazw pewnych cigw

oooooooooooooooooooooooooooooooooooooooooooooooooooooooooooooooooooooo

4.9

Odwrotnoci szecianw

oooooooooooooooooooooooooooooooooooooooooooooooooooooooooooooooooooooo
1
x3

4.9.1. Rwnanie

1
y3

1
z3

nie posiada naturalnych rozwiza.

D. Przypumy, e takie naturalne rozwizanie (x, y, z) istnieje. Wtedy po pomnoeniu stronami


przez (xyz)3 otrzymujemy rwno (yz)3 + (xz)3 = (xy)3 . Dobrze wiadomo jednak, e rwnanie
x3 + y 3 = z 3 nie ma naturalnych rozwiza. 
4.9.2.

1
23

1
23

4.9.3. Rwnanie

1
22 ,

1
x3

1
653

+ y13 =

1
2603

1
z2

1
5202 ,

1
(41361)3

(1)
(2)
(3)

1
123

1
723

1
83 .

1
x3

1
x3

1
y3

1
z3

1
(8271361)2 .

1
t3

1
y3

1
z2 ,

(Wynika z 4.6.2).

to nwd(x, y) > 2.

Zauwamy, e nwd(9, 12, 72) = 3 oraz 3 - 8. Inne

1
1
1
1
953 + 1713 + 5703 = 903 , nwd(95, 171, 570) = 19
1
1
1
1
1403 + 1703 + 3403 = 1193 , nwd(140, 170, 340) =
1
1
1
1
1203 + 2523 + 2663 = 1713 , nwd(120, 252, 266) =

4.9.6. Rwnanie

ma nieskoczenie wiele rozwiza naturalnych.

4.9.4. Jeli x, y, z s liczbami naturalnymi takimi, e


4.9.5 (Maple). 913 +
przykady tego typu:

1
(91361)3

oraz 19 - 90.
10 oraz 10 - 119.
2 oraz 2 - 171.

ma nieskoczenie wiele naturalnych rozwiza.

D. Z poprzednich przykadw wynika, e co najmniej jedno naturalne rozwizanie (x, y, z, t)


istnieje. Kada wic czwrka postaci (ax, ay, az, at), gdzie a N, te jest naturalnym rozwizaniem
rozpatrywanego rwnania. 
4.9.7. 1213 + 1513 + 2013 = 1013 . Rwno t otrzymujemy dzielc obie strony znanej rwnoci
33 + 43 + 53 = 63 przez 603 . Istnieje wic naturalne rozwizanie rwnania x13 + y13 + z13 = t13
takie, e nwd(x, y, z) = 1. Czy istnieje inne tego typu naturalne rozwizanie? Nie znam
odpowiedzi na to pytanie. (15.02.2008).
4.9.8. Dla kadej liczby naturalnej s > 3 rwnanie
rozwiza naturalnych. ([S64] 151).
4.9.9.
4.9.10.

1
13

1
33

1
23

+ +

1
43

1
n3

+ +

< 45 .

1
n3

<

1
x30

1
x31

+ + x13 ma nieskoczenie wiele

([IMO] Longlist 1969, [OM] Grecja 2005).

1
12 . ([OM] Irlandia 1990).

Liczby wymierne.


4.9.11.

1
23

4. Odwrotnoci wyrazw pewnych cigw



1
33



1
43

1
n3

> 12 .

37

([IMO] Longlist 1971).

oooooooooooooooooooooooooooooooooooooooooooooooooooooooooooooooooooooo

4.10

Granice

oooooooooooooooooooooooooooooooooooooooooooooooooooooooooooooooooooooo
4.10.1.

1
1

1
2

1
3

1
4

+ 51 = ln 2.

4.10.2.

1
1

1
3

1
5

1
7

+ = 4 .

4.10.3.

1
12

1
22

1
32

+ =

2
6 . ([Mon] 1/1934 s.29).

4.10.4.

1
12

1
32

1
52

+ =

2
8 . ([Mon] 22(1)(1915) 17-19).

1
13

4.10.5 (Apery 1978). Liczba

4.10.6. Jeli

1
n3

n=1

= s, to

n=1

([Lion]).

(Leibniz, [Kw] 6/97 22).

1
23

1
(2n1)3

1
33

+ . . . jest niewymierna.

= 87 s oraz

n=1

(1)n1
n3

= 34 s.

4.10.7.

1
14

1
24

1
34

+ =

4
90 . ([Dlt] 7/1980, [Mat] 4/88 218, [Ri97] 157).

4.10.8.

1
16

1
26

1
36

+ =

6
945 . ([Dlt] 7/1980).

1
ns

4.10.9.

P
n=1

1+

1
2s

+ +

(funkcja zeta Riemanna).

4.10.10.

4.10.11.

1
12

1
123

1
22

4.10.12. lim

4.10.13.

P
n=1

1
456

1
2n+1

h(n)
n2n

1
789

2
12 ,

1
2

([MM] 32(3)(1959) z.345).

(s)2 + (2s) dla s > 1, gdzie (s) =




1
22

1
32

1
12

1
22

+ =

1
2n+3

1
2n+5

3
12

P
n=1

([Cmj] 28(2)(1997) 149-150).

1
12

1
ns

([Dlt] 7/1980).

1
42

+ =

4
120 . ([Mon] 41(1)(1934) s.29).

ln 3
4 . ([S-kg] 38).

+ +

gdzie h(n) = 1 +

1
32

1
2

1
4n1

1
2

+ + n1 .

ln 2.

([Crux] 1992 s.140 z.1640).

([MM] 60(2)(1987) z.1240).

1
ns

38

Liczby wymierne

4.10.14. Niech m, k N. Liczba


dzieli k.

P
n=1

1
n(nm+k)

4. Odwrotnoci wyrazw pewnych cigw

jest wymierna wtedy i tylko wtedy, gdy m

([Mon] 91(10)(1984) z.6424).

4.10.15. Niech (an ) bdzie cigiem parami rnych liczb naturalnych, ktrych rozwinicia
P
1
dziesitne nie zawieraj cyfry 0. Wtedy
n=1 an < 29. ([Br80] 89, [B-rs] 236).
4.10.16. Niech (an ) bdzie cigiem parami rnych liczb naturalnych, ktrych rozwinicia
P
1
dziesitne nie maj na pocztku cyfry 9. Wtedy szereg
n=1 an jest zbieny.
([MM] 21(2)(1947) s.112).

4.10.17. Niech (an ) bdzie cigiem parami rnych liczb naturalnych, ktrych rozwinicia
P
1
dziesitne nie zawieraj cyfry 9. Wtedy szereg
n=1 an jest zbieny.
([Mon] 1914, [MM] 21(2)(1947) s.112).

4.10.18. Niech (an ) bdzie cigiem parami rnych liczb naturalnych, ktrych rozwinicia
P
1
dziesitne nie zawieraj cyfry 9. Wtedy
n=1 an < 28. ([Br83] s.77).
4.10.19 (Hipoteza Erdosa). Niech A bdzie takim nieskoczonym podzbiorem zbioru liczb
P
naturalnych, e nA n1 = . Wtedy dla kadego k N zbir A zawiera pewien podcig
arytmetyczny dugoci k. ([Dlt] 9/1999).
U. Nic nie wiadomo nawet dla k = 3 (z artykuu Pawa Strzeleckiego). 
F L. Bibiloni, P. Viader, J. Paradis, On series of Goldbach and Euler, [Mon] 113(3)(2006) 206-220.
M. Kline, Euler and infinite series, [MM] 56(5)(1983) 307-314.
P. Strzelecki, Sumowanie odwrotnoci, [Dlt] 9/1999 12-13.
A. D. Wadhwa, Some convergent subseries of the harmonic series, [Mon] Oct./1978 661-663.
P. Zarzycki, O ciekawych wasnociach szeregu harmonicznego, [Dlt] 10/85 1-3.

Rozwinicia dziesitne liczb wymiernych

oooooooooooooooooooooooooooooooooooooooooooooooooooooooooooooooooooooo

5.1

Tablice rozwini dziesitnych pewnych liczb wymiernych

oooooooooooooooooooooooooooooooooooooooooooooooooooooooooooooooooooooo
5.1.1. Tabela przedstawia rozwinicia dziesitne, wraz z dugociami okresw, dla liczb po1
staci m
, gdzie 2 6 m 6 100.
2
3
4
5
6
7
8
9
10
11
12
13
14
15
16
17
18
19
20
21
22
23
24
25
26
27
28
29
30
31
32
33
34
35
36
37
38
39
40
41
42
43
44
45
46
47
48
49
50

0, 5
0, (3)
0, 25
0, 2
0, 1(6)
0, (142857)
0, 125
0, (1)
0, 1
0, (09)
0, 08(3)
0, (076923)
0, 0(714285)
0, 0(6)
0, 0625
0, (0588235294117647)
0, 0(5)
0, (052631578947368421)
0, 05
0, (047619)
0, 0(45)
0, (0434782608695652173913)
0, 041(6)
0, 04
0, 0(384615)
0, (037)
0, 03(571428)
0, (0344827586206896551724137931)
0, 0(3)
0, (032258064516129)
0, 03125
0, (03)
0, 0(2941176470588235)
0, 0(285714)
0, 02(7)
0, (027)
0, 0(263157894736842105)
0, (025641)
0, 025
0, (02439)
0, 0(238095)
0, (023255813953488372093)
0, 02(27)
0, 0(2)
0, 0(2173913043478260869565)
0, (0212765957446808510638297872340425531914893617)
0, 0208(3)
0, (020408163265306122448979591836734693877551)
0, 02

39

0
1
0
0
1
6
0
1
0
2
1
6
6
1
0
16
1
18
0
6
2
22
1
0
6
3
6
28
1
15
0
2
16
6
1
3
18
6
0
5
6
21
2
1
22
46
1
42
0

40

Liczby wymierne.

51
52
53
54
55
56
57
58
59
60
61
62
63
64
65
66
67
68
69
70
71
72
73
74
75
76
77
78
79
80
81
82
83
84
85
86
87
88
89
90
91
92
93
94
95
96
97
98
99
100

5. Rozwinicia dziesitne liczb wymiernych

0, (0196078431372549)
0, 01(923076)
0, (0188679245283)
0, 0(185)
0, 0(18)
0, 017(857142)
0, (017543859649122807)
0, 0(1724137931034482758620689655)
0, (0169491525423728813559322033898305084745762711864406779661)
0, 01(6)
0, (016393442622950819672131147540983606557377049180327868852459)
0, 0(161290322580645)
0, (015873)
0, 015625
0, 0(153846)
0, 0(15)
0, (014925373134328358208955223880597)
0, 01(4705882352941176)
0, (0144927536231884057971)
0, 0(142857)
0, (01408450704225352112676056338028169)
0, 013(8)
0, (01369863)
0, 0(135)
0, 01(3)
0, 01(315789473684210526)
0, (012987)
0, 0(128205)
0, (0126582278481)
0, 0125
0, (012345679)
0, 0(12195)
0, (01204819277108433734939759036144578313253)
0, 01(190476)
0, 0(1176470588235294)
0, 0(116279069767441860465)
0, (0114942528735632183908045977)
0, 011(36)
0, (01123595505617977528089887640449438202247191)
0, 0(1)
0, (010989)
0, 01(0869565217391304347826)
0, (010752688172043)
0, 0(1063829787234042553191489361702127659574468085)
0, 0(105263157894736842)
0, 01041(6)
0, (010309278350515463917525773195876288659793814432989690721649
484536082474226804123711340206185567)
0, 0(102040816326530612244897959183673469387755)
0, (01)
0, 01

5.1.2. Liczby pierwsze p < 1000, dla ktrych uamki


dugoci p 1.
7
113
257
433
593
823

17
131
263
461
619
857

19
149
269
487
647
863

23
167
313
491
659
887

29
179
337
499
701
937

47
181
367
503
709
941

1
p

16
6
13
3
2
6
18
28
58
1
60
15
6
0
6
2
33
16
22
6
35
1
8
3
1
18
6
6
13
0
9
5
41
6
16
21
28
2
44
1
6
22
15
46
18
1
96
42
2
0

maj rozwinicia dziesitne o okresach

59
193
379
509
727
953

61
223
383
541
743
971

97
229
389
571
811
977

109
233
419
577
821
983

Liczby wymierne.

5. Rozwinicia dziesitne liczb wymiernych

41

oooooooooooooooooooooooooooooooooooooooooooooooooooooooooooooooooooooo

5.2

Okresy rozwini dziesitnych liczb wymiernych

oooooooooooooooooooooooooooooooooooooooooooooooooooooooooooooooooooooo
5.2.1. Rozwinicie dziesitne liczby 17 jest rwne 0, (142857). Rozwamy na paszczynie
punkty (1, 4), (4, 2), (2, 8), (8, 5), (5, 7) oraz (7, 1). Wszystkie te punkty le na elipsie o
rwnaniu 19x2 + 36xy + 41y 2 333x 531y + 1638 = 0. ([MM] 60(4)(1987) 245).
5.2.2. Okres rozwinicia liczby 71 przy podstawie 10 ma dugo 6, a przy podstawie 2 ma
dugo 3. Przy podstawie 2 mamy: 17 = 0, 001001001 . . . .
5.2.3. Dla kadej liczby pierwszej p istnieje podstawa q taka, e dugo okresu rozwinicia
przy podstawie q liczby p1 wynosi p 1.
5.2.4. Niech a, b N, a < b, nwd(a, b) = 1. Zamy, e b = 2n 5m c, gdzie 2 - c, 5 - c.
Wwczas liczba cyfr po przecinku, stojcych przed okresem rozwinicia dziesitnego uamka
a
b , jest rwna liczbie max(n, m). ([JaK] 52).
5.2.5. Okres liczby 1/1997 ma co najwyej 200 sidemek.

([OM] S.Petersburg 1997).

5.2.6. Niech L(n) oznacza dugo najmniejszego okresu rozwinicia dziesitnego uamka n1 .
(1) Jeli nwd(n, 10) = 1, to L(n) jest najmniejsz liczb naturaln r tak, e n | 10r 1.
(2) Niech n = 2a 5b k, gdzie nwd(k, 10) = 1. Wtedy L(n) = L(k).
(3) Jeli nwd(a, b) = 1, to L(ab) = nww(L(a), L(b)).
(4) L(n) | (n).


(5) Niech p P i niech k = vp 10L(p) 1 (tzn. liczba 10L(p) 1 dzieli si pk i nie dzieli
si przez pk+1 ). Wtedy L(pk+m ) = pm L(p) dla wszystkich m N.
(6) L(3m+2 ) = 3m , L(7m+1 ) = 6 7m . ([Kw] 2000/2 25-29).
5.2.7. Niech p bdzie liczb pierwsz rn od 2 i 5. Niech 0 < n < p i niech d bdzie
najmniejsz liczb naturaln tak, e p | 10d 1.
(1) Dugo okresu rozwinicia dziesitnego liczby np jest rwna d.
(2) Jeli d jest parzyste, to okres rozwinicia dziesitnego liczby np mona podzieli na
dwie poowy, ktrych suma jest rwna 10d/2 1. Przykad: 17 = 0, (142857) oraz 142 + 857 =
999 = 103 1. (N.Sato).
F C. Hsia, Decimal expansion of fractions, [Crux] 1997 285-290.
H. Rademacher, O. Toeplitz, Okresy rozwini dziesitnych liczb wymiernych, [RaT] 182-200.
K. S. Rao, A note on the recurring period. . . , [Mon] 62(7)(1955) 484-487.
J. K. Schiller, A theorem in the decimal representation of rationals, [Mon] 66(9)(1959) 797-798.
L. Semionowa, Uamki okresowe (po rosyjsku), [Kw] 2000/2 25-29.

42

Liczby wymierne

5. Rozwinicia dziesitne liczb wymiernych

oooooooooooooooooooooooooooooooooooooooooooooooooooooooooooooooooooooo

5.3

Rne zadania o rozwiniciach dziesitnych liczb wymiernych

oooooooooooooooooooooooooooooooooooooooooooooooooooooooooooooooooooooo
5.3.1. Nie ma liczby wymiernej ab takiej, e 0 < a < b < 101, w ktrej rozwiniciu dziesitnym
wystpuj cyfry 1, 6, 7 (w takiej kolejnoci). ([OM] W.Brytania 1978).
5.3.2. Nie ma liczby wymiernej ab takiej, e 0 < a < b < 126, w ktrej rozwiniciu dziesitnym
wystpuje blok 143. ([Dlt] 2/2003 z.1016).

5.3.3. W rozwiniciu dziesitnym liczby

1
3100

wystpuje blok cyfr 123456789.

([Dlt] 2/2003).

5.3.4. Rozwamy tablic nieskoczon T , ktrej kady wiersz przedstawia rozwinicie dziesitne pewnej liczby wymiernej z przedziau (0, 1). Zamy, e zapisalimy w ten sposb
wszystkie liczby wymierne przedziau (0, 1), a jeli ktra ma podwjn reprezentacj, uwzgldnilimy obie. Tak wic na przykad pewien wiersz tablicy bdzie mia posta 1 1 9 9 9 9 . . . ,
a w innym wierszu pojawi si take 1 2 0 0 0 . . . . Bdziemy pisali

T =

a11
a21
a31
...
...
...

a12
a22
a32
...
...
...

a13
a23
a33
...
...
...

...
...
...
...
...
...

gdzie kade aij jest jedn z cyfr 0, 1, 2, 3, 4, 5, 6, 7, 8, 9.


Rozwamy liczb rzeczywist z przedziau (0, 1), ktrej cyfry po przecinku s wzite z przektnej tej tablicy, tzn., x = 0, a11 a22 a33 . . . . Wwczas:
(1) x jest liczb niewymiern,
(2) x zawiera kad cyfr nieskoczenie wiele razy.

([Dlt] 5/2004 12-13).

F A. Balfour, Rapid decimal expansions. . . , [MG] 87(509)(2003) 300-305.


M. Mendes, Przektna Cantora, [Dlt] 5/2004 12-13.

Przystawanie modulo m dla liczb wymiernych

ooooooooooooooooooooooooooooooooooooooooooooooooooooooooooooooooooooo

6.1

Definicje

ooooooooooooooooooooooooooooooooooooooooooooooooooooooooooooooooooooo
Niech m N i x Q. Mwi bdziemy, e x przystaje do 0 modulo m i zapisywa to
bdziemy jako
x 0 (mod m),
jeli x = ab , gdzie a, b Z, nwd(a, b) = 1 oraz m dzieli a. Przykady:
2
3

0 (mod 2),

34
19

0 (mod 17).

Jeli x Z, to przystawanie do 0 w powyszym sensie pokrywa si z przystawaniem do 0 w


sensie klasycznym.
6.1.1. Niech m N, x Q. Nastpujce dwa warunki s rwnowane.
(1) x 0 (mod m).
(2) Dla kadych liczb cakowitych a i b takich, e x =

a
b

zachodzi podzielno: m | a.

Niech m N i niech x, y Q. Mwi bdziemy, e x przystaje do y modulo m i zapisywa


to bdziemy jako
x y (mod m),
jeli x y 0 (mod m). Przykady:
1
3

1 (mod 2),

34
19

6
19

(mod 20).

Jeli x, y Z, to przystawanie takie pokrywa si ze zwykym przystawaniem modulo m.


6.1.2. Niech x = ab , y = dc , gdzie a, b, c, d Z.
(1) Jeli x y (mod m), to ad bc (mod m).
(2) Implikacja w odwrotnym kierunku nie musi by prawdziwa. Mamy na przykad:
5 6 6 1 (mod 6),

5
6

1
6

(mod 6).

(3) Jeli zaoymy dodatkowo, e nwd(b, m) = 1 i nwd(d, m) = 1, to


ad bc (mod m) x y (mod m).
6.1.3. Niech x, y Q, m1 , m2 N, nwd(m1 , m2 ) = 1. Wtedy
x y (mod m1 m2 ) x y (mod m1 ) i x y (mod m2 ).

43

44

Liczby wymierne.

6. Przystawanie modulo m dla liczb wymiernych

ooooooooooooooooooooooooooooooooooooooooooooooooooooooooooooooooooooo

6.2

Przystawanie i mianowniki

ooooooooooooooooooooooooooooooooooooooooooooooooooooooooooooooooooooo
6.2.1. Niech x = ab , y = dc , gdzie a, b, c, d Z, nwd(a, b) = 1, nwd(c, d) = 1. Jeli x y
(mod m), to nwd(b, m) = nwd(d, m).
D. Niech x y =

a
b

c
d

adbc
bd

gdzie i s wzgldnie pierwszymi liczbami cakowitymi.

Wwczas
ad bc = s,

bd = s,

dla pewnej liczby cakowitej s. Ponadto m | .


Niech u = (b, m). Wtedy u | b, u | m i u | . Z rwnoci ad bc = s wynika wic, e u | ad. Ale
(u, a) = 1 (poniewa (a, b) = 1 i u | b), wic u | d. Zatem u | m i u | d, a zatem u | (d, m). Pokazalimy
wic, e (b, m) | (d, m). W ten sam sposb wykazujemy, e (d, m) | (b, m). 

6.2.2. Niech x = ab , y = dc , gdzie a, b, c, d Z, (a, b) = 1, (c, d) = 1. Jeli x y (mod m),


to (b, m) > 1 (d, m) > 1. (Wynika z 6.2.1).
6.2.3. Zamy, e m = pk , k > 1, gdzie p jest liczb pierwsz. Niech x = ab , y = dc ,
a, b, c, d Z, (a, b) = 1, (c, d) = 1. Niech b = pi b1 , d = pj d1 , gdzie i > 0, j > 0, p - b1 , p - d1 .
Jeli x y (mod pk ), to i = j.
D. Jeli i = 0, to (b, m) = 1 i wtedy (na mocy 6.2.2) (d, m) = 1 czyli j = 0, tzn. i = j. Moemy
wic zaoy, e i > 0. Wtedy (na mocy 6.2.2) rwnie j > 0.
Przypumy, e i > j. Mamy wwczas:
xy =

a
b

c
d

a
pi b1

c
pj d1

ad1 pij cb1


.
pi b1 d1

Std otrzymujemy rwnoci: ad1 pi1 cb1 = s, pi b1 d1 = s, gdzie s, , Z, (, ) = 1 oraz


m = pk | . Wtedy p | ad1 . Poniewa p - d1 , wic p | a. Mamy zatem sprzeczno: p | (a, b) = 1.
Podobn sprzeczno uzyskamy zakadajc, e j > i. Zatem i = j. 

Niech m N i niech x Q. Mwi bdziemy, e liczba x jest m-przedstawialna jeli


istniej liczby cakowite a i b takie, e x = ab oraz (b, m) = 1.
Liczba 16 jest 25-przedstawialna, a liczba 15 nie jest 25-przedstawialna.
6.2.4. Niech x = ab , a, b Z, (a, b) = 1. Liczba x jest m-przedstawialna wtedy i tylko wtedy,
gdy (b, m) = 1.
6.2.5. Niech x, y Q i zamy, e x y (mod m). Wwczas liczba x jest m-przedstawialna
wtedy i tylko wtedy, gdy liczba y jest m-przedstawialna. (Wynika z 6.2.1).
6.2.6. Niech m 6 2, a, b Z, (a, m) = 1, (b, m) = 1. Istnieje wtedy liczba cakowita c taka,
e (m, c) = 1 oraz ab c (mod m).
D. Poniewa (b, m) = 1, istniej liczby cakowite u, v takie, e ub + vm = 1. Niech c = ua. Wtedy
(c, m) = 1 i mamy:
abc
a
= avm
b c=
b
b .
Jest jasne, e

avm
b

0 (mod m). 

Liczby wymierne.

6. Przystawanie modulo m dla liczb wymiernych

45

6.2.7. Niech m 6 2, q, a, b Z, (a, m) = 1, (b, m) = 1, (b, q) = 1, (q, m) > 1. Istnieje wtedy


a
liczba cakowita c taka, e (m, c) = 1 oraz qb
qc (mod m).
D. Poniewa (b, qm) = 1, istniej liczby cakowite u, v takie, e ub + vqm = 1. Niech c = ua.
a
qc (mod m). 
Wtedy (c, m) = 1 i jest jasne, e qb
ooooooooooooooooooooooooooooooooooooooooooooooooooooooooooooooooooooo

6.3

Przystawanie i dodawanie

ooooooooooooooooooooooooooooooooooooooooooooooooooooooooooooooooooooo
6.3.1. Niech x, y Q, m N. Jeli x 0 (mod m) i y 0 (mod m), to x + y 0 (mod m).
a
b,
ad+bc
bd .

a, b Z, (a, b) = 1, m | a, y = dc , c, d Z, (c, d) = 1, m | c. Wtedy


x + y = ab + dc =
Niech ad + bc = s, bd = s, s, , Z, (, ) = 1. Naley wykaza, e
m | . Wiemy natomiast, e m | s.
Zauwamy, e (m, s) = 1. Przypumy bowiem, e (m, s) > 1. Istnieje wtedy liczba pierwsza p
taka, e p | m i p | s. Wtedy p | a i p | c. Ponadto, p | bd (gdy bd = s), czyli p | b lub p | d. Jeli p | b,
to mamy sprzeczno: p | (a, b) = 1. Jeli p | d, to mamy sprzeczno: p | (c, d) = 1. Zatem istotnie
(m, s) = 1.
Mamy teraz: m | s i (m, s) = 1, czyli m | . 

D. Niech x =

6.3.2. Niech x, y, x0 , y 0 Q, m N.
Jeli x x0 (mod m) i y y 0 (mod m), to x + y x0 + y 0 (mod m).
D. Niech x x0 (mod m) i y y 0 (mod m). Wtedy x x0 0 (mod m) i y y 0 0 (mod m)
wic, na mocy 6.3.1,
(x + y) (x0 + y 0 ) = (x x0 ) + (y y 0 ) 0 (mod m),
czyli x + y x0 + y 0 (mod m). 

ooooooooooooooooooooooooooooooooooooooooooooooooooooooooooooooooooooo

6.4

Przystawanie jako relacja rwnowanoci

ooooooooooooooooooooooooooooooooooooooooooooooooooooooooooooooooooooo
6.4.1. Niech m N. Wtedy:
(1) x x (mod m), dla x Q;
(2) x y (mod m) = y x (mod m), dla x, y Q;
(3) x y (mod m) i y z (mod m) = x z (mod m), dla x, y, z Q.
D. Wasnoci (1) i (2) s oczywiste. Wykaemy (3). Niech x y (mod m) i y z (mod m).
Wtedy x y 0 (mod m) i y z 0 (mod m). Z 6.3.1 otrzymujemy wtedy, e
x z = (x y) + (y z) 0 (mod m),
tzn. x z (mod m). 

46

Liczby wymierne.

6. Przystawanie modulo m dla liczb wymiernych

ooooooooooooooooooooooooooooooooooooooooooooooooooooooooooooooooooooo

6.5

Przystawanie i mnoenie

ooooooooooooooooooooooooooooooooooooooooooooooooooooooooooooooooooooo
6.5.1. Jeli x x0 (mod m) i y y 0 (mod m), to nie musi by, e xy x0 y 0 (mod m).
Przykad:
2
2
3
1 = 32 23 6 0 (mod 3).
2 0 (mod 3),
3 3 (mod 3),
Przypomnijmy, e liczba wymierna x jest m-przedstawialna, jeli istniej liczby cakowite
a i b takie, e x = ab oraz (b, m) = 1.
6.5.2. Niech x, u Q i niech x 0 (mod m). Jeli u jest m-przedstawialne, to xu 0
(mod m).
D. Niech x = ab , a, b Z, (a, b) = 1, m | a, u = dc , (c, d) = 1, (d, m) = 1. Wtedy ux =
ac = s,

bd = s,

s, , Z,

ac
bd .

Niech

(, ) = 1.

Naley wykaza, e m | . Wiemy natomiast, e m | s. Wystarczy zatem wykaza, e (m, s) = 1. To


jest oczywiste. 

6.5.3. Niech x, y, u Q i niech x y (mod m). Jeli u jest m-przedstawialne, to xu yu


(mod m). (Wynika z 6.5.2).
6.5.4. Niech x, y, u, v Q. Niech x y (mod m) i u v (mod m). Jeli liczby x i u (lub
rwnowanie y i v) s m-przedstawialne, to xu yv (mod m).
D. Z 6.2.5 wiemy, e wszystkie liczby x, y, u, v s m-przedstawialne. Korzystajc z wczeniejszych
faktw mamy kolejno: x y 0 (mod m), (x y)u 0 (mod m), (u v)y 0 (mod m) i po dodaniu:
xu vy = (x y)u + (u v)y 0 (mod m),
czyli xu yv (mod m). 

ooooooooooooooooooooooooooooooooooooooooooooooooooooooooooooooooooooo

6.6

Przystawanie modulo 2

ooooooooooooooooooooooooooooooooooooooooooooooooooooooooooooooooooooo
6.6.1. Jeli a i b 6= 0 s cakowitymi liczbami nieparzystymi, to

a
b

1 (mod 2).

6.6.2. Jeli a jest parzyst liczb cakowit i b 6= 0 jest nieparzyst liczb cakowit, to
(mod 2).

a
b

6.6.3. Niech a, b bd nieparzystymi liczbami cakowitymi i niech n N. Istnieje wtedy nieparzysta liczba cakowita c taka, e 2an b 2cn (mod 2).

Liczby wymierne.

6. Przystawanie modulo m dla liczb wymiernych

47

D. Liczby b i 2n+1 s wzgldnie pierwsze. Istniej wic liczby cakowite u, v takie, e ub+v2n+1 = 1.
Niech c = ua. Wtedy c jest liczb nieparzyst i mamy:
a
2n b

Jest jasne, e

2va
b

c
2n

abc
2n b

auab
2n b

2va
b .

0 (mod 2). 

6.6.4. Niech a bdzie nieparzyst liczb cakowit i niech n N. Istnieje wtedy nieparzysta
liczba cakowita b taka, e 2an 2bn (mod 2) oraz 1 6 b 6 2n+1 1.
D. Niech b bdzie reszt z dzielenia liczby a przez 2n+1 . Wtedy a = 2n+1 + b dla pewnego u Z,
liczba b jest nieparzysta, 1 6 b 6 2n+1 1 oraz
a
2n

czyli

a
2n

b
2n

b
2n

ab
2n

2n+1 u
2n

2u
1

0 (mod 2),

(mod 2). 

6.6.5. Niech a, b bd nieparzystymi liczbami cakowitymi takimi, e 1 6 a, b 6 2n+1 1.


Jeli 2an 2bn (mod 2), to a = b.
ab
2n

0 (mod 2), wic 2n+1 | a b. Przypumy, e a 6= b. Wtedy |a b| > 2n+1 ,


gdy tymczasem |a b| < 2n+1 . 

D. Poniewa

6.6.6. Kada liczba wymierna przystaje modulo 2 do dokadnie jednej z liczb: 0, 1 lub 2k+1
2n ,
gdzie n > 1, k = 0, 1, 2, . . . , 2n 1. (Dla n = 1 mamy liczby 12 i 32 , a dla n = 2 liczby 14 , 34 , 54
i 47 ).
D. Wynika to z poprzednich faktw. 
ooooooooooooooooooooooooooooooooooooooooooooooooooooooooooooooooooooo

6.7

Przystawanie modulo pk

ooooooooooooooooooooooooooooooooooooooooooooooooooooooooooooooooooooo
Zakadamy, e p jest liczb pierwsz oraz, e k jest liczb naturaln.
6.7.1. Niech a, b bd liczbami cakowitymi niepodzielnymi przez p i niech n N. Istnieje
wtedy liczba cakowita c niepodzielna przez p taka, e
a
pn b

c
pn

(mod pk ).

D. Liczby b i pn+k s wzgldnie pierwsze. Istniej wic liczby cakowite u, v takie, e ub +vpn+k =
1. Niech c = ua. Wtedy c jest liczb niepodzieln przez p i mamy:
a
pn b

Jest jasne, e

avpk
b

c
pn

0 (mod pk ). 

abc
pn b

auab
pn b

avpn+k
pn b

avpk
b .

48

Liczby wymierne

6. Przystawanie modulo m dla liczb wymiernych

6.7.2. Niech a bdzie liczb cakowit niepodzieln przez p i niech n N. Istnieje wtedy liczba
cakowita b niepodzielna przez p taka, e
a
pn

b
pn

(mod pk )

oraz 1 6 b 6 pn+k 1.
D. Niech b bdzie reszt z dzielenia liczby a przez pn+k . Wtedy a = pn+k + b dla pewnego u Z
oraz p - b, 1 6 b 6 pn+k 1 oraz
a
pn

czyli

a
pn

b
pn

b
pn

ab
pn

pn+k u
pn

upk
1

0 (mod pk ),

(mod pk ). 

6.7.3. Niech a, b bd liczbami cakowitymi niepodzielnymi przez p takimi, e 1 6 a, b 6


pn+k 1. Jeli pan pbn (mod pk ), to a = b.
ab
pn

0 (mod pk ), wic pn+k | a b. Przypumy, e a 6= b. Wtedy |a b| > pn+k ,


gdy tymczasem |a b| < pn+k . 

D. Poniewa

6.7.4. Niech a i b bd wzgldnie pierwszymi liczbami cakowitymi niepodzielnymi przez p.


Istnieje wtedy liczba cakowita c taka, e
1 6 c < pk ,

p - c,

a
c (mod pk ).
b

D. Liczby b i pk s wzgldnie pierwsze. Istniej wic liczby cakowite u, v takie, e ub + vpk = 1.


Niech c bdzie reszt z dzielenia liczby ua przez pk . Wtedy p - c, 1 6 c < pk oraz:
a
b

Jest jasne, e

avpk
b

c=

abc
b

auab
b

avpk
b .

0 (mod pk ). 

U. Przy pomocy powyszych faktw atwo opisa klasy abstrakcji rozwaanej relacji przystawania
modulo pk . 

Podzielno dla liczb wymiernych

Oznaczenia: Q := Q r {0}, Z := Z r {0}.


oooooooooooooooooooooooooooooooooooooooooooooooooooooooooooooooooooooo

7.1

Rozkad kanoniczny liczb wymiernych

oooooooooooooooooooooooooooooooooooooooooooooooooooooooooooooooooooooo
7.1.1. Kada liczba wymierna q, rna od 1, 0, 1, ma przedstawienie w postaci
q = p1 1 p2 2 ps s ,
gdzie {1, 1}, p1 , . . . , ps s parami rnymi liczbami pierwszymi i 1 , . . . , s s niezerowymi liczbami cakowitymi. Przedstawienie to jest jednoznaczne z dokadnoci do porzdku.
15
= 22 31 51 71 .
Przykad: 28

7.1.2. Kada niezerowa liczba wymierna q ma jednoznaczne przedstawienie w postaci


q=

p p ,

gdzie = 1, iloczyn przebiega zbir wszystkich liczb pierwszych, kade p jest liczb cakowit
przy czym p = 0 dla prawie wszystkich liczb pierwszych p. Przedstawienie takie nazywa
bdziemy kanonicznym przedstawieniem liczby wymiernej q. W szczeglnoci, q Z
p > 0 dla wszystkich liczb pierwszych p.
oooooooooooooooooooooooooooooooooooooooooooooooooooooooooooooooooooooo

7.2

Relacja podzielnoci w Q

oooooooooooooooooooooooooooooooooooooooooooooooooooooooooooooooooooooo
Q
Q
Niech a, b Q i niech a = a pp , b = b pp bd rozkadami kanonicznymi. Mwi
bdziemy, e a dzieli b i pisa a | b, jeli p 6 p . W sytuacji, gdy a nie dzieli b pisa
p

bdziemy a - b.
Przyjmowa bdziemy rwnie, e kade a Q dzieli liczb 0.
7.2.1. Liczba wymierna
1 < 0, 1 < 0 i 0 < 1.
7.2.2.

1
3

1
5

1
5

1
6

dzieli liczb wymiern 5, gdy

- 13 .

7.2.3. Niech a Q. Wtedy 1 | a a Z.


7.2.4. Niech a, b Q, a 6= 0. Wtedy a | b b = xa.
xZ

49

1
6

= 21 31 50 , 5 = 20 30 51 oraz

50

Liczby wymierne.

7. Podzielno dla liczb wymiernych

7.2.5. Niech a, b, c Q przy czym a 6= 0 i b 6= 0. Wtedy:


(1) a | a;
(2) a | b i b | a = a = b;
(3) a | b i b | c = a | c.
7.2.6. Niech a Q , b, c Q oraz x, y Z. Jeli a | b i a | c, to a | xa + yb.
Std w szczeglnoci mamy:
7.2.7. Niech a Q , b, c Q. Jeli a | b i a | c, to a | a b. Jeli a | b i x Z, to a | xb.
oooooooooooooooooooooooooooooooooooooooooooooooooooooooooooooooooooooo

7.3

Nwd i nww dla liczb wymiernych. Definicje i przykady

oooooooooooooooooooooooooooooooooooooooooooooooooooooooooooooooooooooo
Niech A bdzie skoczonym niepustym zbiorem liczb wymiernych. Jeli b Q , to piszemy
b | A w przypadku, gdy b | a dla wszystkich a A. Analogicznie, jeli 0 6 A i b Q, to
piszemy A | b w przypadku, gdy a | b dla wszystkich a A.
Mwimy, e liczba wymierna d jest najwikszym wsplnym dzielnikiem zbioru A, jeli
spenione s nastpujce trzy warunki.
(0) d > 0;
(1) d | A;
(2) e | A = e | d.
eQ

Mwimy, e liczba wymierna w jest najmniejsz wspln wielokrotnoci zbioru A, jeli


spenione s nastpujce trzy warunki.
(0) w > 0;
(1) A | w;
(2) A | u = w | u.
uQ

7.3.1. Niech A bdzie niepustym skoczonym zbiorem liczb wymiernych. Jeli A 6= {0}, to
najwikszy wsplny dzielnik zbioru A istnieje i jest wyznaczony jednoznacznie. Jeli 0 6 A,
to najmniejsza wsplna wielokrotno zbioru A istnieje i jest wyznaczona jednoznacznie.
D. Niech A = {a1 , . . . , an }. Zamy, e kada z liczb wymiernych a1 , . . . , an jest rna od zera.
Q (i)
Niech ai = i pp bdzie rozkadem kanonicznym liczby ai , dla kadego i = 1, . . . , n. Dla kadej
liczby pierwszej p przyjmijmy:




p := min p(1) , p(2) , . . . , p(n) , p := max p(1) , p(2) , . . . , p(n)
Q
Q
i niech d := pp , w := pp . atwo sprawdzi, e wtedy d jest najwikszym wsplnym dzielnikiem
zbioru A oraz, e w jest najmniejsz wspln wielokrotnoci zbioru A. Jednoznaczno jest oczywista.
Jeli do zbioru A naley zero (w przypadku najwikszego wsplnego dzielnika), to zamiast A
rozpatrujemy zbir A r {0}. 

Oznaczenia dla najwikszego wsplnego dzielnika i najmniejszej wsplnej wielokrotnoci


liczb wymiernych stosujemy takie same, jak dla liczb cakowitych. W szczeglnoci, stosowa
bdziemy te oznaczenia (a, b) := nwd(a, b), [a, b] := nww(a, b).

Liczby wymierne.


7.3.2.

1 1
2, 3

7. Podzielno dla liczb wymiernych

= 16 ;

1 1
2, 3

51

= 1.

7.3.3. Niech a1 , . . . , an Q . Jeli nwd(a1 , . . . , an ) jest liczb cakowit, to wszystkie liczby


a1 , . . . , an s cakowite.
D. Niech d = nwd(a1 , . . . , an ) i zamy, e d Z. Wtedy d N. Istnieje wic liczba cakowita x
taka, e a1 = xd. Zatem a1 Z. Analogicznie a2 , . . . , an Z. 

7.3.4. Jeli a1 , . . . , an Q , to nwd(1, a1 , a2 , . . . , an ) =


nym mianownikiem liczb a1 , . . . , an . ([Hass]).

1
m,

gdzie m jest najmniejszym wspl-

oooooooooooooooooooooooooooooooooooooooooooooooooooooooooooooooooooooo

7.4

Nwd i nww dla liczb wymiernych. Wasnoci

oooooooooooooooooooooooooooooooooooooooooooooooooooooooooooooooooooooo
7.4.1. Niech a, b, c Q . Wtedy:
(1)

(a, b) = (b, a), [a, b] = [b, a],

(2)

((a, b), c) = (a, (b, c)) = (a, b, c), [[a, b], c) = [a, [b, c]] = [a, b, c],

(3)

(ac, bc) = (a, b)|c|, [ac, bc] = [a, b]|c|.

Q
pap , b = b pbp i c =
Q
Q
Q
c pcp bd rozkadami kanonicznymi liczb a, b, c. Wtedy ac = pap +cp i bc = pbp +cp s
Q dp
Q wp
Q 0
rozkadami kanonicznymi liczb ac i bc. Mamy zatem: (a, b) = p , [a, b] = p , (ac, bc) = pdp ,
Q 0
[ac, bc] = pwp , gdzie dp = min(ap , bp ), wp = max(ap , bp ), d0p = min(ap + cp , bp + cp ), wp0 = max(ap +
cp , bp +cp ). Poniewa min(ap +cp , bp +cp ) = min(ap , bp )+cp oraz max(ap +cp , bp +cp ) = max(ap , bp )+cp ,
Q 0
Q
Q dp  Q cp
wic d0p = dp + cp i wp0 = wp + cp . Zatem (ac, bc) = pdp = pdp +cp =
p
( p ) = (a, b)|c| i
Q wp Q cp
Q w0
Q wp +cp
p
= ( p ) ( p ) = [a, b]|c|. 
analogicznie [ac, bc] = p = p

D. Wasnoci (1) i (2) s oczywiste. Udowodnimy (3). Niech a = a

7.4.2. ([a, b], c) = [(a, c), (b, c)), [(a, b), c] = ([a, c], [b, c]).
D. Wynika to z rwnoci min(max(x, y), z) = max(min(x, z), min(y, z), max(min(x, y), z) =
min(max(x, z), max(y, z), dla x, y, z R. 
7.4.3 ([Hass] 13). Jeli a1 , . . . , an Q , to
[a1 , . . . , an ] = 

(i)

pap

(a1 , . . . , an ) = h

1
1
,..., a1
a1
n

i.

(i)

(1)

(n)

oznaczmy: wp := max ap , . . . , ap


,

pap bdzie rozkadem kanonicznym liczby ai , dla i = 1, . . . , n. Wtedy a1


=
i
1
jest rozkadem
kanonicznym
liczby ai  (dla i = 1, . . . ,n). Dla kadej liczby pierwszej p



D. Niech ai = i
i

1
1
,..., a1
a1
n

1
1
a1 , . . . , an

1

pdp

1

(1)

(n)

, dp := min ap , . . . , ap

. Wtedy dp = wp . Zatem

Q
1 Q
Q
1
1
= ( pwp ) = ( pwp )
= pwp = [a1 , . . . , an ].

Analogicznie wykazujemy drug rwno. 

52

Liczby wymierne

7. Podzielno dla liczb wymiernych

7.4.4. Niech a1 , . . . , an Q i niech b = |a1 a2 an |. Wtedy:


(a1 , . . . , an )

b
b
a1 , . . . , an

= b,

[a1 , . . . , an ]

b
b
a1 , . . . , an

= b.

D. Wynika to z 7.4.1 i 7.4.3. 


7.4.5. (a, b)[a, b] = |ab|, dla a, b Q .

(Wynika z 7.4.4 dla n = 2).

7.4.6. (a, b, c)[bc, ac, ab] = |abc|, [a, b, c](bc, ca, ab) = |abc|, dla a, b, c Q .
(Wynika z 7.4.4 dla n = 3).
7.4.7. (a, b, c, d)[bcd, acd, abd, abc] = |abcd|, [a, b, c, d](bcd, acd, abd, abc) = |abcd|,
dla a, b, c, d Q . (Wynika z 7.4.4 dla n = 4).
7.4.8. Niech a1 , . . . , an Q i niech d = nwd(a1 , . . . , an ). Wtedy a1 = a01 d, . . . , an = a0n d,
gdzie a01 , . . . , a0n s liczbami cakowitymi i przy tym nwd(a01 , . . . , a0n ) = 1.
D. Z 7.4.1 wynika, e d = (a1 , . . . , an ) = (a01 d, . . . , a0n d) = (a01 , . . . , a0n )d i std (po podzieleniu
przez d) (a01 , . . . , a0n ) = 1. 

7.4.9. Niech a1 , . . . , an Q i niech d = nwd(a1 , . . . , an ). Istniej wtedy liczby cakowite


x1 , . . . , xn takie, e d = x1 a1 + + xn an .
D. Korzystamy z 7.4.8. Niech a01 , . . . , a0n bd takimi liczbami cakowitymi, e ai = a0i d dla
i = 1, . . . , n. Wiemy, e wtedy nwd(a01 , . . . , a0n ) = 1. Na mocy klasycznego twierdzenia o najwikszym
wsplnym dzielniku liczb cakowitych istniej liczby cakowite x1 , . . . , xn takie, e 1 = x1 a01 + +xn a0n .
Mnoc stronami t rwno przez d, otrzymujemy dan rwno. 

7.4.10. Niech a1 , . . . , an Q i niech d = nwd(a1 , . . . , an ). Wwczas d naley do zbioru


{x1 a1 + + xn an ; x1 , . . . , xn Z} .
Liczba d jest najmniejsz dodatni liczb wymiern nalec do tego zbioru.

([Hass] 16).

D. Fakt, e d naley do tego zbioru wynika z 7.4.9. Przypumy, e q jest dodatni liczb wymiern
mniejsz od d i nalec do rozwaanego zbioru. Niech q = y1 a1 + + yn an , y1 , . . . , yn Z. Poniewa
d | a1 , . . . , d | an , wic z rwnoci q = y1 a1 + + yn an wynika, e d | q. Zatem q = md, gdzie m N.
Std d 6 q i mamy sprzeczno. 

oooooooooooooooooooooooooooooooooooooooooooooooooooooooooooooooooooooo

7.5

Wzgldnie pierwsze liczby wymierne

oooooooooooooooooooooooooooooooooooooooooooooooooooooooooooooooooooooo
Mwimy, e dwie niezerowe liczby wymierne a, b s wzgldnie pierwsze jeli nie istnieje
adna liczba pierwsza jednoczenie je dzielca, tzn. jeli p - nwd(a, b) dla kadej liczby pierwszej p ([Hass] 13). Pojcie to pokrywa si z klasyczn wzgldnie pierwszoci w przypadku,
gdy liczby a i b s cakowite. Dla niecakowitych liczb wymiernych to pojcie nie ma specjalnego sensu. Nie zachodz adne uoglnienia faktw znanych dla wzgldnie pierwszych liczb
cakowitych. Dla przykadu, jeli liczby a, b Q s wzgldnie
pierwsze, to nww(a, b) nie musi
h
i
1 1
1 1
rwna si |ab|. Liczby 2 , 3 s wzgldnie pierwsze i 2 , 3 = 1 6= 12 13 .
7.5.1. Niech a, b Q . Jeli co najmniej jedna z tych liczb nie jest liczb cakowit, to liczby
a i b s wzgldnie pierwsze.

Twierdzenie Wolstenholme i jego uoglnienia

oooooooooooooooooooooooooooooooooooooooooooooooooooooooooooooooooooooo

8.1

Wspczynniki Ai

oooooooooooooooooooooooooooooooooooooooooooooooooooooooooooooooooooooo
Jeli n jest liczb naturaln, to przez Fn (x) oznaczamy wielomian stopnia n zmiennej x
zdefiniowany wzorem
Fn (x) = (x 1)(x 2) (x n).
Przyjmujemy ponadto oznaczenia:
Fn (x) = xn A1 xn1 + A2 xn2 A3 xn3 + + (1)n An .

8.1.1. Przykady:
F1

= x1

F2

= x2 3x + 2

F3

= x3 6x2 + 11x 6

F4

= x4 10x3 + 35x2 50x + 24

F5

= x5 15x4 + 85x3 225x2 + 274x 120

F6

= x6 21x5 + 175x4 735x3 + 1624x2 1764x + 720

F7

= x7 28x6 + 322x5 1960x4 + 6769x3 13132x2 + 13068x 5040

F8

= x8 36x7 + 546x6 4536x5 + 22449x4 67284x3 + 118124x2 109584x + 40320

F9

= x9 45x8 + 870x7 9450x6 + 63273x5 269325x4 + 723680x3 1172700x2


+1026576x 362880

F10

= x10 55x9 + 1320x8 18150x7 + 157773x6 902055x5 + 3416930x4 8409500x3


+12753576x2 10628640x + 3628800

F11

= x11 66x10 + 1925x9 32670x8 + 357423x7 2637558x6 + 13339535x5 45995730x4


+105258076x3 150917976x2 + 120543840x 39916800

F12

= x12 78x11 + 2717x10 55770x9 + 749463x8 6926634x7 + 44990231x6 206070150x5


+657206836x4 1414014888x3 + 1931559552x2 1486442880x + 479001600

F13

= x13 91x12 + 3731x11 91091x10 + 1474473x9 16669653x8 + 135036473x7


790943153x6 + 3336118786x5 9957703756x4 + 20313753096x3 26596717056x2
+19802759040x 6227020800

F14

= x14 105x13 + 5005x12 143325x11 + 2749747x10 37312275x9 + 368411615x8


2681453775x7 + 14409322928x6 56663366760x5 + 159721605680x4
310989260400x3 + 392156797824x2 283465647360x + 87178291200

53

54

Liczby wymierne.

8. Twierdzenie Wolstenholme i jego uoglnienia

8.1.2.
A1 =

n
X
i=1

i=

n(n + 1)
,
2

A2 =

ij,

A3 =

16i<j6n

ijk,

...

16i<j<k6n

8.1.3.
An = n!,

An1 = n!

n
X
1
i=1

An2 = n!

1
,
ij
16i<j6n
X

An3 = n!

1
,
ijk
16i<j<k6n
X

...

8.1.4. (x 1)Fn (x 1) = (x n 1)Fn (x).


8.1.5. (x 1)(x 2)Fn (x 2) = (x n 1)(x n 2)Fn (x).
8.1.6 ([HW4] 87).

4A4 =

n+1
2
n+1
3
n+1
4
n+1
5

..
.
=

n+1
n

A1 =
2A2 =
3A3 =

(n 1)An1

n
2
n
A1 3

A1 n4

+ A1
+
+

+ A1

n1
2
n1
A2 3

+ A2
+

n 
n1

+ A2

+ A3

n1
n2

n2
2

+ An2

3
2

nAn = 1 + A1 + A2 + An1 .
Dowd. Wynika to z rwnoci 8.1.4.
(k)

8.1.7. Niech Fn (x) oznacza k-t pochodn wielomianu Fn (x). Zachodzi rwno:
Ank = (1)nk

1 (k)
F (0),
k! n

dla k = 0, 1, . . . n.
oooooooooooooooooooooooooooooooooooooooooooooooooooooooooooooooooooooo

8.2

Wspczynniki Ai dla liczb pierwszych

oooooooooooooooooooooooooooooooooooooooooooooooooooooooooooooooooooooo
W tym podrozdziale zakadamy, e p jest nieparzyst liczb pierwsz. Zajmowa si bdziemy wspczynnikami A1 , A2 , . . . , An dla n = p 1. Przypomnijmy:
()

Fp1 (x) = (x 1)(x 2) (x (p 1))


= xp1 A1 xp2 + A2 xp3 A3 xp4 + Ap2 x + Ap1 .

8.2.1. Liczby A1 , A2 , . . . , Ap2 s podzielne przez p.

Liczby wymierne.

8. Twierdzenie Wolstenholme i jego uoglnienia

55



p
, p2 , . . . , p1
s podzielne
przez p. Teza wynika zatem ze wzorw 8.1.6 (zastosowanych dla n = p 1) i prostej indukcji.

D. Sposb I. Poniewa p jest liczb pierwsz wic wszystkie liczby

p
1

Sposb II. Z maego twierdzenia Fermata i twierdzenia Bezouta wynika, e w piercieniu wielomianw Zp [x] zachodzi rwno
xp1 1 = (x 1)(x 2) (x (p 1)) = Fp1 (x).
Oznacza to, ze liczby A1 , A2 , . . . , Ap2 s rwne zero modulo p, tzn., s podzielne przez p. 

8.2.2 (Twierdzenie Wilsona). Liczba (p 1)! + 1 jest podzielna przez p.


D. Ze wzorw 8.1.6 (zastosowanych dla n = p 1) mamy: (p 1)Ap1 = 1 + A1 + . . . Ap2 . Zatem
(na mocy 8.2.1) (p 1)Ap1 1 (mod p) i std (p 1)! = Ap1 1 (mod p). 
8.2.3. Jeli p > 5, to p2 | Ap2 .
D. ([HW4] 90). W rwnoci () podstawiamy x = p. Zauwamy, e Fp1 (p) = (p 1)! = Ap1 .
Mamy zatem: 0 = pp1 A1 pp2 + A2 pp3 A3 pp4 + Ap2 p, czyli
Ap2 = pp2 A1 pp3 + A2 pp4 A3 pp5 + + Ap3 p.
Poniewa liczby A1 , A2 , . . . , Ap3 s podzielne przez p (na mocy 8.2.1), wic p2 | Ap2 . 
U. Powyszy fakt nie zachodzi dla p = 3. W tym przypadku bowiem Ap2 = 3 nie jest podzielne
przez 9. 

8.2.4. Jeli p > 5, to wszystkie liczby A3 , A5 , . . . , Ap4 , Ap2 s podzielne przez p2 .


D. Wykorzystamy 8.1.7 dla n = p1. Niech m {0, 1, . . . , (p5)/2}. Rwno () rniczkujemy
stronami 2m razy i do wyniku podstawiamy x = p. Wwczas (na mocy 8.1.7) otrzymujemy rwno
postaci
0 = u0 pp12m + u1 A1 pp22m + u2 A2 pp32m + + up22m Ap22m p,
gdzie u0 , u1 , . . . , up2+2m s liczbami cakowitymi niepodzielnymi przez p. Poniewa liczby A1 , A2 , . . . ,
Ap32m s podzielne przez p (na mocy 8.2.1), wic p2 | Ap22m , dla wszystkich m = 0, 1, . . . , (p
5)/2. 

oooooooooooooooooooooooooooooooooooooooooooooooooooooooooooooooooooooo

8.3

Zastosowania dla liczb pierwszych i iloczynw

oooooooooooooooooooooooooooooooooooooooooooooooooooooooooooooooooooooo
8.3.1. Jeli p > 5 jest liczb pierwsz, liczba
bp =

p1
X
i=1

jest podzielna przez p2 .

1 2 bi (p 1)

56

Liczby wymierne.

8. Twierdzenie Wolstenholme i jego uoglnienia

D. Liczba bp pokrywa si z liczb Ap2 . Teza wynika zatem z 8.2.3. 


U. Rozkady na czynniki pierwsze liczb postaci bn dla n niekoniecznie pierwszych.
n
bn
3
3
4
11
5
50
6
274
7
1764
8
13068
9
109584
10 1026576
n
bn
11
10628640
12
120543840
13
1486442880
14
19802759040
15
283465647360
16
4339163001600
17
70734282393600
18
1223405590579200
19 22376988058521600
20 431565146817638400
n
21
22
23
24
25
26
27
28
29
30

bn
8752948036761600000
186244810780170240000
4148476779335454720000
96538966652493066240000
2342787216398718566400000
59190128811701203599360000
1554454559147562279567360000
42373564558110787183902720000
1197348677077520393310044160000
35027999979859805266492784640000

rozkad
(3)
(11)
(2)(5)2
(2)(137)
(2)2 (3)2 (7)2
(2)2 (3)3 (11)2
(2)4 (3)2 (761)
(2)4 (3)2 (7129)
rozkad
(2)5 (3)2 (5)(11)2 (61)
(2)5 (3)2 (5)(97)(863)
(2)7 (3)3 (5)(13)2 (509)
(2)7 (3)3 (5)(29)(43)(919)
(2)8 (3)3 (5)(7)(1049)(1117)
(2)8 (3)4 (5)2 (7)(29)(41233)
(2)11 (3)4 (5)2 (7)(17)2 (8431)
(2)11 (3)4 (5)2 (7)(37)(1138979)
(2)12 (3)7 (5)2 (7)(19)2 (39541)
(2)12 (3)7 (5)2 (7)(37)(7440427)
rozkad
(2)14 (3)7 (5)5 (7)(11167027)
(2)14 (3)9 (5)4 (7)2 (18858053)
(2)15 (3)10 (5)4 (7)2 (11)(23)2 (53)(227)
(2)15 (3)9 (5)4 (7)2 (11)(761)(583859)
(2)18 (3)9 (5)5 (7)2 (11)(577)(467183)
(2)18 (3)9 (5)4 (7)2 (11)(109)(312408463)
(2)19 (3)9 (5)4 (7)2 (11)(13)(34395742267)
(2)19 (3)10 (5)4 (7)2 (11)(13)(521)(215087)(2789)
(2)21 (3)10 (5)4 (7)3 (11)(13)(29)2 (375035183)
(2)21 (3)10 (5)4 (7)3 (11)(13)(43)2 (4990290163)

8.3.2. Jeli p > 5 jest liczb pierwsz, to liczba


cp =

1 2 bi bj (p 1)

16i<j6p1

jest podzielna przez p.


D. Liczba cp pokrywa si z liczb Ap3 . Teza wynika zatem z 8.2.1. 
8.3.3. Jeli p > 7 jest liczb pierwsz, to liczba
dp =

X
16i<j<k6p1

jest podzielna przez p2 .

1 2 bi bj kb (p 1)

Liczby wymierne.

8. Twierdzenie Wolstenholme i jego uoglnienia

57

D. Liczba dp pokrywa si z liczb Ap4 . Teza wynika zatem z 8.2.4. 


oooooooooooooooooooooooooooooooooooooooooooooooooooooooooooooooooooooo

8.4

Sumy odwrotnoci iloczynw

oooooooooooooooooooooooooooooooooooooooooooooooooooooooooooooooooooooo
8.4.1. Jeli p > 5 jest liczb pierwsz, to licznik uamka
X
a
1
=
b 16i<j6p1 ij

jest podzielny przez p.


D. Wynika to z rwnoci
Ap3 = (p 1)!

X
16i<j6p1

1
ij

oraz faktu 8.2.1. 


1
8.4.2. Niech n > 1 bdzie liczb naturaln. Rozwamy wszystkie uamki postaci pq
, gdzie
(p, q) = 1, 0 < p < q 6 n oraz p + q > n. Suma wszystkich takich uamkw jest rwna 12 .
([WaJ] 120(69), [B-zm] 71, [Dlt] 6/1999).

8.4.3. Suma wszystkich liczb postaci


cakowit. ([WaJ] 437(86)).

1
mn ,

gdzie m, n N, 1 6 m < n 6 1986, nie jest liczb

8.4.4. Jeli p > 7 jest liczb pierwsz, to licznik uamka


X
1
a
=
b 16i<j<k6p1 ijk

jest podzielny przez p2 .


D. Wynika to z rwnoci
X

Ap4 = (p 1)!

16i<j<k6p1

1
ijk

oraz faktu 8.2.4. 

8.4.5.

n
X

k=1 16i1 <<ik

1
= n.
i i ik
6n 1 2

([Bryn] 7.5).

D. 1 +

Pn

k=1

1
16i1 <<ik 6n i1 i2 ik

= 1+

1
1

1+

1
2

1 +

1
n

2
1

3
2

4
3

n
n1
= n. 

58

Liczby wymierne.

8. Twierdzenie Wolstenholme i jego uoglnienia

oooooooooooooooooooooooooooooooooooooooooooooooooooooooooooooooooooooo

8.5

Odwrotnoci liczb wzgldnie pierwszych: podstawowe fakty

oooooooooooooooooooooooooooooooooooooooooooooooooooooooooooooooooooooo
Niech m, k N. W tym podrozdziale i dalszych podrozdziaach przez Um oznaczamy
zbir wszystkich liczb naturalnych z przedziau [1, m] wzgldnie pierwszych z m. Przykady:
U5 = {1, 2, 3, 4}, U10 = {1, 3, 7, 9}. Ponadto stosujemy nastpujce oznaczenie.
S(m, k) =

X 1
iUm

ik

8.5.1. Dla kadej liczby naturalnej m 6= 2 licznik uamka


X
16i6m
(i, m) = 1

1
i

jest podzielny przez m.


D. Jeli m jest liczb nieparzyst, to teza wynika z 8.6.3. Podamy dowd nie korzystajcy z tego
faktu.
Dla m = 1 twierdzenie jest oczywiste. Zamy, e m > 2. Jeli m = 2n jest liczb parzyst, to
niech A = {i1 , i2 , . . . , is } bdzie zbiorem wszystkich liczb naturalnych mniejszych od n i wzgldnie
pierwszych z m. Jeli natomiast m = 2n + 1 jest liczb nieparzyst, to niech A = {i1 , i2 , . . . , is }
bdzie zbiorem wszystkich liczb naturalnych mniejszych od n + 1 i wzgldnie pierwszych z m. W obu
przypadkach mamy:
Um = {i1 , i2 , . . . , is , (m i1 ), (m i2 ), . . . , (m is )} .
Zatem

 X
X
X 1 X 1
1
1
m
=
+
=m
=
S(m, 1) =
i
i
mi
i(m i)
i(m i)
iUm

iA

iA

iA

i std wynika teza. 

8.5.2. Jeli k jest nieparzyst liczb naturaln, to dla kadej liczby naturalnej m 6= 2 licznik
uamka
X
1
ik
16i6m
(i, m) = 1

jest podzielny przez m.


D. Przy pewnym dodatkowym zaoeniu o k teza wynika z 8.6.3. Podamy dowd nie korzystajcy
z tego faktu.
Dla m = 1 twierdzenie jest oczywiste. Zamy, e m > 2. Jeli m = 2n jest liczb parzyst, to
niech A = {i1 , i2 , . . . , is } bdzie zbiorem wszystkich liczb naturalnych mniejszych od n i wzgldnie
pierwszych z m. Jeli natomiast m = 2n + 1 jest liczb nieparzyst, to niech A = {i1 , i2 , . . . , is }
bdzie zbiorem wszystkich liczb naturalnych mniejszych od n + 1 i wzgldnie pierwszych z m. W obu
przypadkach mamy:
Um = {i1 , i2 , . . . , is , (m i1 ), (m i2 ), . . . , (m is )} .

Liczby wymierne.

8. Twierdzenie Wolstenholme i jego uoglnienia

Zatem
S(m, k) =

59

 X
X 1
X 1
1
(m i)k + ik
=
+
=
.
ik
ik
(m i)k
ik (m i)k

iUm

iA

iA

Poniewa k jest nieparzyste, liczniki (m i)k + ik s podzielne przez m i std wynika teza. 

8.5.3. Jeli k jest parzyst liczb naturaln i m N, to teza 8.5.2 nie musi zachodzi.
Przykady. Liczby S(m, k) nie s podzielne przez m dla nastpujcych k i m:
k
2
4
6
8

m
3, 4, 6, 8, 9, 12, 15, 16, 18, 24, 27, 30, 32, 33, 36, 45, 48, 50, 51, 54, 60, 64, 66
3, 4, 5, 6, 8, 9, 10, 12, 15, 16, 18, 20, 24, 25, 27, 30, 32, 33, 35, 36, 40, 45, 48, 50
3, 4, 6, 7, 8, 9, 12, 14, 15, 16, 18, 21, 24, 27, 28, 30, 32, 33, 35, 36, 42, 45, 48, 49
3, 4, 5, 6, 8, 9, 10, 12, 15, 16, 18, 20, 24, 25, 27, 30, 32, 33, 35, 36, 40, 45, 48, 50

8.5.4 (Hipoteza). Jeli k jest liczb parzyst i m jest potg trjki, to licznik liczby S(m, k)
nie jest podzielny przez m.

8.5.5. Jeli m jest liczb parzyst, to licznik kadej z liczb S(3, m), S(4, m), S(8, m), S(9, m),
S(16, m) nie jest podzielny przez m.

8.5.6. Wykaz pewnych par (m, k) takich, e liczniki liczb S(m, k) s podzielne przez m3 .
k
1
3
5
7

m
39, 78, 155, 310, 465, 546, 793, 798, 930
37, 55, 110, 111, 222, 310, 333, 355, 407, 666, 710, 814, 930
5, 203, 406
7, 21, 39, 63, 67, 78, 117, 133, 146, 201, 234, 259, 273, 399,
438, 469, 518, 546, 603, 737, 777, 793, 798, 819, 876, 877
9 3, 5, 15, 30, 69, 178, 230, 345, 534, 690, 759, 890
11 11, 689
13 7, 13, 21, 39, 55, 59, 78, 91, 110, 133, 155, 182, 217, 231, 273,
275, 310, 355, 385, 399, 413, 429, 462, 465, 507, 550, 607, 627,
649, 651, 710, 715, 767, 770, 775, 793, 798, 806, 819, 858, 905, 930
15
17 17, 34, 68, 149, 311, 401, 802, 955
19 19, 39, 57, 78, 171, 203, 406, 546, 609, 674, 741, 793, 798
21 7, 11, 21, 55, 77, 110, 231, 329, 385, 462, 770, 987
23 23, 55, 110, 253, 275, 310, 355, 505, 550, 710
25 13, 25, 65, 70, 78, 95, 117, 130, 146, 155, 182, 190, 210, 234, 259,
260, 285, 310, 325, 327, 350, 351, 420, 438, 465, 475, 507, 518, 545,
546, 570, 585, 630, 650, 654, 702, 730, 763, 775, 777, 780, 793, 798,
855, 876, 910, 930, 950, 975, 981
27 7, 9, 21, 63, 177, 189, 413, 466, 531
29 5, 29, 145, 290, 580

60

Liczby wymierne.

8. Twierdzenie Wolstenholme i jego uoglnienia

8.5.7. Wykaz pewnych par (m, k) takich, e liczniki liczb S(m, k) s podzielne przez m4 .
k m
13 546
25 5, 39, 195, 390
27 3
33 55
49 5, 35
53 55

oooooooooooooooooooooooooooooooooooooooooooooooooooooooooooooooooooooo

8.6

Odwrotnoci liczb wzgldnie pierwszych: Twierdzenia Gessela

oooooooooooooooooooooooooooooooooooooooooooooooooooooooooooooooooooooo
Przypominamy, e Um jest zbiorem wszystkich liczb naturalnych nalecych do przedziau
[1, m] wzgldnie pierwszych z m. Ponadto:
X 1

S(m, k) =

iUm

ik

8.6.1 ([Gess]). Jeli a jest liczb cakowit wzgldnie pierwsz z m, to licznik uamka
(ak 1)S(m, k)
jest podzielny przez m.
D. Zbir {ai; i Um } zredukowany modulo m pokrywa si ze zbiorem Um . Stosujc relacj
przystawania modulo m dla liczb wymiernych (w sensie takim, jak w jednym z poprzednich rozdziaw)
mamy:
X 1
1
= k S(m, k) (mod m)
S(m, k)
(ai)k
a
iUm

i std wynika teza. 

8.6.2 ([Gess]). Jeli k jest liczb nieparzyst, to


2S(m, k) mkS(m, k + 1) (mod m2 ).
D. Stosujemy relacj przystawania modulo m dla liczb wymiernych (w sensie takim, jak w jednym
z poprzednich rozdziaw). Zauwamy najpierw, e

X 1
X ik + (m i)k
1
2S(m, k) =
+
=
.
ik
(m i)k
ik (m i)k
iUm
k

iUm

k1

Poniewa k jest nieparzyste, wic i + (m i) ki

m (mod m2 ). Std wynika, e

ik + (m i)k
kik1 m
1
k
= km
(mod m2 ).
k
k
i (m i)
i (m i)k
i(m i)k
1
1
Ale (m i)k ik (mod m), wic (mi)
k ik (mod m), std
1
1
2
km i(mi)k km ik+1 (mod m ). Zatem:

2S(m, k)

1
ik+1
(mod m) czyli

X kik1 m
X 1
km
(mod m2 )
k
k
i (1)
ik+1

iUm

i std wynika teza. 

1
i(mi)k

iUm

Liczby wymierne.

8. Twierdzenie Wolstenholme i jego uoglnienia

61

8.6.3 ([Gess]). Zamy, e dla kadej liczby pierwszej p dzielcej m liczba k nie jest podzielna przez p 1. Wtedy licznik uamka S(m, k) jest podzielny przez m.
D. Z zaoenia wynika, e dla kadej liczby pierwszej p dzielcej m istnieje liczba cakowita ap
taka, e p nie dzieli akp 1. Istnieje (na mocy twierdzenia chiskiego o resztach) a Z takie, e
a ap (mod p) dla wszystkich takich p. Wystarczy teraz zastosowa 8.6.1. 
8.6.4 ([Gess]). Zamy, e k jest liczb nieparzyst oraz, e dla kadej liczby pierwszej p
dzielcej m liczba k + 1 nie jest podzielna przez p 1. Wtedy licznik uamka S(m, k) jest
podzielny przez m2 .
D. Poniewa k + 1 nie jest podzielne przez p 1, wic p nie moe by rwne 2. Zatem m jest
liczb nieparzyst. Teza wynika zatem z 8.6.2 i 8.6.3. 
8.6.5 ([Gess]). Jeeli k i n s liczbami naturalnymi, to licznik uamka S(2n , k) jest podzielny
przez 2n1 .
8.6.6 ([Gess]). Jeeli k i n s liczbami naturalnymi i przy tym k jest nieparzyste, to licznik
uamka S(2n , k) jest podzielny przez 22(n1) .
oooooooooooooooooooooooooooooooooooooooooooooooooooooooooooooooooooooo

8.7

Twierdzenie Wolstenholme i inne twierdzenia

oooooooooooooooooooooooooooooooooooooooooooooooooooooooooooooooooooooo
8.7.1 (J. Wolstenholme 1862). Jeli p > 5 jest liczb pierwsz, to licznik uamka
a
b

jest podzielny przez p2 .

=1+

1
2

1
3

+ +

1
p1

([Dic1] 96, [HW4] 88, [ShCY] 60, [WyKM] 733-78, [Ri97] 38, [Tao] 27).

D. Sposb 1. Niech d = nwd(a, b). Wtedy a = da1 , b = db1 , dla pewnych wzgldnie pierwszych
liczb naturalnych a1 i b1 . Zauwamy, e


1
(p 1)! ab = (p 1)! 1 + 21 + 31 + + p1
= Ap2 ,
czyli a1 (p1)! = b1 Ap2 . Liczba Ap2 jest podzielna przez p2 (na mocy 8.2.3), zatem a1 jest podzielne
przez p2 i konsekwentnie a jest podzielne przez p2 .
Sposb 2. Wynika to z Twierdzenia Gessela 8.6.4 zastosowanego dla k = 1 i m = p. Moemy to
twierdzenie zastosowa gdy liczba k jest nieparzysta oraz p 1 - 2 = k + 1. 
U. Dla p = 3 powyszy fakt nie zachodzi: 1 +

1
2

= 32 , 9 - 3. 

8.7.2 (P. Ossowski 2001). Jeli p jest liczb pierwsz, to licznik uamka
a
b

=1+

1
2

1
3

+ +

1
(p1)

(gdzie nwd(a, b) = 1) moe by podzielny przez p3 . Najmniejsz tak liczb pierwsz jest
p = 16843, liczby a i b maj wtedy odpowiednio 7309 i 7308 cyfr. (Maple).

62

Liczby wymierne.

8. Twierdzenie Wolstenholme i jego uoglnienia

8.7.3. Jeli p > 5 jest liczb pierwsz, to licznik uamka


a
b

jest podzielny przez p.

=1+

1
22

1
32

+ +

1
(p1)2

([HW4] 90, [ShCY] 60, [Ri97] 38).

D. Sposb 1. Oznaczmy ai = 1 2 bi (p 1). Wwczas dla i < j mamy:


ai aj = (p 1)!1 2 bi b
j (p 1).
Zatem
1+

1
22

1
32

+ +

1
(p1)2

1
((p1)!)2

1
((p1)!)2

1
((p1)!)2

Pp1
i=1



a2i

Pp1
i=1

ai

2


2

i<j

A2p2 2(p 1)!Ap3

ai aj

i teza wynika z 8.2.1.


Sposb 2. Wynika to z Twierdzenia Gessela 8.6.3 zastosowanego dla k = 2 i m = p. Moemy to
twierdzenie zastosowa gdy p 1 - 3 = k + 1. 

8.7.4 (P. Ossowski 2001). Jeli p jest liczb pierwsz, to licznik uamka
a
b

=1+

1
22

1
32

+ +

1
(p1)2

(gdzie nwd(a, b) = 1) moe by podzielny przez p2 . Najmniejsz tak liczb pierwsz jest
p = 16843, liczby a i b maj wtedy odpowiednio 14618 i 14617 cyfr. (Maple).
8.7.5. Niech p > 2 bdzie liczb pierwsz oraz niech
a
b

=1+

1
23

1
33

+ +

1
(p1)3 ,

gdzie a i b s wzgldnie pierwszymi liczbami naturalnymi. Wtedy a jest podzielne przez p.


([Balt] 1994, wynika to z 8.5.2).
8.7.6. Niech p > 7 bdzie liczb pierwsz oraz niech
a
b

=1+

1
23

1
33

+ +

1
(p1)3 ,

gdzie a i b s wzgldnie pierwszymi liczbami naturalnymi. Wtedy a jest podzielne przez p2 .


D. Stosujemy twierdzenie Gessela 8.6.4 dla k = 3 i m = p. 
8.7.7 (P. Ossowski 2001). Jeli p jest liczb pierwsz, to licznik uamka
a
b

=1+

1
23

1
33

+ +

1
(p1)3

(gdzie nwd(a, b) = 1) moe by podzielny przez p3 . Najmniejsz tak liczb pierwsz jest
p = 37, kada z liczb a i b ma wtedy 43 cyfry. (Maple).

Liczby wymierne.

8. Twierdzenie Wolstenholme i jego uoglnienia

63

8.7.8. Niech p > 2 bdzie liczb pierwsz. Niech k bdzie liczb naturaln oraz niech
a
b

=1+

1
2k

1
3k

+ +

1
,
(p1)k

gdzie a i b s wzgldnie pierwszymi liczbami naturalnymi. Wtedy:


(1) Jeli p > k + 1, to p | a.
(2) Jeli k jest nieparzyste, to p | a.
(3) Jeli k jest nieparzyste i p > k + 2, to p2 | a.
D. (1). Twierdzenie Gessela 8.6.3 dla m = p.
(2). 8.5.2.
(3). Twierdzenie Gessela 8.6.4 dla m = p.
Podobne fakty udowodni w 1866 roku P. Frost i w roku 1901 Glaisher ([Dic1] 96-100). 

8.7.9 (P. Ossowski 2001). Jeli p jest liczb pierwsz, to licznik uamka
a
b

=1+

1
24

1
34

+ +

1
(p1)4

(gdzie nwd(a, b) = 1) moe by podzielny przez p2 . Najmniejsz tak liczb pierwsz jest
p = 37, kada z liczb a i b ma wtedy 56 cyfr. (Maple).
8.7.10 (P. Ossowski 2001). Jeli p jest liczb pierwsz, to licznik uamka
a
b

=1+

1
25

1
35

+ +

1
(p1)5

(gdzie nwd(a, b) = 1) moe by podzielny przez p3 . Najmniejsz tak liczb pierwsz jest
p = 5, wtedy a = 53 2063, b = 210 35 . (Maple).
8.7.11. Jeli p jest liczb pierwsz nie nalec do zbioru {2, 3, 7, 13, 53, 79, 157}, to licznik
uamka
1
1
1
1155 + 2155 + + (p1)155 ,
jest podzielny przez p2 .

([Zw] 2006).

8.7.12. Niech p > 2 bdzie liczb pierwsz. Niech k, n bd liczbami naturalnymi oraz niech
a
=
b

X
1 6 i 6 pn
(i, p) = 1

1
ik

gdzie a i b s wzgldnie pierwszymi liczbami naturalnymi. Wtedy:


(1) Jeli p > k + 1, to pn | a.
(2) Jeli k jest nieparzyste, to pn | a.
(3) Jeli k jest nieparzyste i p > k + 2, to p2n | m.

64

Liczby wymierne.

8. Twierdzenie Wolstenholme i jego uoglnienia

D. (1). Twierdzenie Gessela 8.6.3 dla m = p.


(2). 8.5.2.
(3). Twierdzenie Gessela 8.6.4 dla m = p. 
1
1p

8.7.13. Niech p > 5 bdzie liczb pierwsz i niech


p3 | a. ([Ibe] 2005).

1
2p

1
3p

+ +

1
(p1)p

= ab . Wtedy

F I. mieszek, Twierdzenie Wolstenholme i jego uoglnienia, pr. magisterska, UMK, Toru 2001.

oooooooooooooooooooooooooooooooooooooooooooooooooooooooooooooooooooooo

8.8

Rne zadania

oooooooooooooooooooooooooooooooooooooooooooooooooooooooooooooooooooooo
8.8.1. Niech

a
b

=1+

1
2

1
3

+ + n1 , gdzie a, b N, nwd(a, b) = 1.

(1) Jeli n = 20, to 5 | a. Czy 5 | a dla n = 100?

([WyKM] 827,831).

(2) Liczba a jest podzielna przez 5 dla n = 4, 20, 24. Nie ma innych liczb naturalnych n
mniejszych od 10000 speniajcych t wasno. (Maple).
(3) Liczba a jest podzielna przez 3 dla n = 2, 7, 22. Nie ma innych liczb naturalnych n
mniejszych od 10000 speniajcych t wasno. (Maple)
(4) Liczba a nie jest podzielna przez 3 dla n = 67.
8.8.2. Licznik sumy 1 +
liczb zoon. ([Zw] 2003).

1
2

1
3

+ +

1
131

([OM] Bugaria 2004)

zapisanej w postaci uamka nieskracalnego jest

1
8.8.3. Licznik kadej liczby postaci 2k=1 2k1
, zapisanej w postaci uamka nieskracalnego,
2n
jest podzielny przez 2 . ([Cmj] 16(1)(1985) z.243).

1
1
8.8.4. Licznik kadej liczby postaci n1 + n+1
+ + n+m
(gdzie n, m N), zapisanej w postaci
uamka nieskracalnego, jest liczb nieparzyst. ([Cmj] 29(3)(1998) z.601).

8.8.5 (Glaisher 1901, [Dic1] 100). Jeli p > 3 bdzie liczb pierwsz, to
1+

1
32n

1
52n

+ +

1
(p2)2n

0 lub

1
2

(mod p),

odpowiednio gdy p 1 - 2n lub p 1 | 2n.


8.8.6. Jeeli n N, to przez pn , qn oznaczmy wzgldnie pierwsze liczby naturalne takie, e
2
1

Wykaza, e jeeli n > 3, to 8 | pn .

22
2

23
3

+ +

2n
n

([Kw] 12/77 M 434).

pn
qn .

Liczby wymierne

8. Twierdzenie Wolstenholme i jego uoglnienia

65

8.8.7. Jeeli n N, to przez pn , qn oznaczmy wzgldnie pierwsze liczby naturalne takie, e


2
1

22
2

23
3

+ +

2n
n

pn
qn .

Wykaza, e dla kadego k N istnieje n N takie, e liczby pn+1 , pn+2 , pn+3 , . . . s podzielne
przez 2k . ([Ko01]).
8.8.8. Istnieje nieskoczenie wiele liczb naturalnych n takich, e licznik uamka nieskracalnego 11 + 12 + + n1 nie jest potg liczby pierwszej. ([OM] Rosja 2002).

8.8.9. Dla kadej liczby pierwszej p licznik uamka


([Zw] 2006).

p1
P
k=0

p12
k

jest podzielny przez p.

F L. E. Dickson, Symmetric functions of 1, 2, . . . , p 1 modulo p, [Dic1] 95-103.

Liczby postaci x1 /x2 + x2 /x3 + + xs /x1

Niech s N. Interesowa nas bd dodatnie liczby wymierne postaci


x1
x2

x2
x3

+ +

xs
x1 ,

gdzie x1 , x2 , . . . , xs s liczbami naturalnymi. Zbir wszystkich takich dodatnich liczb wymiernych oznacza bdziemy przez Bs . W szczeglnoci interesowa nas bd liczby naturalne tej
postaci. Zbir wszystkich takich liczb naturalnych oznacza bdziemy przez As . Mamy wic
As = Bs N, A1 = B1 = {1} oraz
(

Bs =

Q+ ;

q=

x1 ,...,xs N

x1
x2

x2
x3

+ +

xs
x1

n N;

As =

x1
x2

n=

x1 ,...,xs N

x2
x3

+ +

xs
x1

dla s > 2. Przez Q+ oznaczamy zbir wszystkich liczb wymiernych wikszych od zera.
oooooooooooooooooooooooooooooooooooooooooooooooooooooooooooooooooooooo

9.1

Podstawowe wasnoci zbiorw Bs i As

oooooooooooooooooooooooooooooooooooooooooooooooooooooooooooooooooooooo
9.1.1. Niech s N, q Q+ . Jeli q Bs , to q > s. W szczeglnoci, jeli liczba naturalna
n naley do zbioru As , to n > s.
x1
x2

+ xx23 + + xx1s , dla pewnych x1 , . . . , xs N. Z nierwnoci


pomidzy redni arytmetyczn i redni geometryczn liczb xx21 , . . . , xx1s otrzymujemy:

D. Niech q Bs . Wtedy q =

q =s

1
s

x1
x2

x2
x3

xs
x1

>s

q
s

x1 x2
x2 x3

xx1s = s

1 = s.

Zatem q > s. Std oraz z faktu, e As = Bs N wynika, e jeli n As , to n > s. 

9.1.2. Niech s N. Wtedy s As . Jeli x1 , . . . , xs s liczbami naturalnymi takimi, e


s = xx21 + xx23 + + xx1s , to x1 = x2 = = xs .
+ xx23 + + xx1s , dla x1 = x2 = = xs = 1. Zamy teraz,
e s =
+ ++
gdzie x1 , . . . , xs N. Wtedy rednia arytmetyczna liczb xx21 , xx23 . . . . , xx1s , jest
rwna redniej geometrycznej tych liczb. Wszystkie wic te liczby s jednakowe. Niech a = xx12 = xx23 =
= xx1s . Wtedy s = sa, wic a = 1 i std x1 = x2 = = xs . 

D. Liczba s naley do As , gdy s =


x1
x2

x2
x3

xs
x1 ,

x1
x2

9.1.3. Niech s N. Jeli x1 , . . . , xs s liczbami naturalnymi takimi, e s =


oraz nwd(x1 , . . . , xs ) = 1, to x1 = x2 = = xs = 1. (Wynika z 9.1.2).

x1
x2

+ xx23 + + xx1s

9.1.4 (Bondarenko 2000). Kada liczba naturalna n > 12 naley do zbioru A12 .

66

([Bond]).

9. Liczby postaci x1 /x2 + x2 /x3 + + xs /x1

Liczby wymierne.

67

9.1.5. Niech s > 2. Jeli q jest dodatni liczb wymiern, to nastpujce warunki s rwnowane.
(1) q Bs .
x1
x2
y1
y2

(2) q =
(3) q =

+
+

x2
x3
y2
y3

+ +
+ +

xs
x1 ,
ys
y1 ,

dla pewnych x1 , . . . , xs N takich, e nwd(x1 , . . . , xs ) = 1.


dla pewnych y1 , . . . , ys Q+ .

(4) q = u1 + u2 + + us , dla pewnych u1 , . . . , us Q+ takich, e u1 u2 us = 1.


+ aa23 + + aa1s , gdzie a1 , . . . , as N. Niech d =
nwd(a1 , . . . , as ). Istniej wtedy liczby naturalne x1 , . . . , xs takie, e a1 = x1 d, a2 = x2 d, . . . , as = xs d.
Wtedy nwd(x1 , . . . , xn ) = 1 oraz xx21 + xx23 + + xx1s = xx12 dd + xx32 dd + + xx1s dd = aa12 + aa23 + + aa1s = q.
Wykazalimy wic implikacj (1) (2). Implikacja (2) (1) jest oczywista.
(1) (3). Jest oczywiste, e zachodzi implikacja (1) (3). Wykaemy implikacj (3) (1).
Niech q = yy12 + yy23 + + yy1s , gdzie y1 , . . . , ys Q+ . Niech d bdzie wsplnym mianownikiem wszystkich
liczb wymiernych y1 , . . . , ys . Wtedy y1 = xd1 , y2 = xd2 , . . . , ys = xds , dla pewnych x1 , . . . , xs N. Mamy
/d
/d
y2
ys
y1
wtedy xx21 + xx32 + + xx1s = xx21 /d
+ xx32 /d
+ + xx1s /d
/d = y2 + y3 + + y1 = q. Zatem q Bs .

D. (1) (2). Zamy, e q Bs . Niech q =

a1
a2

(3) (4). Zamy, e q = yy21 + yy32 + + yy1s , gdzie y1 , . . . , ys Q+ . Niech u1 = yy12 , u2 = yy23 , . . . ,
us = yy1s . Wtedy u1 , . . . , us s dodatnimi liczbami wymiernymi, u1 us = 1 oraz q = u1 + + us .
Wykazalimy wic implikacj (1) (4).
Niech teraz q = u1 + + us , gdzie u1 , . . . , us s dodatnimi liczbami wymiernymi takimi, e
u1 us = 1. Niech
y1 = 1, y2 =
Wtedy

y1
y2

y2
y3

+ +

ys
y1

1
u1 ,

y3 =

1
u 1 u2 ,

. . . , ys1 =

1
u1 u2 us2 ,

ys =

1
u1 u2 us1 .

= u1 + u2 + + us = q. 

Jeli w 9.1.5 zaoymy dodatkowo, e q jest liczb naturaln, to otrzymamy nastpujce


twierdzenie.
9.1.6. Niech s > 2. Jeli n jest liczb naturaln, to nastpujce warunki s rwnowane.
(1) n As .
(2) n =
(3) n =

x1
x2
y1
y2

+
+

x2
x3
y2
y3

+ +
+ +

xs
x1 ,
ys
y1 ,

dla pewnych x1 , . . . , xs N takich, e nwd(x1 , . . . , xs ) = 1.


dla pewnych y1 , . . . , ys Q+ .

(4) n = u1 + u2 + + us , dla pewnych u1 , . . . , us Q+ takich, e u1 u2 us = 1.


Nastpne fakty s wnioskami z twierdze 9.1.5 i 9.1.6.
9.1.7. Kada liczba postaci

xs1 +xs2 ++xss


x1 x2 xs ,

xs1 +xs2 ++xss


,
x1 x2 xs
s
x
ui = x1 x2ixs ,

D. Oznaczmy: q =

gdzie x1 , . . . , xs Q+ , naley do zbioru Bs .

gdzie x1 , . . . , xs Q+ . Pokaemy, e q Bs .

(Sposb I). Niech


dla i = 1, . . . , s. Wtedy u1 , . . . , us Q+ , u1 u2 us = 1 oraz
q = u1 + u2 + + us . Teza wynika zatem z twierdenia 9.1.5.

s2
2
1
(Sposb II). Oznaczmy w = x1 x2 xs i niech yi = w1 xsi xs1
i+1 xi+2 xi+s2 xi+s1 , dla i =
1, 2, . . . , s przy czym xs+j = xj dla j N. Wtedy y1 , . . . , ys s liczbami naturalnymi oraz yy12 + yy32 +
+

ys
y1

xs1 +xs2 ++xss


x1 x2 xs

= q. Zatem q Bs . 

68

9. Liczby postaci x1 /x2 + x2 /x3 + + xs /x1

Liczby wymierne.

9.1.8. Niech s > 2. Kada liczba postaci


Q+ , naley do zbioru Bs .

s1
xs1
x2 +xs1
x3 ++xs1
x1
1
2
s1 xs +xs
,
x1 x2 xs

x1s1 x2 +x2s1 x3 ++xs1


x +xs1
x1
s
s1 s
,
x1 x2 xs

gdzie x1 , . . . , xs
xs1 x

i+1
gdzie x1 , . . . , xs Q+ . Niech ui = xi1 x2 x
,
s
+
dla i = 1, . . . , s, przy czym xs+1 = x1 . Wtedy u1 , . . . , us Q , u1 u2 us = 1 oraz q = u1 + u2 +
+ us . Teza wynika zatem z twierdenia 9.1.5. 

D. Oznaczmy: q =

9.1.9. Bn + Bm Bn+m , dla n, m N.


D. Niech a Bn , b Bm . Pokaemy, e a + b Bn+m .
(Sposb I). Z twierdzenia 9.1.5 wynika, e istniej dodatnie liczby wymierne u1 , . . . , un oraz
v1 , . . . , vm takie, e u1 un = 1, v1 vm = 1, a = u1 + un i b = v1 + + vm . Wtedy
u1 u2 un v1 v2 vm = 1 oraz a + b = u1 + + un + v1 + vm . Teza wynika wic z twierdzenia 9.1.5.
(Sposb II). Istniej liczby naturalne x1 , . . . , xn , y1 , . . . , ym takie, e a = xx12 + + xxn1 , b =
ym
y1
y2 + y1 . Mamy wtedy
a+b=

x1 y1
x2 y1

x2 y1
x3 y1

+ +

xn y1
x1 y1

y1 x1
y2 x1

y2 x1
y3 x1

+ yym1 xx11 .

Zatem a + b Bn+m . 

9.1.10. Bm Bn Bmn , dla n, m N.


D. Niech a Bm , b Bn . Pokaemy, e ab Bnm .
(Sposb I). Z twierdzenia 9.1.5 wynika, e istniej dodatnie liczby wymierne u1 , . . . , um oraz
v1 , . . . , vn takie, e u1 um = 1, v1 vn = 1, a = u1 + um i b = v1 + + vn . Niech wij = ui vj ,
dla i = 1, . . . , m, j = 1, . . . , n. Iloczyn wszystkich liczb postaci wij jest rwny 1 i ich suma wynosi ab.
Teza wynika wic z twierdzenia 9.1.5.
(Sposb II). Istniej liczby naturalne x1 , . . . , xm , y1 , . . . , yn takie, e a = xx12 + + xxm1 , b =
y1
yn
y2 + y1 . Przyjmijmy:

z(p1)n+i = xni+1
xi1
p
p+1 yi ,
dla p = 1, 2, . . . , m, i = 1, 2, . . . , n, przy czym xm+1 = x1 . Mamy wtedy mn liczb naturalnych
z(p1)n+i
xp
yi
z1 , z2 , . . . , zmn . Zauwamy, e z(p1)n+i+1
= xp+1
yi+1 , dla p = 1, 2, . . . , m oraz i < n. Ponadto,
z(p1)n+n
zpn+1

xp yn
xp+1 y1 ,

dla p = 1, 2, . . . , m. Std wynika, e


z1
z2

z2
z3

+ +

zmn
z1

x1
x2

+ +

xm
x1



y1
y2

+ +

yn
y1

= ab.

Zatem ab Bmn . 

Z powyszych faktw wynika:


9.1.11. Am + An Am+n , Am An Amn , dla m, n N.
9.1.12. Jeli q Bs , to q + 1 Bs+1 . Jeli n As , to n + 1 As+1 .

Liczby wymierne.

9. Liczby postaci x1 /x2 + x2 /x3 + + xs /x1

69

oooooooooooooooooooooooooooooooooooooooooooooooooooooooooooooooooooooo

9.2

Zbir B2

oooooooooooooooooooooooooooooooooooooooooooooooooooooooooooooooooooooo
9.2.1. Jedyn liczb naturaln n nalec do zbioru B2 jest n = 2. Innymi sowy: A2 = {2}.
D. Niech n A2 = B2 N. Niech n = xy + xy , gdzie x, y N, nwd(x, y) = 1. Wtedy x2 + y 2 = nxy.
Przypumy, e x > 2. Istnieje wtedy liczba pierwsza p taka, e p | x. Wtedy prawa strona rwnoci
x2 +y 2 = nxy jest podzielna przez p, wic (poniewa p | x2 i p | x2 +y 2 ) liczba y rwnie jest podzielna
przez p. To jest jednak sprzecznoci, gdy nwd(x, y) = 1. Zatem x = 1 i analogicznie y = 1. Std
n = 11 + 11 = 2. 

9.2.2. Niech p bdzie liczb pierwsz i n liczb naturaln. Liczba


i tylko wtedy, gdy n = 2p lub n = p2 + 1.
2p
p

= 2 = 11 + 11 i
typu liczb w zbiorze B2 nie ma.
Zamy, e np B2 . Niech np = xy + xy =

D. Oczywicie liczby

p2 +1
p

x2 +y 2
xy ,

p
1

1
p

n
p

naley do zbioru B2 wtedy

nale do B2 . Pokaemy, e innych tego

gdzie x, y N, nwd(x, y) = 1. Wtedy

p(x2 + y 2 ) = nxy.
2

+y
Zamy najpierw, e p | n. Niech n = pa, a N. Wtedy x2 + y 2 = axy, wic a = x xy
= xy + xy
A2 . Ale A2 = {2}, wic a = 2. Jeli wic p | n, to n = 2p.
Zamy teraz, e p - n. Wtedy p | xy, wic p | x lub p | y. Dla ustalenia uwagi niech p | x. Wtedy
p - y, gdy nwd(x, y) = 1. Niech x = p a, a N, p - a, > 1. Wtedy p2+1 a2 + py 2 = np ay. Std
wynika, e = 1 (bowiem gdy > 2, to mamy sprzeczno z tym, e p - y). Zatem (pa)2 + y 2 = nay
Przypumy, e a > 2. Niech q bdzie liczb pierwsz dzielc a. Wtedy z rwnoci (pa)2 + y 2 = nay
wynika, e q | y; wbrew temu, e nwd(x, y) = 1. Zatem a = 1, tzn. x = p. Mamy wic p2 + y 2 = ny,
p - n, p - y. Jeli y > 2, to mamy oczywist sprzeczno. Zatem y = 1 i std n = p2 + 1. 

9.2.3. Niech n N. Liczba


(Wynika z 9.2.2 dla p = 2).

n
2

naley do zbioru B2 wtedy i tylko wtedy, gdy n = 4 lub n = 5.

9.2.4. Niech n N. Liczba


(Wynika z 9.2.2 dla p = 3).

n
3

naley do zbioru B2 wtedy i tylko wtedy, gdy n = 6 lub n = 10.

9.2.5. Niech p bdzie liczb pierwsz oraz s, n N, p - n. Liczba


wtedy i tylko wtedy, gdy n = p2s + 1.
2s

n
ps

naley do zbioru B2

B2 , gdy wtedy pns = p ps+1 = p1 + p1s . Zamy teraz, e


niech x, y bd wzgldnie pierwszymi liczbami naturalnymi takimi, e xy + xy = pns . Wtedy

D. Jeli n = p2s + 1, to

(1)

n
ps

n
ps

B2 i

ps (x2 + y 2 ) = nxy.

Poniewa p - n, wic p | x lub p | y. Zmieniajc ewentualnie kolejno wystpowania liczb x, y, moemy


zaoy, e p | x. Wtedy ps | x oraz p - y. Niech x = pt u, u N, p - u, t > s. Jeli t > s, to prawa

70

Liczby wymierne.

9. Liczby postaci x1 /x2 + x2 /x3 + + xs /x1

strona rwnoci (1) jest podzielna przez p i lewa strona tej rwnoci nie jest podzielna przez p. Zatem
t = s i mamy p2s u2 + y 2 = nuy. Jeli u > 2, to istnieje liczba pierwsza q dzielca u i wtedy z rwnoci
p2s u2 + y 2 = nuy wynika, e q | y wbrew temu, e nwd(x, y) = 1. Zatem u = 1, czyli x = ps . Mamy
wic rwno p2s + y 2 = ny. Jeli y > 2, to mamy oczywist sprzeczno z tym, e nwd(x, y) = 1.
Zatem y = 1. Ostatecznie n = p2s + 1. 

9.2.6. Niech a, b, c, d bd liczbami naturalnymi takimi, e a 6 b, c 6 d, nwd(a, b) = 1 oraz


nwd(c, d) = 1. Jeli ab + ab = dc + dc , to a = c i b = d.
+ ab = dc + dc . Wtedy cd(a2 + b2 ) = ab(c2 + d2 ). Poniewa nwd(a, b) = 1 i
nwd(c, d) = 1, wic nwd(ab, a2 + b2 ) = 1 i nwd(cd, c2 + d2 ) = 1. Zatem ab | cd i cd | ab, czyli ab = cd
i std a2 + b2 = c2 + d2 . Std dalej mamy: (b a)2 = b2 2ab + a2 = d2 2cd + c2 = (d c)2 , czyli
b a = d c = u, gdzie u > 0. Zatem b = a + u, d = c + u. Ale ab = cd, wic a2 + ua = c2 + cu i std
(a c)(a + c + u) = 0. Poniewa a + c + u > 0, wic a = c i std wynika, e b = d. 

D. Zamy, e

a
b

oooooooooooooooooooooooooooooooooooooooooooooooooooooooooooooooooooooo

9.3

Zbir B3 i liczby (a3 + b3 + c3 )/abc

oooooooooooooooooooooooooooooooooooooooooooooooooooooooooooooooooooooo
Przypomnijmy, e B3 jest zbiorem wszystkich dodatnich liczb wymiernych xy + yz + xz , gdzie
x, y, z N, natomiast A3 jest zbiorem wszystkich naturalnych liczb tej postaci. Wiemy ju, e
A1 = {1}, A2 = {2}. Oczywicie 3 A3 . Do zbioru A3 nale rwnie inne liczby naturalne,
2
9
na przykad 5 = 21 + 24 + 41 lub 6 = 12
+ 12
9 + 2 . Wykaemy w nastpnym podrozdziale, e
zbir A3 jest nieskoczony. W tym podrozdziale wykaemy, e dodatnia liczba wymierna q
3
3 +c3
naley do zbioru B3 wtedy i tylko wtedy, gdy q = a +b
dla pewnych liczb naturalnych
abc
a, b, c.
9.3.1. Niech q bdzie liczb wymiern tak, e x2 z + y 2 x + z 2 y = qxyz, dla pewnych liczb
cakowitych x, y, z. Niech
a = xyz(x + y + z)(x2 + y 2 + z 2 xz yz xy),
b

= (xz + yz + xy)(x2 y 2 + z 2 x2 + y 2 z 2 y 2 zx yzx2 z 2 yx),

= (x2 y 4 + y 2 z 4 + z 2 x4 ) xyz(x2 y + y 2 z + z 2 x).

Wtedy a, b, c s liczbami cakowitymi takimi, e a3 + b3 + c3 = qabc.

([BrG], [Rus1]).

D. Standardowy rachunek; sprawdziem to przy pomocy Maple. 


9.3.2. Zachodzi rwno zbiorw:
B3 =

x
y

y
z

+ xz ; x, y, z N =

a3 +b3 +c3
;
abc

a, b, c N .

([BrG], [Rus1]).

D. Oznaczmy zbir po prawej stronie przez C3 . Inkluzja C3 B3 wynika z 9.1.7. Niech q B3 ,


niech q = xy + yz + xz , gdzie x, y, z N. Moemy zaoy, e nwd(x, y, z) = 1. Jeli x = y = z = 1, to
3

+1
q = 3 i wtedy 3 = 1 +1
. Dalej moemy wic zaoy, e (x, y, z) 6= (1, 1, 1). Z rwnoci q = xy + yz + xz
111
wynika, e x2 z + y 2 x + z 2 y = qxyz. Zatem a3 + b3 + c3 = qabc, gdzie liczby a, b, c s zdefiniowane
w twierdzeniu 9.3.1. Korzystajc z klasycznych nierwnoci atwo stwierdzamy, e a, b, c N. Zatem
q C3 i tym samym wykazalimy, e B3 C3 . Ostatecznie B3 = C3 . 

Z powyszych faktw otrzymujemy:

Liczby wymierne.

9. Liczby postaci x1 /x2 + x2 /x3 + + xs /x1

71

9.3.3 (Erdos, Niven 1946). Niech q bdzie dodatni liczb wymiern. Nastpujce warunki
s rwnowane.
(1) q B3 tzn. istniej liczby naturalne x, y, z takie, e q =
(2) Istniej liczby naturalne x, y, z takie, e q =

x
y

y
z

z
x oraz
= xy + yz

(4) Istniej dodatnie liczby wymierne a, b, c takie, e q =

(6) Istniej liczby naturalne a, b, c takie, e q =

y
z

(3) Istniej dodatnie liczby wymierne x, y, z takie, e q

(5) Istniej liczby naturalne a, b, c takie, e q =

x
y

+ xz .

nwd(x, y, z) = 1.
+ xz .

a3 +b3 +c3
.
abc

a3 +b3 +c3
.
abc
a3 +b3 +c3
oraz
abc

nwd(a, b, c) = 1.

([BrG], [Rus1]).

9.3.4. Niech n N. Nastpujce warunki s rwnowane.


(1) n A3 tzn. istniej liczby naturalne x, y, z takie, e n =
(2) Istniej liczby naturalne x, y, z takie, e n =

x
y

y
z

(4) Istniej dodatnie liczby wymierne a, b, c takie, e n =

(6) Istniej liczby naturalne a, b, c takie, e n =

z
x oraz
= xy + yz

(3) Istniej dodatnie liczby wymierne x, y, z takie, e n

(5) Istniej liczby naturalne a, b, c takie, e n =

x
y

y
z

+ xz .

nwd(x, y, z) = 1.
+ xz .

a3 +b3 +c3
.
abc

a3 +b3 +c3
.
abc
a3 +b3 +c3
oraz
abc

nwd(a, b, c) = 1.

(7) Istniej parami wzgldnie pierwsze liczby naturalne a, b, c takie, e n =


([Mon] 53(4)(1946) 223-224, [BrG], [Bond], [Rus1]).

a3 +b3 +c3
.
abc

9.3.5 (Erdos, Niven 1946). Niech x, y, z bd liczbami naturalnymi takimi, e liczba xy +


+ xz jest naturalna oraz nwd(x, y, z) = 1. Niech a = nwd(x, y), b = nwd(y, z), c =
nwd(z, x). Wtedy liczby a, b, c s parami wzgldnie pierwsze oraz x = c2 a, y = a2 b i z = b2 c.
([Mon] 53(4)(1946) 223-224).
y
z

D. (Erdos, Niven [Mon] 1946). Poniewa nwd(x, y, z) = 1, wic jest oczywiste, e liczby a, b, c s
parami wzgldnie pierwsze. Niech xy + yz + xz = m N. Mamy wtedy rwno
(1)

x2 z + y 2 x + z 2 y = mxyz.

Cz I. Pokaemy najpierw, e a2 | y. Jeli a = 1, to nie ma czego wykazywa. Zamy, e a > 2.


Niech a = pr11 prss bdzie rozkadem kanonicznym liczby a. Niech i {1, 2, . . . , s} i oznaczmy p = pi ,
r = ri . Wwczas p | x oraz p | y (gdy p | a = nwd(x, y)). Std p - z (bo nwd(x, y, z) = 1). Niech
x = p u, y = p v, gdzie u, v N, p - u, p - v, > r > 1, > r > 1. Wstawiajc to do rwnoci (1)
otrzymujemy rwno
p2 u2 z + p2+ v 2 u + p z 2 v = mp+ uvz,
z ktrej jasno wynika, e = 2. Zatem a = nwd(x, y) = nwd(p u, p v) = nwd(p u, p2 v) =
2
p nwd(u, p v) = p w, gdzie p - w. Std wnioskujemy, e = r i std, e = 2 = 2r. Zatem (pr )
ri 2
dzieli y. Dla kadego wic i ze zbioru {1, 2, . . . , s} mamy podzielno (pi ) | y. To implikuje, e liczba
Qs
2
a2 = i=1 (pri i ) dzieli liczb y. W ten sam sposb pokazujemy, e b2 | z, c2 | x. Zanotujmy:
(2)

c2 | x,

a2 | y,

b2 | z.

72

9. Liczby postaci x1 /x2 + x2 /x3 + + xs /x1

Liczby wymierne.

Cz II. Poniewa a | x, c2 | x oraz nwd(a, c2 ) = 1, wic ac2 | x. Analogicznie ba2 | y, cb2 | z.


Zatem x = iac2 , y = jba2 , z = kcb2 , dla pewnych i, j, k N. Pokaemy, e i = j = k = 1.
Przypumy, e i > 2. Istnieje wtedy liczba pierwsza p taka, e p | i. Wtedy p | x (bo x = iac2 ). Z
rwnoci (1) wynika wic, e p | z 2 y czyli, e p | y lub p | z.
Przypumy, e p | y. Wtedy p - z (bo p | x, p | y oraz nwd(x, y, z) = 1). Ponadto, p | a = nwd(x, y).
Ale a = nwd(x, y) = nwd(iac2 , jba2 ) = nwd(ic2 , jba)a, wic nwd(ic2 , jba) = 1. Tymczasem liczba
nwd(ic2 , jba) jest podzielna przez p (bo p | i oraz p | a). Sprzeczno ta implikuje, e p - y.
Zatem p | z, p | x oraz p - y. Std wynika, e p | c = nwd(z, x). Niech x = p u, z = pw, gdzie
u, w N, p - u, p - w, > 1, > 1. Wstawiajc to do rwnoci (1) otrzymujemy rwno
p2+ u2 w + p uy 2 + p2 w2 y = mp+ uyw,
z ktrej wynika, e = 2. Std dalej mamy: p2 u = p u = x = iac2 = iap2 w2 = p2+1 r, dla
pewnego r N. Zatem p | u wbrew temu, e p - u.
Otrzymana sprzeczno implikuje, e i = 1. Analogicznie dowodzimy, e j = 1 oraz k = 1. Zatem
x = ac2 , y = ba2 , z = cb2 i to koczy dowd. 

9.3.6. Niech x, y, z N. Jeli


([OM] Serbia-Czrnogra 2004).

x
y

y
z

z
x

jest liczb naturaln, to xyz jest szecianem.

D. Niech d = nwd(x, y, z), x = dx1 , y = dy1 , z = dz1 , gdzie x1 , y1 , z1 N, nwd(x1 , y1 , z1 ) = 1.


Poniewa xy11 + yz11 + xz11 = xy11dd + yz11 dd + xz11dd = xy + yz + xz , wic xy11 + yz11 + xz11 jest liczb naturaln. Z
twierdzenia 9.3.5 wynika wic, e istniej liczby naturalne a, b, c takie, e x1 = ac2 , y1 = ba2 , z1 = cb2 .
Mamy zatem xyz = (dx1 )(dy1 )(dz1 ) = d3 (ac2 )(ba2 )(cb2 ) = (abcd)3 , czyli xyz jest szecianem liczby
naturalnej. 
oooooooooooooooooooooooooooooooooooooooooooooooooooooooooooooooooooooo

9.4

Nieskoczono zbioru A3

oooooooooooooooooooooooooooooooooooooooooooooooooooooooooooooooooooooo
Wiemy (patrz 9.3.4), e zbir A3 pokrywa si ze zbiorem wszystkich liczb naturalnych
3
3 +c3
postaci a +b
, gdzie a, b, c N. Moemy nawet zaoy, e liczby a, b, c s parami wzgldabc
3
3 +c3
jest nieskoczenie wiele.
nie pierwsze. Wykaemy teraz, e liczb naturalnych postaci a +b
abc
Wykaemy to nawet przy dodatkowym zaoeniu, e c = 1. Przedstawione tu fakty i ich dowody pochodz z rozwizania zadania E682 z czasopima [Mon] 53(4)(1946) 223-224, podanego
przez Erdosa i Nivena.
9.4.1 (Erdos, Niven 1946). Niech a, b bd liczbami naturalnymi takimi, e:
(a) nwd(a, b) = 1;
(b) a < b;
(c) ab | a3 + b3 + 1.
Wtedy a | b3 + 1. Oznaczmy u =
liczbami naturalnymi oraz:

b3 +1
a ,

m1 =

a3 +b3 +1
,
ab

(1) nwd(b, u) = 1;
(2) b < u;
(3) bu | b3 + u3 + 1, tzn. m2 jest liczb naturaln;
(4) m1 < m2 .

([Mon] 53(4)(1946) 223-224).

m2 =

b3 +u3 +1
.
bu

Wtedy u i m1 s

Liczby wymierne.

9. Liczby postaci x1 /x2 + x2 /x3 + + xs /x1

73

D. (1). Poniewa au = b3 + 1, wic 1 = au + (b2 )b, czyli nwd(b, u) = 1.


(2). Przypumy, e b > u. Wtedy: b3 > ab2 > ab > au = b3 + 1 i mamy sprzeczno: b3 > b3 + 1.
(3). Poniewa u | b3 + 1, wic u | b3 + u3 + 1. Naley wic tylko pokaza, e b | b3 + u3 + 1
czyli, e b | u3 + 1. Z zaoenia b dzieli a3 + 1. Niech a3 + 1 = vb, gdzie v N. Mamy wtedy:
u3 + 1 = u3 + (ua)3 + 1 (ua)3 = u3 (1 + a3 ) + 1 (b3 + 1)3 = u3 vb b9 + 3b6 3b3 , a zatem b | u3 + 1
i ostatecznie bu | b3 + u3 + 1.
3

+ua
+1
+1
= a ab
= a b+u , m2 = b +u
= ua+u
= a+u
(4). Zauwamy, e m1 = a +b
ab
bu
bu
b . Naley wic
2
2
pokaza, e a + u < a + u czyli, e a(a 1) < u(u 1). Ale to jest oczywiste, gdy 1 6 a < u. 
3

+c
9.4.2. Istnieje nieskoczenie wiele liczb naturalnych postaci a +b
, gdzie a, b, c N. Istabc
nieje nawet nieskoczenie wiele liczb naturalnych tej postaci speniajcych warunek c = 1.
(Wynika to z twierdzenia 9.4.1).

9.4.3. Zbir A3 jest nieskoczony. Innymi sowy, istnieje nieskoczenie wiele liczb naturalnych postaci xy + yz + xz , gdzie x, y, z N. (Jest to konsekwencja faktw 9.4.2 i 9.3.4).
9.4.4 (Dofs 1995). Niech t bdzie dowoln liczb naturaln i niech x = a2 b, y = b2 c, z = c2 a,
gdzie a = 2, b = t2 t + 1, c = t2 + t + 1. Wtedy
x
y

y
z

z
x

= t2 + 5.

Oznacza to, e kada liczba naturalna postaci t2 + 5, gdzie t N, naley do zbioru A3 . Std
w szczeglnoci wynika, e zbir A3 jest nieskoczony. ([Dofs], [Bond]).
9.4.5. Jeli s > 3, to zbir As jest nieskoczony.
D. Wiemy (na mocy 9.4.3), e zbir A3 jest nieskoczony. Udowodnilimy (patrz 9.1.12), e jeli
n As , to n + 1 As+1 . Std wynika, e zbir A4 jest nieskoczony. Std dalej wynika, e zbir A5
jest nieskoczony, itd. 
oooooooooooooooooooooooooooooooooooooooooooooooooooooooooooooooooooooo

9.5

Przykady liczb naturalnych nalecych do A3

oooooooooooooooooooooooooooooooooooooooooooooooooooooooooooooooooooooo
W 1964 roku Wacaw Sierpiski ([S64] 136-138) napisa, e nie wiadomo czy czy liczba 4
naley do A3 . Dzisiaj ju wiadomo, e nie naley. Udowodni to w 2000 roku A.V. Bondarenko
([Bond]). On udowodni nawet wicej:
9.5.1 (Bondarenko 2000). Kada liczba naturalna postaci 4m2 , gdzie 3 - m, nie naley do
zbioru A3 . ([Bond], patrz 9.8.7).
W tym podrozdziale stosowa bdziemy nastpujc terminologi. Zamy, e liczba
naturalna n naley do zbioru A3 . Istnieje wtedy trjka (x,y,z) liczb naturalnych takich, e
n = xy + yz + xz oraz nwd(x, y, z) = 1. Trjki (y, z, x) i (z, x, y) maj wwczas te same wasnoci.
Z tych trzech trjek wybierzmy t, ktra na pierwszym miejscu ma liczb rwn min{x, y, z}.
Tak trjk nazywa bdziemy -trjk liczby n.

74

Liczby wymierne.

9. Liczby postaci x1 /x2 + x2 /x3 + + xs /x1


3

+c
Wiemy, e jeli n A3 , to istnieje trjka (a, b, c) liczb naturalnych takich, e n = a +b
.
abc
W tym przypadku moemy zakada, e a 6 b 6 c oraz, e liczby a, b, c s parami wzgldnie
pierwsze. Kad trjk o tych wasnociach nazywa bdziemy -trjk liczby n. -Trjki
oznacza bdziemy przy pomocy zwykych nawiasw. Natomiast -trjki przy pomocy nawiasw kwadratowych. Liczba naturalna moe posiada wicej ni jedn -trjk. Podobnie
jest z -trjkami.

9.5.2. Jli [a, b, c] jest -trjk liczby naturalnej n, to trjki (ac2 , ba2 , cb2 ) i (ab2 , ca2 , bc2 ),
po cyklicznym przestawieniu najmniejszej liczby na pierwsze miejsce, tworz -trjki liczby n. Dla przykadu, z -trjki [1, 2, 9] liczby 41 otrzymujemy dwie rne -trjki liczby 41;
mianowicie (2, 36, 81) i (4, 9, 162).

9.5.3 (Rusin 2003). Istnieje dokadnie 57 liczb naturalnych n mniejszych od 200, dla ktrych
rwnanie xy + yz + xz = n posiada rozwizanie w zbiorze liczb naturalnych. S to nastpujce
liczby:
3, 5, 6, 9, 10, 13, 14, 17, 18, 19, 21, 26, 29, 30, 38,
41, 51, 53, 54, 57, 66, 67, 69, 73, 74, 77,
83, 86, 94, 101, 102, 105, 106, 110, 113, 117,
122, 126, 129, 130, 133, 142, 145, 147, 149, 154, 158,
161, 162, 166, 174, 177, 178, 181, 186, 195, 197.

Dla kadej z tych liczb, oprcz liczb 3 i 5, rozwaane rwnanie posiada nieskoczenie wiele
3
3 +z 3
rozwiza (x, y, z) takich, e nwd(x, y, z) = 1. To samo dotyczy rwnania x +y
= n.
xyz
([Rus1]).

9.5.4 (Maple). Pewne liczby naturalne n 6 200 nalece do A3 wraz z ich pewnymi trjkami.
(3) (1, 1, 1);
(5) (1, 2, 4);
(6) (2, 12, 9), (3, 18, 4);
(9) (12, 63, 98), (18, 28, 147);
(10) (175, 882, 1620);
(14) (28, 637, 338), (52, 1183, 98);
(19) (5, 225, 81), (9, 405, 25);
(41) (2, 36, 81), (4, 9, 162), (5, 350, 196), (14, 980, 25);
(53) (28, 1323, 1458);
(66) (3, 126, 196), (9, 14, 588);
(106) (35, 66150, 2916), (64, 102060, 1225);
(149) (14, 8820, 2025), (45, 28350, 196);
(154) (52, 10647, 7938).

Liczby wymierne.

9. Liczby postaci x1 /x2 + x2 /x3 + + xs /x1

75

9.5.5 (Rusin 2003, [Rus2]). Wszystkie liczby naturalne n 6 200 nalece do A3 wraz z ich
pewnymi -trjkami.
(3) [1, 1, 1];
(5) [1, 1, 2];
(6) [1, 2, 3], [1817, 3258, 5275], [4904676969, 10840875082, 15051171563];
(9) [2, 3, 7], [970703, 2982043, 4461282];
(10) [5, 7, 18], [4192875343, 11021882957, 19765145610];
(13) [9, 13, 38], [2197345737653, 6384056084353, 12689495542854];
(14) [2, 7, 13], [279025573, 759054842, 1638591583];
(17) [5, 18, 37], [1932849997397, 7649960172210, 14857581287413];
(18) [13, 42, 95], [5902844861231317, 35013190193908290, 54059017558123943];
(19) [1, 5, 9], [728051, 1279935, 4135819];
(21) [2, 13, 21], [38304582498, 44899033717, 187979061005];
(26) [9, 38, 91], [2592527851712161, 16461714780091854, 31072284713059955];
(29) [27, 43, 182], [725188306504448123, 2863730199603918763, 7554216031389795222];
(30) [2, 21, 31], [907576024698, 2555537666039, 8213238158509];
(38) [70, 151, 629];
(41) [1, 2, 9], [1, 5, 14], [2, 31, 43], [61, 1133, 1314], [1541, 10690, 25029], [13547, 17314, 97663],
[11441, 86425, 192834], [240322, 681959, 2567203], [193669, 2829857, 4119086];
(51) [9, 13, 77], [9496944543173, 28497283786885, 116604793962657];
(53) [2, 7, 27], [210121627, 5309015927, 5755076082];
(54) [2, 43, 57], [370030298454, 3412808117911, 7948993687541];
(57) [19, 91, 310], [278307036741995371, 5726573130751998070, 8251115886938879299];
(66) [1, 3, 14], [55075, 1201649, 1852326];
(67) [1133, 7525, 23517];
(69) [2, 57, 73], [42, 95, 523], [38808119, 45866266, 349822755],
[907290117, 16844207218, 29911475693], [11708394650, 69802887831, 234380785219];
(73) [89200900157319, 1391526622949983, 2848691279889518];
(74) [133, 2502, 4607];
(77) [67, 630, 1763], [133, 1382, 3665], [40225, 221062, 819413], [401247, 1986038, 7768135],
[8123011655, 138755312182, 277792875423], [321489851593, 5998665668870, 11380945916077];
(83) [5, 9, 61], [406164641531, 2343744686659, 8805786469335];
(86) [2, 73, 91], [5660399432462138, 114038591571428467, 220904967896959585];
(94) [27, 182, 673], [19, 746, 945], [20400692347, 64738300490, 351211722633],
[180053104598, 478460823507, 2838538679977];
(101) [79, 1271, 3078];
(102) [459338480695732254, 3816006884967068935, 13212742329826830581];
(105) [2, 91, 111], [35, 1171, 1854], [4934775, 86738143, 204325982],
[22107891903, 239901074434, 733520068619];

76

Liczby wymierne.

9. Liczby postaci x1 /x2 + x2 /x3 + + xs /x1

(106) [1, 35, 54], [1342, 15929, 46683], [100054843, 4555645497, 5608864334],
[327256085169, 710839851638, 4957711976947];
(110) [1147, 2745, 18578];
(113) [345842, 6313383, 15170275];
(117) [545, 1677, 10318];
(122) najmniejsza -trjka jest bardzo duga;
(126) [2, 111, 133], [1093, 4199, 23982], [843543, 6610037, 26297374],
[1437546238546, 8374810124997, 38751798984143];
(129) [31, 774, 1679], [70, 629, 2361], [11970393, 28883125, 210898982],
[11235206, 80300179, 338895771];
(130) najmniejsza -trjka jest bardzo duga;
(133) najmniejsza -trjka jest bardzo duga;
(142) najmniejsza -trjka jest bardzo duga;
(145) [44634584148027469, 157591646586434781, 1007950541819512850];
(147) [21, 925, 1529];
(149) [1, 14, 45], [2, 133, 157], [45257, 87913, 769298], [1261745, 32670622, 75361293],
[3617906033, 3624015553, 44175121682], [155269296833, 1140528906910, 5108338064637];
(154) [2, 13, 63], [62, 1183, 3285], [94550101, 2427158214, 5731153295],
[394133054, 847190695, 7164362061], [689430032438, 33597986722807, 53457471559053];
(158) [5642215349875, 7336556299898, 80828288788977];
(161) [11, 38, 259], [109, 3933, 7826], [146, 6517, 11349], [39927179, 179799907, 1072531846],
[124939654, 330937307, 2577222931], [8233174563067629, 444973594136388818, 678169113751189021];
(162) [35, 1854, 2881];
(166) [9, 611, 790];
(174) [5, 7, 78], [2, 157, 183], [608242, 46497117, 55872983], [65441098, 3473800847, 5671311957],
[455934805, 600648279, 6899701406], [28379531231, 73248432669, 600842282950];
(177) istnieje przypuszczenie, e najmniejsza -trjka jest bardzo duga;
(178) [2, 27, 97], [14392834313297, 170840056879242, 655798399654747];
(181) [10672860536839861, 21088064331923949, 201705586625136962];
(186) [2269, 15938, 81711], [11403, 22774, 219641], [5246451, 376524257, 513247054],
[145592437, 3886992711, 9968391914], [2334061450181786, 54431784269157829, 150189528932685207];
(195) [7, 15, 143], [39, 703, 2279], [12303811, 814494411, 1230936587],
[2114540363, 8501821579, 59118924099], [109899928179, 4982656556915, 9696551744971];
(197) [127, 6278, 11655].

9.5.6 (Maple). Pewne liczby naturalne 200 < n 6 500 nalece do A3 wraz z ich pewnymi
i -trjkami.
(201) [2, 183, 211];
(209) [5, 254, 481];
(230) [2, 211, 241];

Liczby wymierne.

9. Liczby postaci x1 /x2 + x2 /x3 + + xs /x1

77

(237) (65, 481650, 12996); [1, 65, 114];


(243) [9, 77, 409];
(250) (36, 5427, 8978); [2, 9, 67];
(261) (63, 3626, 16428); [2, 241, 273], [3, 7, 74], [18, 637, 1685];
(269) (14, 11956, 3721), (61, 52094, 196); [1, 14, 61];
(294) [2, 273, 307];
(323) [9, 49, 377], [13, 869, 1813];
(326) [5, 14, 151];
(329) [2, 307, 343];
(339) [7, 543, 1067];
(366) [2, 343, 381];
(405) [2, 381, 421];
(413) [62, 3285, 8953];
(446) [2, 421, 463];
(451) [23, 31, 567];
(478) [13, 23, 378];
(489) [2, 463, 507].

oooooooooooooooooooooooooooooooooooooooooooooooooooooooooooooooooooooo

9.6

Wystpowanie danej liczby w rozkadach liczb ze zbioru A3

oooooooooooooooooooooooooooooooooooooooooooooooooooooooooooooooooooooo
9.6.1. Jeli r jest liczb naturaln, to istnieje tylko skoczenie wiele par (y, z) N2 takich,
e yr + yz + zr jest liczb naturaln.
D. Niech r bdzie ustalon liczb naturaln. Zamy, e y, z s liczbami naturalnymi takimi, e
r
y

(1)

y
z

z
r

= n, gdzie n jest pewn liczb naturaln. Mamy wtedy rwno


r2 z + y 2 r + z 2 y = nryz,

z ktrej wynika, e y | r2 z oraz z | ry 2 . Niech r2 z = ay, ry 2 = bz, gdzie a, b N . Wtedy r3 y 2 =


r2 (ry 2 ) = r2 bz = bay, czyli r3 y = ab. Std r5 z = r3 (r2 z) = r3 (ay) = a2 b. Zatem y, z s liczbami
naturalnymi postaci
(2)

y=

ab
r3 ,

Wstawiajc to do (1) i mnoc stronami przez


(3)

r 13
a2 b ,

z=

a2 b
r5 .

otrzymujemy rwno

r10 + r8 b + a3 b2 = nr6 ab.

Z tej rwnoci wynika, e b jest podzielnikiem liczby r10 . Takich podzielnikw jest oczywicie tylko
skoczenie wiele. Niech r10 = ub. Mamy wtedy u + r8 + a3 b = nr6 a. Liczba a jest wic podzielnikiem
liczby u + r8 . Takich liczb a jest wic te tylko skoczenie wiele. Z rwnoci (2) wynika zatem, e
rozwaanych par (y, z) jest tylko skoczenie wiele. 
U. Z powyszego dowodu otrzymujemy algorytm na znajdowanie, dla danej liczby naturalnej r,
wszystkich trjek (x, y, z) N3 , w ktrych wystpuje liczba r i liczba xy + yz + xz jest naturalna.

78

Liczby wymierne.

9. Liczby postaci x1 /x2 + x2 /x3 + + xs /x1

Moemy przyj, e x = r. Wtedy liczby y, z otrzymujemy w nastpujcy sposb. Niech a bdzie


dowolnym podzielnikiem liczby r10 . Wtedy r10 = ua, gdzie u N. Niech b bdzie dowolnym podziela2 b
nikiem liczby r8 + u. Mamy wtedy skoczenie wiele liczb wymiernych y = ab
r 3 , z = r 5 . Wybieramy
tylko te, ktre s liczbami naturalnymi i speniaj warunek yr + yz + zr N. 

9.6.2. Niech n bdzie liczb naturaln i niech x, y, z bd liczbami naturalnymi takimi, e


n = xy + yz + xz . Jeli min{x, y, z} = 1, to n = 3 lub 5. W tych przypadkach mamy: 3 = 11 + 11 + 11 ,
5 = 12 + 42 + 41 .
D. Zamy, e x = 1.
(Sposb I). Niech a = nwd(x, y), b = nwd(y, z), c = nwd(z, x). Wtedy a = 1, b N oraz c = 1. Z
3
3
3
+c3
= 2+b
= 2b + b2 . Std wynika, e b = 1 lub b = 2.
twierdzenia 9.3.5 wiemy, e wtedy n = a +b
abc
b
Jeli b = 1, to n = 3. Jeli b = 2, to n = 5.
(Sposb II). Z rwnoci n = xy + yz + xz wynika rwno x2 z + y 2 x + z 2 y = nxyz, ktra w naszym
przypadku ma posta
z + y 2 + z 2 y = nyz.
Std ywnika, e y | z oraz z | y 2 . Niech z = uy, y 2 = vz, gdzie u, v N. Wtedy y 2 = vz = vuy i
std y = uv, z = u2 v. Zatem u2 v + u2 v 2 + u5 v 3 = nu3 v 3 i po podzieleniu stronami przez u2 v mamy:
1 + v + u3 v 2 = nuv i std v = 1. Zatem z = u2 = y 2 . Podstawiajc to do rwnoci z + y 2 + z 2 y = nyz,
otrzymujemy rwno 2 + y 3 = ny z ktrej wynika, e y | 2, Jeli y = 1, to n = 3. Jeli y = 2, to n = 5.


9.6.3. Niech n bdzie liczb naturaln i niech x, y, z bd liczbami naturalnymi takimi, e


n = xy + yz + xz oraz nwd(x, y, z) = 1. Jeli co najmniej jedna z liczb x, y, z jest rwna 2, to
2
9
2
36
81
n = 5, 6 lub 41. W tych przypadkach mamy: 5 = 42 + 41 + 12 , 6 = 12
+ 12
9 + 2 , 41 = 36 + 81 + 2 .
D. Zamy, e x = 2. Niech a = nwd(x, y), b = nwd(y, z), c = nwd(z, x). Wtedy a = 1 lub 2
oraz c = 1 lub 2. Przypadek a = c = 2 odpada, gdy nwd(x, y, z) = 1. Z twierdzenia 9.3.5 wiemy, e
3
3
3
3
+c3
wtedy n = a +b
. Moliwe s wic tylko przypadki: n = 2+b
= 2b + b2 lub n = 9+b
abc
b
2b . W pierwszym
przypadku 2 | b, wic b = 1 lub b = 2. Jeli b = 1, to x = y = z = 1 wbrew temu, e x = 2. Jeli b = 2,
to n = 5 = 42 + 41 + 12 . W drugim przypadku b = 1, 3 lub 9 i wtedy odpowiednio n = 5, 6 lub 41. 
Nastpne fakty otrzymano przy pomocy komputera i algorytmu opisanego w uwadze po
dowodzie twierdzenia 9.6.1.
9.6.4. Niech n bdzie liczb naturaln i niech x, y, z bd liczbami naturalnymi takimi, e
n = xy + yz + xz oraz nwd(x, y, z) = 1. Jeli co najmniej jedna z liczb x, y, z jest rwna 3, to
3
4
3
126
126
n = 6 lub 66. W tych przypadkach mamy: 6 = 18
+ 18
4 + 6 , 66 = 126 + 9 + 196 . (Maple).
9.6.5. Niech n bdzie liczb naturaln i niech x, y, z bd liczbami naturalnymi takimi, e
n = xy + yz + xz oraz nwd(x, y, z) = 1. Jeli co najmniej jedna z liczb x, y, z jest rwna 4, to
3
4
9
162
n = 5, 6 lub 41. W tych przypadkach mamy: 5 = 14 + 12 + 24 , 6 = 43 + 18
+ 18
4 , 41 = 9 + 162 + 4 .
(Maple).
9.6.6. Niech n bdzie liczb naturaln i niech x, y, z bd liczbami naturalnymi takimi, e
n = xy + yz + xz oraz nwd(x, y, z) = 1. Jeli co najmniej jedna z liczb x, y, z jest rwna 5, to
5
81
5
350
196
n = 19 lub 41. W tych przypadkach mamy: 19 = 225
+ 225
81 + 5 , 41 = 350 + 196 + 5 . (Maple).

9. Liczby postaci x1 /x2 + x2 /x3 + + xs /x1

Liczby wymierne.

79

9.6.7. Niech n bdzie liczb naturaln i niech x, y, z bd liczbami naturalnymi takimi, e


n = xy + yz + xz oraz nwd(x, y, z) = 1. Jeli co najmniej jedna z liczb x, y, z jest rwna 9,
2
+ 12
to n = 6, 19, 41, 66, 2369 lub 14 803. W tych przypadkach mamy: 6 = 29 + 12
9 , 19 =
405
25
9
162
4
9
14
588
9
11826
21316
9
405 + 25 + 9 , 41 = 162 + 4 + 9 , 66 = 14 + 588 + 9 , 2369 = 11826 + 21316 +
9 ,
9
29565
133225
14 803 = 29565 + 133225 + 9 . (Maple).
9.6.8. Niech x, y, z bd liczbami naturalnymi takimi, e xy + yz + xz jest liczb naturaln i
nwd(x, y, z) = 1. Wtedy kada z liczb x, y, z jest rna od 6, 7 i 9. (Maple).
9.6.9. Niech x, y, z bd liczbami naturalnymi takimi, e xy + yz + xz jest liczb naturaln i
nwd(x, y, z) = 1. Jeli ktra z liczb x, y, z jest mniejsza od 77, to moe ona by jedynie jedn
z liczb: 1, 2, 3, 4, 5, 9, 12, 14, 18, 20, 25, 28, 35, 36, 45, 50, 52, 54, 61, 63, 65. (Maple).
oooooooooooooooooooooooooooooooooooooooooooooooooooooooooooooooooooooo

9.7

Zbir B3

oooooooooooooooooooooooooooooooooooooooooooooooooooooooooooooooooooooo
Przypomnijmy, e
B3 =

y
z

x
y

x3 +y 3 +z 3
;
xyz

+ xz ; x, y, z N =

x
y

x3 +y 3 +z 3
;
xyz

y
z

+ xz ; x, y, z Q+ .

Oznaczmy:
C3 =

x, y, z N =

x, y, z Q+ .

Wiemy (patrz 9.3.2), e C3 = B3 .


Niech q Q+ . Podobnie jak w poprzednim podrozdziale mwi bdziemy, e (x, y, z) jest
-trjk liczby q, jeli: x, y, z N, nwd(x, y, z) = 1, x = min{x, y, z} oraz q = xy + yz + xz .
Kada dodatnia liczba wymierna naleca do B3 ma oczywicie co najmniej jedn -trjk.
Mwi bdziemy, e [a, b, c] jest -trjk liczby q jeli: a, b, c s liczbami naturalnymi,
3
3 +c3
a 6 b 6 c, nwd(a, b, c) = 1 oraz q = a +b
. Jeli q posiada -trjk, to oczywicie posiada
abc
-trjk i odwrotnie.
Niech [a, b, c] bdzie -trjk liczby wymiernej q. Wwczas nwd(a, b, c) = 1. W przypadku,
gdy q jest liczb naturaln, to std wynika, e liczby a, b, c s parami wzgldnie pierwsze.
Tak nie musi by jednak, gdy q nie jest liczb naturaln. Dla przykadu [1, 2, 2] jest -trjk
liczby 17
4 i liczby 1, 2, 2 nie s parami wzgldnie pierwsze. Nie znam odpowiedzi na nastpujce
pytanie.
9.7.1. Zamy, e liczba wymierna q posiada -trjk. Czy wtedy istnieje taka -trjka
[a, b, c] liczby q, e liczby a, b, c s parami wzgldnie pierwsze? (03.04.2007).
Podamy teraz przykady pewnych liczb wymiernych nalecych do B3 wraz z ich i
-trjkami. Wszystkie te przykady znaleziono przy pomocy Maple.
9.7.2 (Maple). Przykady liczb wymiernych postaci
lub -trjkami.
(7) (1, 1, 2), (1, 2, 2); [5, 7, 8];

n
2,

gdzie n N oraz 2 - n, wraz z ich

80

Liczby wymierne.

9. Liczby postaci x1 /x2 + x2 /x3 + + xs /x1

(11) (2, 3, 9), (2, 6, 9), [629, 1204, 1737];


(19) (4, 80, 25), (5, 100, 16); [1, 4, 5];
(33) (1, 4, 16); [1, 1, 4];
(37) (1, 3, 18), (1, 6, 18), (3, 90, 50), (5, 150, 9); [13, 72, 119], [63, 551, 604];
(41) [27, 155, 268];
(45) (3, 72, 64), (8, 192, 9); [1, 3, 8], [4, 5, 21], [63, 412, 695];
(49) (22, 1815, 450), (30, 2475, 121); [20, 37, 133];
(51) (1, 10, 25), (2, 5, 50);
(57) [7, 93, 104];
(61) [7, 104, 109];
(73) [7, 8, 45];
(85) [8, 117, 175];
(87) (4, 208, 169), (13, 676, 16); [1, 4, 13];
(97) (8, 1216, 361), (9, 1620, 400), (19, 2888, 64), (20, 3600, 81); [1, 8, 19], [1, 9, 20].

9.7.3 (Maple). Przykady liczb wymiernych postaci


lub -trjkami.
(10) (2, 4, 3), (3, 6, 4); [62, 81, 91];
(13) (1, 1, 3), (1, 3, 3); [7, 13, 15];
(16) (12, 45, 50), (18, 20, 75); [2, 3, 5];
(17) (1, 6, 4), (1, 15, 25), (2, 12, 3); [627, 818, 1547];
(19) (3, 4, 16), (3, 12, 16);
(20) (5, 50, 12), (6, 60, 25);
(23) (3, 36, 16), (4, 48, 9); [1, 3, 4];
(29) (1, 3, 9); [1, 1, 3];
(38) (1, 2, 12), (1, 6, 12);
(40) (6, 252, 49), (7, 294, 36); [1, 6, 7];
(41) (8, 320, 75), (15, 600, 64);
(44) (36, 208, 507); [3, 4, 13];
(53) (3, 63, 49), (7, 147, 9), [1, 3, 7];
(56) (15, 400, 256);
(62) (6, 44, 121), (12, 33, 242);
(65) [31, 37, 156];
(70) [14, 61, 135];
(74) (7, 588, 144), (12, 1008, 49); [1, 7, 12];
(77) (3, 5, 75);
(79) [57, 527, 776];
(85) [5, 11, 39];
(89) (33, 3509, 841);
(92) [28, 67, 237].

n
3,

gdzie n N oraz 3 - n, wraz z ich

Liczby wymierne.

9. Liczby postaci x1 /x2 + x2 /x3 + + xs /x1

9.7.4 (Maple). Przykady liczb wymiernych postaci


z ich lub -trjkami.

n
4,

81

gdzie n N oraz nwd(n, 4) = 1, wraz

(17) (1, 4, 2); [1, 2, 2];


(21) (1, 1, 4), (1, 4, 4); [3, 7, 8];
(27) (12, 126, 49), (14, 147, 36);
(29) (2, 24, 9), (3, 36, 8), (4, 5, 25), (4, 20, 25); [7, 8, 19];
(35) (1, 2, 8), (1, 4, 8);
(45) [8, 19, 39];
(69) (6, 45, 100), (8, 576, 81), (9, 20, 150), (9, 648, 64); [1, 8, 9], [8, 39, 67];
(75) (1, 12, 18), (2, 3, 36).

9.7.5 (Maple). Przykady liczb wymiernych postaci


lub -trjkami.

n
5,

gdzie n N oraz 5 - n, wraz z ich

(18) (36, 80, 75); [3, 4, 5];


(19) (2, 4, 5), (3, 9, 5), (4, 5, 10), (5, 15, 9); [4, 5, 7], [5, 7, 9], [333, 551, 595], [385, 589, 698];
(28) (4, 15, 18), (10, 12, 45), (15, 72, 64); [13, 35, 36], [54, 133, 155];
(29) [9, 25, 26];
(31) (1, 1, 5), (1, 5, 5), (9, 20, 48), (15, 36, 80); [11, 31, 35], [14, 37, 45], [95, 189, 292];
(32) [14, 43, 45];
(33) (1, 10, 4), (2, 20, 5); [86, 175, 279];
(34) (20, 175, 98), (28, 245, 50); [2, 5, 7];
(36) [7, 9, 20];
(39) [7, 15, 26];
(41) (5, 6, 36), (5, 30, 36); [35, 36, 97];
(44) [37, 140, 171];
(51) (3, 45, 25), (5, 75, 9); [1, 3, 5];
(54) [40, 147, 221];
(56) (2, 60, 9), (3, 90, 20);
(57) (5, 150, 36), (6, 180, 25); [1, 5, 6], [9, 20, 43];
(59) (30, 612, 289);
(62) (4, 48, 45), (15, 180, 16);
(67) (2, 20, 25), (4, 5, 50), (5, 175, 49), (7, 245, 25); [1, 5, 7], [37, 55, 161], [40, 221, 283];
(72) [8, 19, 45], [15, 26, 73];
(83) (3, 180, 16), (4, 240, 45); [37, 209, 315];
(87) (10, 52, 169), (20, 65, 338);
(88) [7, 54, 65];
(89) [50, 91, 279];
(96) (7, 490, 100), (10, 700, 49); [1, 7, 10], [5, 11, 28].

82

Liczby wymierne.

9. Liczby postaci x1 /x2 + x2 /x3 + + xs /x1

9.7.6 (Maple). Przykady liczb wymiernych postaci


z ich lub -trjkami.

n
6,

gdzie n N oraz nwd(n, 6) = 1, wraz

(19) (2, 2, 3), (2, 3, 3); [16, 19, 21];


(23) (1, 3, 2), (2, 6, 3); [79, 108, 143];
(25) (1, 2, 3), (2, 3, 6); [108, 143, 211];
(31) (1, 6, 2), (1, 6, 3), (6, 10, 25), (6, 15, 25), (36, 112, 147); [3, 4, 7], [11, 27, 28], [223, 380, 567];
(41) (1, 2, 6), (1, 3, 6); [380, 567, 1123];
(43) (1, 1, 6), (1, 6, 6); [13, 43, 48];
(47) (5, 75, 18), (6, 90, 25);
(55) (6, 7, 49), (6, 42, 49);
(59) (1, 6, 9), (2, 3, 18).

9.7.7 (Maple). Przykady liczb wymiernych postaci


lub -trjkami.

n
7,

gdzie n N oraz 7 - n, wraz z ich

(22) (9, 14, 12), (18, 28, 21); [7, 8, 9];


(23) (14, 20, 25), (28, 35, 50);
(30) (4, 16, 7), (7, 28, 16); [388, 629, 819];
(32) (2, 4, 7), (4, 7, 14);
(33) [3, 5, 7];
(38) (35, 150, 36);
(40) [19, 45, 56];
(41) (4, 21, 18), (12, 63, 14), (35, 275, 121); [5, 7, 13], [26, 31, 63];
(52) [26, 63, 97];
(53) (1, 14, 4), (2, 28, 7);
(57) (1, 1, 7), (1, 7, 7); [5, 19, 21];
(59) [45, 56, 139];
(60) (28, 441, 162), (36, 567, 98); [2, 7, 9], [12, 13, 35];
(71) (3, 18, 28), (7, 8, 64), (7, 56, 64), (9, 14, 84);
(72) [19, 21, 62];
(73) [7, 27, 38];
(83) [9, 26, 49];
(97) (2, 84, 9), (3, 126, 28), (5, 75, 63), (21, 315, 25); [99, 533, 721];
(99) [12, 35, 73].

9.7.8. Niech q = n +n+1


, gdzie n N. Wtedy (1, 1, n) jest -trjk liczby q oraz q =
n
gdzie a = 2n + 1, b = n2 + n + 1, c = n2 + 2n.

a3 +b3 +c3
,
abc

Liczby wymierne.

9. Liczby postaci x1 /x2 + x2 /x3 + + xs /x1

83

9.7.9 (M.Klamkin, [Crux] 2001 s.78). Jeli a, b, c s dugociami bokw trjkta, to


3 min

a
b

+ cb + ac ,

a
c

+ cb +

b
a

> (a + b + c)

1
a

1
b

1
c

oooooooooooooooooooooooooooooooooooooooooooooooooooooooooooooooooooooo

9.8

Liczby postaci x/y + y/z + z/x, gdzie x, y, z s liczbami cakowitymi

oooooooooooooooooooooooooooooooooooooooooooooooooooooooooooooooooooooo
Przedstawione tu fakty pochodz gwnie z pracy [Rus1].
Niech n bdzie liczb naturaln. Interesowa nas bdzie problem istnienia rozwiza rwnania
x
y

(I)

y
z

z
x

=n

w zbiorze niezerowych liczb cakowitych. Po pomnoeniu obu stron przez xyz, rwnanie to
przyjmuje posta
x2 z + y 2 x + z 2 y = nxyz.

(II)

Wiemy (patrz 9.3.1), e problem istnienia rozwiza rwnania (II) w zbiorze nieujemnych
liczb cakowitych sprowadza si do analogicznego problemu dla rwnania
x3 + y 3 + z 3 = nxyz.

(III)

Zajmiemy si wic problemem istnienia rozwiza rwnania (III) w zbiorze niezerowych


liczb cakowitych. Poniewa rwnanie (III) jest jednorodne, wic wystarczy zbada problem
istnienia rozwiza rwnania
x3 + y 3 + 1 = nxy

(IV )

w zbiorze niezerowych liczb wymiernych.


9.8.1 (Rusin 2003). Niech x =
muje posta
(V )

nX36+Y
2(3X+4n2 )

, y=

(nX+36+Y )
2(3X+4n2 ) .

Wtedy rwnanie (IV ) przyj-

Y 2 = X 3 + n2 X 2 72nX 16(4n3 + 27).

Jeli n 6= 3, to dane przeksztacenie jest odwracalne. Przeksztacenie odwrotne ma posta


2 (x+y)+9)
4(n3 27)(xy)
X = 4(n
(3(x+y)+n) , Y = (3(x+y)+n) . ([Rus1]).
Przy pomocy powyszego faktu D. Rusin ([Rus1]) sprowadzi problem istnienia rozwiza rwnia (I) w zbiorze niezerowych liczb cakowitych do badania struktury grupy krzywej
eliptycznej zadanej rwnaniem (V ). Dziki temu Rusin otrzyma nastpujce wyniki.
9.8.2 (Rusin 2003). Niech n bdzie liczb naturaln rn od 5. Jeli rwnanie xy + yz + xz = n
posiada rozwizanie w zbiorze nieujemnych liczb cakowitych, to posiada nieskoczenie wiele
prymitywnych takich rozwiza tzn. z warunkiem nwd(x, y, z) = 1. ([Rus1]).

84

9. Liczby postaci x1 /x2 + x2 /x3 + + xs /x1

Liczby wymierne.

9.8.3 (J.W.S. Cassels 1960). Rwnanie


([Mat] 2/61 68, [Rus1]).

x
y

y
z

z
x

= 1 nie posiada rozwiza cakowitych.

Zanotujmy przy okazji:


9.8.4. Nastpujce warunki s rwnowane.
(1) Rwnanie xy + yz + xz = 1 ma rozwizanie cakowite.
(2) Istniej liczby wymierne u, v, w takie, e u + v + w = uvw = 1.
(3) Istniej liczby cakowite a, b, c takie, e a3 + b3 + c3 = abc 6= 0.
(4) Istniej liczby cakowite a, b, c takie, e (a + b + c)3 = abc 6= 0.
(5) Istnieje liczba wymierna a taka, e rwnanie x3 x2 + ax 1 = 0 ma trzy pierwiastki
wymierne. ([Mat] 3/57 11-13, 1/58 57).
9.8.5. Rwnanie

x
y

y
z

z
x

= 2 nie posiada rozwiza cakowitych.

([Rus1]).

9.8.6. Rwnanie xy + yz + xz = 3 posiada nieskoczenie wiele prymitywnych rozwiza w


zbiorze niezerowych liczb cakowitych. Wrd tych rozwiza tylko jedno jest w zbiorze liczb
naturalnych, mianowicie x = y = z = 1. ([Rus1]).
9.8.7 (Rusin 2003). Rwnanie
([Rus1], porwnaj 9.5.1).

x
y

y
z

z
x

= 4 nie posiada rozwiza cakowitych.

9.8.8 (Rusin 2003). Istnieje dokadnie 111 liczb naturalnych n mniejszych od 200, dla ktrych rwnanie xy + yz + xz = n posiada rozwizanie w zbiorze niezerowych liczb cakowitych.
S to nastpujce liczby:
3, 5, 6, 9, 10, 13, 14, 15, 16, 17, 18, 19, 20, 21, 26, 29, 30, 31, 35, 36, 38, 40,
41, 44, 47, 51, 53, 54, 57, 62, 63, 64, 66, 67, 69, 70, 71, 72, 73, 74, 76, 77,
83, 84, 86, 87, 92, 94, 96, 98, 99,
101, 102, 103, 105, 106, 107, 108, 109, 110, 112, 113, 116, 117, 119, 120,
122, 123, 124, 126, 127, 128, 129, 130, 132, 133, 136,
142, 143, 145, 147, 148, 149, 151, 154, 155, 156, 158, 159, 160,
161, 162, 164, 166, 167, 172, 174, 175, 177, 178,
181, 185, 186, 187, 189, 190, 191, 192, 195, 196, 197.

Dla kadej z tych liczb, oprcz liczby 5, rozwaane rwnanie posiada nieskoczenie wiele
3
3 +z 3
rozwiza prymitywnych. To samo dotyczy rwnania x +y
= n. ([Rus1]).
xyz
9.8.9 (Rusin 2003). Rwnanie xy + yz + xz = 112 posiada rozwizanie w zbiorze niezerowych
liczb cakowitych. Najprostszym rozwizaniem jest
x =
y
z

444882220325179840803472420042062236091767720844845203037
340381653808676781078204185344064777425
= 1800010639340561476631947037621286947915240684971323481
294582383858472523311320365128373281158
= 13318091576854113300162838591657841686993519959959149
070559988026538909081959649861205201860.

Wystpuj tu liczby majce okoo 90 cyfr. Podobna sytuacja ma miejsce, gdy zamiast liczby
112 rozpatrzymy liczby 122, 130, 133, 142, 164, 177, 187 i 190. ([Rus1]).

Liczby wymierne.

9. Liczby postaci x1 /x2 + x2 /x3 + + xs /x1

85

Udowodnilimy (patrz 9.3.6), e jeli x, y, z s liczbami naturalnymi takimi, e liczba


jest naturalna, to xyz jest szecianem liczby naturalnej. Co si stanie, gdy rozwaymy
ten sam problem w przypadku, gdy x, y, z s niezerowymi liczbami cakowitymi? Zanotujmy:
x y z
y +z +x

9.8.10. Niech x, y, z bd niezerowymi liczbami cakowitymi takimi, e xy + yz + xz jest liczb


cakowit. Czy wtedy xyz jest szecianem liczby cakowitej? (27.03.2007; nie znam odpowiedzi).
9.8.11. Niech a, b, c Z r {0}. Jeli
([OM] Bonia-Hercegowina 2005).

a
b

9.8.12. Jeeli liczby a, b, c, ab + cb + ac ,

+ cb + ac = 3, to abc jest szecianem liczby cakowitej.

a
c

+ cb +

b
a

s cakowite, to |a| = |b| = |c|.

9.8.13. Jeli x, y, z s niezerowymi liczbami cakowitymi takimi, e


liczb xy , yz , xz nie jest cakowita. ([Mat] 4/59 214).

x
y

y
z

([TTjs] 1995).

+ xz , to adna z

oooooooooooooooooooooooooooooooooooooooooooooooooooooooooooooooooooooo

9.9

Zbir A4

oooooooooooooooooooooooooooooooooooooooooooooooooooooooooooooooooooooo
Przypomnijemy, e A4 jest zbiorem wszystkich liczb naturalnych postaci xy + yz + zt + xt ,
gdzie x, y, z, t N. Wiemy (patrz 9.4.5), e zbir A4 jest nieskoczony.
9.9.1 (Maple). Przykady pewnych liczb naturalnych n 6 30 postaci n = xy + yz + zt + xt ,
gdzie x, y, z, t s liczbami naturalnymi. Dla kadej takiej liczby n podano jej pewne czwrki
(x, y, z, t) speniajce warunek nwd(x, y, z, t) = 1.
(4) (1, 1, 1, 1);
(5) (1, 2, 1, 2), (1, 2, 4, 2);
(6) (1, 1, 2, 4), (1, 2, 2, 4), (1, 6, 4, 3), (2, 10, 4, 5), (3, 18, 6, 4);
(7) (6, 45, 30, 25), (2, 2, 12, 9), (10, 12, 45, 50), (12, 45, 50, 60), (4, 48, 18, 9);
(9) (1, 3, 18, 4), (1, 6, 18, 4), (2, 6, 36, 9), (4, 40, 32, 5), (3, 18, 4, 12);
(10) (5, 150, 36, 9), (6, 12, 45, 50), (9, 18, 20, 75), (6, 180, 45, 25), (12, 45, 90, 100);
(11) (1, 2, 12, 9), (4, 48, 72, 9), (2, 60, 9, 5), (10, 372, 180, 31), (7, 294, 84, 36);
(12) (1, 35, 25, 7), (3, 45, 15, 25), (5, 175, 25, 7), (9, 63, 245, 75), (5, 75, 9, 15);
(13), (3, 36, 24, 32), (1, 10, 25, 10), (4, 40, 160, 25), (4, 16, 160, 25), (12, 18, 264, 121);
(14) (4, 5, 150, 36), (2, 10, 20, 25), (4, 9, 162, 18), (4, 20, 25, 50), (1, 18, 4, 9);
(15) (2, 105, 50, 21), (1, 10, 25, 2), (10, 145, 50, 116), (6, 468, 54, 13), (8, 160, 175, 98);
(17) (3, 126, 18, 28), (9, 22, 242, 132), (4, 9, 162, 12), (2, 36, 81, 6), (9, 22, 12, 132);
(18) (4, 60, 225, 54), (10, 350, 28, 49), (4, 140, 245, 50), (10, 150, 36, 135), (4, 15, 225, 54);
(19) (10, 42, 180, 175), (4, 240, 72, 5), (9, 14, 245, 150), (2, 396, 121, 18), (3, 180, 144, 10);
(20) (5, 5, 225, 81), (9, 405, 25, 25), (9, 405, 25, 9), (5, 225, 81, 5), (9, 405, 405, 25);
(21) (3, 198, 36, 44), (2, 132, 9, 11), (6, 252, 63, 98), (6, 396, 484, 33), (1, 4, 48, 18);
(22) (4, 39, 169, 78), (2, 60, 9, 30), (12, 20, 75, 250), (1, 30, 100, 15), (5, 450, 500, 36);

86

9. Liczby postaci x1 /x2 + x2 /x3 + + xs /x1

Liczby wymierne.

(23) (3, 90, 15, 50), (5, 450, 324, 18), (4, 336, 18, 7), (1, 30, 100, 6), (10, 100, 375, 18);
(25) (4, 5, 150, 90), (6, 396, 99, 121), (4, 240, 18, 45), (3, 90, 4, 5), (6, 180, 225, 10);
(26) (5, 450, 150, 108), (4, 35, 294, 90), (5, 75, 18, 108), (5, 75, 450, 108), (3, 495, 450, 44);
(27) (4, 55, 242, 10), (4, 420, 147, 90), (7, 15, 126, 180), (7, 10, 84, 180), (2, 372, 36, 31);
(28) (1, 3, 9, 27), (3, 369, 27, 41);
(29) (1, 18, 12, 27), (3, 114, 36, 76), (2, 36, 81, 3), (4, 9, 162, 6), (4, 9, 162, 108);
(30) (2, 84, 252, 49), (4, 55, 242, 110), (10, 44, 20, 275), (4, 180, 25, 90), (2, 6, 252, 49).

9.9.2 (Maple). Liczby naturalne n 6 100 nalece do zbioru A4 .


4, 5, 6, 7, 9, 10, 11, 12, 13, 14, 15, 17, 18, 19,
20, 21, 22, 23, 25, 26, 27, 28, 29, 30, 31, 33, 35, 36, 37, 38, 39,
41, 42, 43, 45, 47, 49, 50, 51, 52, 54, 55, 57,
61, 62, 63, 67, 68, 70, 71, 73, 75, 76, 77,
82, 84, 90, 91, 93, 97, 98.

Nie wiem czy to s wszystkie liczby naturalne (mniejsze od 100) o tej wasnoci.
Z oblicze przy pomocy Maple nasuwaj si nastpujce pytania, na ktre nie znam
odpowiedzi.
9.9.3. Czy prawd jest, e jeli n A4 , to 8 - n?

(31.03.2007).

9.9.4. Wiadomo, e liczby 7, 10, 17, 18, 19, 20, 25, 26, 27, 30 s postaci xy + yz + zt + xt , gdzie
x, y, z, t N. Czy dla tych liczb istniej takie czwrki (x, y, z, t), e co najmniej jedna z liczb
x, y, z, t jest rwna 1? (31.03.2007).

9.9.5. Rozpatrzmy rwnanie

x
y

y
z

z
t

t
x

= m.

(1) Jeli m = 1, to rwnanie to nie ma naturalnych rozwiza. Ma natomiast nieskoczenie wiele rozwiza w zbiorze niezerowych liczb cakowitych.
(2) Dla m = 2 i m = 3 nie ma rozwiza naturalnych.
(3) Jeli m = 4, to kade naturalne rozwizanie jest postaci (n, n, n, n), n N.
([Mat] 3/57 13, [S64] 141-142).

Wiemy (patrz 9.3.4), e zbir A3 pokrywa si ze zbiorem wszystkich liczb naturalnych


3
3 +c3
postaci a +b
, gdzie a, b, c N. Czy co podobnego zachodzi dla liczb naturalnych naleabc
4
4 +c4 +d4
cych do zbioru A4 ? Wiemy (patrz 9.1.7), e kada liczba naturalna postaci a +babcd
,
gdzie a, b, c, d N, naley do zbioru A4 . Czy kad liczb naturaln ze zbioru A4 mona tak
przedstawi? Udowodnimy, e tak nie jest.
9.9.6. Liczba 5 naley do zbioru A4 i nie jest postaci

a4 +b4 +c4 +d4


,
abcd

gdzie a, b, c, d N.

9. Liczby postaci x1 /x2 + x2 /x3 + + xs /x1

Liczby wymierne

87

1
2
1
2
2 + 1 + 2 + 1 , wic 5 A4 . Przypumy, e istniej liczby naturalne a, b, c, d
4
a +b +c +d
. Skracajc ewentualnie przez najwikszy wsplny dzielnik, moemy zaoy,
abcd

D. Poniewa 5 =
4

takie, e 5 =
e nwd(a, b, c, d) = 1. Mamy wic rwno

a4 + b4 + c4 + d4 = 5abcd.
Jeli liczba cakowita u nie jest podzielna przez 5, to (na mocy maego twierdzenia Fermata)
u4 1 (mod 5).
W naszym przypadku a4 + b4 + c4 + d4 0 (mod 5). Kada wic z liczb a, b, c, d musi by podzielna
przez 5. Jest to jednak sprzeczne z tym, e nwd(a, b, c, d) = 1. 

88

Liczby wymierne

Literatura

Literatura
[B-rs] J. Browkin, J. Rempaa, S. Straszewicz, 25 lat Olimpiady Matematycznej, WSiP, Warszawa,
1975.
[B-zm] V. I. Bernik, I. K. uk, O. W. Melnikow, Zbir zada olimpijskich z matematyki (po rosyjsku),
Narodnaja Aswieta, Minsk, 1980.
[BaL] I. W. Baranowa, C. E. Lapin, Zadania z algebry (po rosyjsku), Leningrad, 1954.
[Balt] Zawody Matematyczne Pastw Batyckich.
[Bedn] W. Bednarek, Zbir zada dla uczniw lubicych matematyk, Gdaskie Wydawnictwo Owiatowe, Gdask, 1995.
[BoL] W. G. Botiaski, W. G. Leman, Zbir zada moskiewskich olimpiad matematycznych (po
rosyjsku), Moskwa, 1965.
[Bond] A. V. Bondarenko, Investigation of a class of Diophantine equations, (po rosyjsku), Ukrain
Math. Zh. 52(6)(2000), 831-836.
[Br80] J. Browkin, Zadania z Olimpiad Matematycznych, tom 5, 21-25, 69/70 - 73/74, WSiP, Warszawa, 1980.
[Br83] J. Browkin, Zbir Zada z Olimpiad Matematycznych, tom 6, 26-30, 74/75 - 78/79, WSiP,
Warszawa, 1983.
[BrG] A. Bremner, R. K. Guy, Two more representation problems, Proc. Edin. Math. Soc., 40(1997),
1-17.
[Bryn] M. Bryski, Olimpiady Matematyczne, tom 7, 31-35, 79/80 - 83/84, WSiP, Warszawa, 1995.
[Cmj] College Mathematics Journal.
[Crux] Crux Mathematicorum, 1974-2008.
[Dic1] L. E. Dickson, History of the Theory of Numbers, Vol. I. Divisibility and primality, Carnegie
Institute of Washington, 1919. Reprinted by AMS Chelsea Publishing, New York, 1992.
[Dlt]

Delta, Popularny Miesicznik Matematyczno-Fizyczno-Astronomiczny.

[DoC] S. Doduniekow, K. Czakyrjan, Zadania z teorii liczb (po rosyjsku), Narodna Poswieta, Sofia,
1985.
[Dofs] E. Dofs, Solutions of x3 + y 3 + z 3 = nxyz, Acta Mathematica, 73(3)(1995), 201-213.
[Fom] D. W. Fomin, Sankt-Petersburskie Olimpiady Matematyczne (po rosyjsku), Politechnika, SanktPetersburg, 1994.
[G-if] S. A. Genkin, I. W. Itenberg, D. W. Fomin, Leningradzkie Kka Matematyczne (po rosyjsku),
Kirow, ASA, 1994.
[GaT] G. A. Galpierin, A. K. Topygo, Moskiewskie Olimpiady Matematyczne (po rosyjsku), 19351985, Moskwa, 1986.
[GeG] S. I. Gelfand, M. L.Gerwer, A. A. Kiryow, N. N. Konstantinow, A. G. Kusznirenko, Zadania
z elementarnej matematyki, Cigi, Kombinatoryka, Granice (po rosyjsku), Nauka, Moskwa,
1965.
[Gess] Ira M. Gessel, Wolstenholme revisited, Amer. Math. Monthly 104(1998)(7), 657-658.
[Gy04] R. K. Guy, Unsolved Problems in Number Theory, Third edition, Springer-Verlag, New York,
2004.

Liczby wymierne

Literatura

89

[Hass] H. Hasse, Number Theory, Springer-Verlag, 1980.


[HW4] G. H. Hardy, E. M. Wright, An introduction to the theory of numbers, Fourth edition, Oxford
at the Clarendon Press, 1960.
[Ibe]

Iberoamerican Mathematical Olympiad.

[IMO] Midzynarodowa Olimpiada Matematyczna.


[Isaa] I. M. Isaacs, Algebra, A Graduate Course, Brooks/Cole Publishing Company, Pacific Grove,
California, 1994.
[JaK] R. Jajte, W. Krysicki, Z matematyk za pan brat, Iskry, Warszawa, 1985.
[Jedr] P. Jdrzejewicz, Bukiety Matematyczne dla Gimnazjum, Gdaskie Wydawnictwo Owiatowe,
2002.
[Ko01] L. Kourliandtchik, Etiudy matematyczne, Tutor, Toru, 2000.
[Ko04] L. Kourliandtchik, Synne nierwnoci, Aksjomat, Toru, 2002.
[KoM] KoMal, Kozepiskolai Matematikai Lapok, Wgierskie czasopismo matematyczne, 1894-2008.
[Kurs] J. K
urschak, Wgierskie Olimpiady Matematyczne (po rosyjsku), MIR, Moskwa, 1976.
[Kw]

Kwant 1970-2008, czasopismo rosyjskie.

[LeH] H. Lee, Problems in Elementary Number Theory, Version 08, Internet 2003.
[Lion] F. Le Lionnais, Les nombres remarquables, Herman, Paris, 1983.
[M-sj] The Mathematics Student Journal.
[Mat] Matematyka, Czasopismo dla nauczycieli.
[Maza] W. Marzantowicz, P. Zarzycki, Elementarna Teoria Liczb, Wydawnictwo Naukowe PWN,
Warszawa, 2006.
[MG] The Mathematical Gazette.
[MM] Mathematics Magazine.
[Mon] The American Mathematical Monthly.
[Nar86] W. Narkiewicz, Classical Problems in Number Theory, Monografie Matematyczne 62, Warszawa, 1986.
[OM] Olimpiada Matematyczna.
[OMm] Maa Olimpiada Matematyczna.
[Oss]

Onatari Secondary School Mathematics Bulletin.

[Putn] Putnam (William Lowell) Mathematical Competition.


[RaT] H. Rademacher, O. Toeplitz, O liczbach i figurach, PWN, Warszawa, 1956.
[Ri97] P. Ribenboim, Maa ksiga wielkich liczb pierwszych, WNT, Warszawa, 1997.
[Rus1] D. Rusin, For which value of n is a/b + b/c + c/a = n solvable?, July, August 2003.
http://www.math.niu.edu/rusin/reserach-math/abcn/.
[Rus2] D. Rusin, Small solutions for a/b + b/c + c/a = n, July, August 2003.
http://www.math.niu.edu/rusin/reserach-math/abcn/smollsols,
[S-kg] W. A. Sadowniczij, A. A. Grigorjan, S. W. Konjagin, Zadania studenckich olimpiad matematycznych (po rosyjsku), Moskwa, 1987.

90

Liczby wymierne

[S50]

W. Sierpiski, Teoria Liczb, Warszawa - Wrocaw, 1950.

[S54]

W. Sierpiski, Trjkty pitagorejskie, PWN, Warszawa, 1954.

Literatura

[S57a] W. Sierpiski, O rozkadach liczb wymiernych na uamki proste, PWN, Warszawa, 1957.
[S59]

W. Sierpiski, Teoria Liczb II, PWN, Warszawa, 1959.

[S64]

W. Sierpiski, 200 zada z elementarnej teorii liczb, Biblioteczka Matematyczna 17, PZWS,
Warszawa, 1964.

[San4j] D. A. Santos, Junior Problem Seminar, Preprint, Internet 2004.


[Sand] J. Sandor, Geometric Theorems, Diophantine Equations, and Arithmetic Functions, American
Research Press, Rehoboth, 2002.
[ShCY] D. O. Shklarsky, N. N. Chentzov, I. M. Yaglom, The USSR Olympiad Problem Book,
W. F. Freeman and Company, San Francisco, London, 1962.
[Siw]

I. H. Siwaszinskij, Nierwnoci w zadaniach (po rosyjsku), Nauka, Moskwa, 1967.

[Szn]

L. B. Szneperman, Zbir zada z algebry i teorii liczb (po rosyjsku), Minsk, 1982.

[Szu87] M. Szurek, Opowieci matematyczne, WSiP, Warszawa, 1987.


[Tao] T. Tao, Solving Mathematical Problema, a personal perspective, Oxford, 2006.
[Tri]

Ch. Trigg, Mathematical Quickies, McGraw-Hill Book Company, New York-London, 1967.
Tumaczenie rosyjskie: Moskwa, 2000.

[TT]

Tournament of the Towns.

[TTjs] Tournament of the Towns, Junior, Spring.


[UsaT] USA Mathematical Talent Search.
[WaJ] N. B. Wasilew, A. A. Jegorow, Zadania Olimpiad Matematycznych Zwizku Radzieckiego (po
rosyjsku), 1961-1987, Moskwa, Nauka, 1988.
[Wino] I. Winogradow, Elementy teorii liczb, PWN, Warszawa, 1954.
[Wm] Wiadomoci Matematyczne, Roczniki Polskiego Towarzystwa Matematycznego, 1956-2008.
[WyKM] W. A. Wyszenskij, I. W. Kartaszow, W. I. Michaiowskij, M. I. Jadrenko, Zbir zada
Kijowskich Olimpiad Matematycznych (po rosyjsku), 1935-1983, Kijw, 1984.
[Zw]

Zwardo, Obz Naukowy Olimpiady Matematycznej.

Uniwersytet M. Kopernika, Wydzia Matematyki i Informatyki, Toru


Olsztyska Wysza Szkoa Informatyki i Zarzdzania, Olsztyn
e-mail: anow@mat.uni.torun.pl

Skorowidz
American Mathematical Monthly, 1
Anning N., 5
Apery, 37
Bonning A., 28
Balfour A., 42
Bibiloni L., 38
bikwadrat liczby cakowitej, 5, 37, 63, 86, 87
Boas R.P. Jr., 32
boki trjkta, 83
Bondarenko A.V., 3, 66, 73
Cassels J.W.S., 84
cig
arytmetyczny, 29, 30, 32, 38
Fibonacciego, 31
rekurencyjny, 12, 13
CoCoA, 1
Cohen R., 28
Crux Mathematicorum, 1
cyfry, 5, 38, 41, 42, 62, 63, 85
cz cakowita, 2, 31
cz uamkowa, 9
czwrka liczb naturalnych, 34, 36, 85, 86
Delta, 1
Derive, 1
Derksen H., 10
Dickson L.E., 65
Dofs E., 73
elipsa, 41
Erdos P., 71, 72
Frost P., 63
funkcja
, 2, 31, 41
, 7, 22, 23
z Q do Q, 10
z Q Q do Q, 10
zeta Riemanna, 37
Gessel I.M., 3
Glaisher, 63, 64
granica cigu, 37, 38
Guy R.K., 28
Hagedorn T.R., 28
hipoteza, 38, 59
Hoke O.H., 15

Hsia C., 41
Iboldin O.T., 16
iloczyn kartezjaski, 10
indukcja matematyczna, 10, 12, 16, 30, 31, 55
International Mathematical Olympiad, 1
kanoniczne przedstawienie, 49
Klamkin M., 83
Kline M., 38
kolejne liczby naturalne, 5, 6, 9, 2832, 34, 3638,
53, 57, 6165
krzywa eliptyczna, 83
Kurlandczyk L.D., 16
Kwant, 1
Leibniz, 37
liczba
, 37
m-przedstawialna, 44
bezkwadratowa, 17
dzielnikw naturalnych, 7, 22, 23
kwadratowa, 16, 18, 23, 28, 34, 35, 37, 62
nieparzysta, 5, 7, 9, 15, 22, 28, 46, 47, 58, 60,
61, 63, 64
niewymierna, 37, 42
parzysta, 14, 15, 29, 32, 41, 46, 59
pierwsza, 3, 8, 9, 22, 24, 32, 33, 40, 41, 49, 54,
55, 57, 6163, 65, 69
trjktna, 35, 36
wymierna, 3, 5
zero-jedynkowa, 5
liczby wzgldnie pierwsze, 9, 18, 20, 21, 23, 2629,
3133, 44, 47, 48, 52, 58, 6065, 69, 71,
72, 74, 79
logarytm, 30, 37
maksymalny element, 21, 28
Maple, 1
Matematical Excalibur, 1
Matematyka, 1
Mathematica, 1
Mathematical Gazette, 1
Mendes M., 42
Miksa F.L., 15
Moser L., 35
MuPad, 1
Nagara P.N., 15
Narkiewicz W., 28

91

92
nierwno, 12, 13, 17, 30, 31, 34, 3638, 57, 66
Niven I., 71, 72
nwd, 2, 36, 44, 5052, 71
nww, 2, 31, 32, 41, 5052
okres rozwinicia dziesitnego, 30, 3941
Ossowski P., 6163
para

Liczby wymierne

Skorowidz

Sniady P.W., 11
Strzelecki P., 38
suma szeregu, 37, 38
symbol Newtona, 54, 55, 65
system numeracji, 5
szecian liczby cakowitej, 3, 5, 7, 8, 36, 37, 62, 63,
7074, 78, 79, 82, 8486
szecian liczby wymiernej, 10

liczb cakowitych, 13
liczb naturalnych, 7, 8, 13, 1922, 26, 27, 33,
59, 60, 77
liczb wymiernych, 10
Paradis J., 38
pochodna wielomianu, 54
podpiercie, 11
podzbir, 911, 38
potga
dwjki, 6, 9, 13, 21, 30, 32, 33, 36, 47, 61, 64,
65
liczby pierwszej, 21, 22, 47, 65
liczby wymiernej, 10
trjki, 59
przedstawienie liczby naturalnej, 6, 7, 17
przedstawienie liczby wymiernej, 9, 1719
przekrj zbiorw, 10, 66, 69
Putnam Mathematical Competition, 1
pytanie, 3, 36, 79, 86

Toeplitz O., 41
trjka
liczb naturalnych, 13, 14, 17, 36, 73, 74, 76,
77, 7982
liczb wymiernych, 6
twierdzenie
Bezouta, 55
Bondarenko, 66, 73
Erdosa-Nivena, 71, 72
Gessela, 6064
mae Fermata, 55, 87
Rusina, 84
Wilsona, 55
Wolstenholme, 3, 61

rwnanie diofantyczne
n zmiennych, 12, 18, 3336
czterech zmiennych, 14, 24, 28, 34, 36, 86
dwch zmiennych, 7, 13, 1924
piciu zmiennych, 14
szeciu zmiennych, 14
trzech zmiennych, 15, 21, 23, 2528, 33, 35,
36, 74, 8385
Rademacher H., 41
Rao K.S., 41
relacja typu rwnowanoci, 45
rozkad kanoniczny, 22, 23, 4951, 71
rozwinicie dziesitne, 3, 30, 3842
Rusin D., 74, 75, 83, 84

Wadhwa A.D., 38
Wagutien W.N., 11
warunki rwnowane, 18, 22, 38, 43, 44, 67, 6971,
84
wielomian, 5355
Wilenkin N.J., 11
Wolstenholme J., 3, 61
Wrblewski J., 11
Wrench J.W. Jr., 32

Sandor J., 23, 28, 35


Sato N., 41
Schiller J.K., 41
Schinzel A., 18
Semionowa L., 41
Sierpiski W., 25, 28, 35, 73
silnia, 9, 13, 32, 54, 55, 57, 61, 62
Simmons H.A., 28
Smieszek I., 64

uamek egipski, 3
uamek prosty, 3, 12
Viader P., 38

Zarzycki P., 38
zbir
N0 , 1
gsty, 32
liczb cakowitych, 1
liczb naturalnych, 1
liczb pierwszych, 1
liczb rzeczywistych, 1
liczb wymiernych, 1, 5, 9
liczb zespolonych, 1, 7, 11
multyplikatywny, 11

You might also like